122
МИНИСТЕРСТВО СЕЛЬСКОГО ХОЗЯЙСТВА РОССИЙСКОЙ ФЕДЕРАЦИИ ДЕПАРТАМЕНТ НАУЧНО-ТЕХНОЛОГИЧЕСКОЙ ПОЛИТИКИ И ОБРАЗОВАНИЯ ФЕДЕРАЛЬНОЕ ГОСУДАРСТВЕННОЕ БЮДЖЕТНОЕ ОБРАЗОВАТЕЛЬНОЕ УЧРЕЖДЕНИЕ ВЫСШЕГО ПРОФЕССИОНАЛЬНОГО ОБРАЗОВАНИЯ «АЗОВО-ЧЕРНОМОРСКАЯ ГОСУДАРСТВЕННАЯ АГРОИНЖЕНЕРНАЯ АКАДЕМИЯ» Кафедра высшей математики М.Н. Середина, Л.Ю. Шипик МАТЕМАТИКА Практикум Учебное пособие Зерноград – 2014

М.Н. Середина, Л.Ю. Шипик МАТЕМАТИКАачии.рф/files/2018-11-16-7b4836c8-02e7-466b-aa5c...2018/11/16  · функции нескольких переменных,

  • Upload
    others

  • View
    1

  • Download
    0

Embed Size (px)

Citation preview

Page 1: М.Н. Середина, Л.Ю. Шипик МАТЕМАТИКАачии.рф/files/2018-11-16-7b4836c8-02e7-466b-aa5c...2018/11/16  · функции нескольких переменных,

МИНИСТЕРСТВО СЕЛЬСКОГО ХОЗЯЙСТВА РОССИЙСКОЙ ФЕДЕРАЦИИ ДЕПАРТАМЕНТ НАУЧНО-ТЕХНОЛОГИЧЕСКОЙ ПОЛИТИКИ И ОБРАЗОВАНИЯ

ФЕДЕРАЛЬНОЕ ГОСУДАРСТВЕННОЕ БЮДЖЕТНОЕ ОБРАЗОВАТЕЛЬНОЕ УЧРЕЖДЕНИЕ

ВЫСШЕГО ПРОФЕССИОНАЛЬНОГО ОБРАЗОВАНИЯ «АЗОВО-ЧЕРНОМОРСКАЯ ГОСУДАРСТВЕННАЯ АГРОИНЖЕНЕРНАЯ АКАДЕМИЯ»

Кафедра высшей математики

М.Н. Середина, Л.Ю. Шипик

МАТЕМАТИКА

Практикум

Учебное пособие

Зерноград – 2014

Page 2: М.Н. Середина, Л.Ю. Шипик МАТЕМАТИКАачии.рф/files/2018-11-16-7b4836c8-02e7-466b-aa5c...2018/11/16  · функции нескольких переменных,

2

УДК 517(075)

Печатается по решению методической комиссии по направлению подготовки 110400.62 − Агрономия ФГБОУ ВПО АЧГАА

Рецензенты:

канд. техн. наук, доцент Кравченко Л.В.; канд. хим. наук, доцент Посохова С.В.

Середина, М.Н. Математика. Практикум: учебное пособие / М.Н. Середина, Л.Ю. Шипик. – Зерноград: ФГБОУ ВПО АЧГАА, 2014. – 122 с.

Учебное пособие предназначено для студентов 1 курса очной и заочной

форм обучения высших учебных заведений, обучающихся по направлению подготовки 110400.62 − Агрономия.

Учебное пособие охватывает все разделы дисциплины «Математика», содержит теоретические сведения в кратком изложении, примеры решений всех типов задач, задания для аудиторной и самостоятельной работы студен-тов.

Настоящее учебное пособие составлено в соответствии с ФГОС ВПО по направлению подготовки 110400.62 − Агрономия.

Рассмотрено и одобрено на заседании кафедры высшей математики.

Протокол № 1 от 27.08.2013 г.

Рассмотрено и одобрено методической комиссией по направлению подготовки 110400.62 − Агрономия.

Протокол № 1 от 17.09.2013 г.

© Середина М.Н., Шипик Л.Ю., 2014 © ФГБОУ ВПО АЧГАА, 2014

Page 3: М.Н. Середина, Л.Ю. Шипик МАТЕМАТИКАачии.рф/files/2018-11-16-7b4836c8-02e7-466b-aa5c...2018/11/16  · функции нескольких переменных,

3

Содержание

Введение............................................................................................................ 1 Ведение в математический анализ............................................................. 1.1 Множества и операции над ними..................................................... 1.2 Понятие функции одной переменной............................................... 1.3 Понятие предела функции................................................................. 2 Дифференцирование функции.................................................................... 2.1 Понятие производной………………................................................ 2.2 Правило Лопиталя раскрытия неопределенностей......................... Вопросы для самопроверки............................................................................. Задания для аудиторной и самостоятельной работы………….................... Примерный вариант практического рейтинга № 1....................................... 3 Исследование функции и построение графика......................................... 3.1 Монотонность функции. Экстремумы............................................. 3.2 Наибольшее и наименьшее значения функции на отрезке............ 3.3 Выпуклость и вогнутость графика функции. Точки перегиба...... 3.4 Асимптоты графика функции…....................................................... 3.5 Построение графика функции……................................................... Вопросы для самопроверки............................................................................. Задания для аудиторной и самостоятельной работы………........................ Примерный вариант контрольной работы № 1……………………............. 4 Неопределенный интеграл…….................................................................. 4.1 Непосредственное интегрирование…………………….................. 4.2 Интегрирование методом подстановки............................................ 4.3 Интегралы, содержащие квадратный трехчлен………................... 4.4 Метод интегрирования по частям..................................................... 4.5 Интегрирование рациональных дробей........................................... Вопросы для самопроверки............................................................................. Задания для аудиторной и самостоятельной работы………........................ 5 Определѐнный интеграл и его применение .............................................. 5.1 Вычисление определѐнных интегралов ………………….............. 5.2 Вычисление площадей плоских фигур............................................. 5.3 Вычисление длины дуги …………………....................................... 5.4 Вычисление объѐмов тел вращения................................................. 5.5 Численность популяции………………............................................. 6 Несобственные интегралы…………………............................................... 6.1 Несобственные интегралы с бесконечными пределами

интегрирования……………………….…………............................. 6.2 Несобственные интегралы от неограниченных функций.............. Вопросы для самопроверки............................................................................. Задания для аудиторной и самостоятельной работы………........................

5 6 6 7 8

13 13 16 17 18 24 24 24 27 27 30 32 35 35 37 37 39 40 43 44 46 49 50 52 52 54 58 60 62 63

63 64 66 66

Page 4: М.Н. Середина, Л.Ю. Шипик МАТЕМАТИКАачии.рф/files/2018-11-16-7b4836c8-02e7-466b-aa5c...2018/11/16  · функции нескольких переменных,

4

7 Понятие функции нескольких переменных............................................... 7.1 Частные производные функции нескольких переменных. Полный дифференциал. Градиент ………….……………….......... 7.2 Экстремум функции нескольких переменных................................ Вопросы для самопроверки............................................................................. Задания для аудиторной и самостоятельной работы………….................... Примерный вариант практического рейтинга № 2……………………… 8 Элементы дискретной математики………................................................. 8.1 Элементы математической логики...………….…………………... 8.2 Элементы комбинаторного анализа…………................................. Вопросы для самопроверки............................................................................. Задания для аудиторной и самостоятельной работы……………................ 9 Теория вероятностей…………………………............................................ 9.1 Классическое, статистическое и геометрическое определения

вероятности. Теоремы сложения и умножения............................... 9.2 Формула полной вероятности. Формулы Бейеса………................ 9.3 Формула Бернулли. Теоремы Лапласа. Формула Пуассона…………………………………………………. 9.4 Случайные величины……….…........................................................ Вопросы для самопроверки............................................................................. Задания для аудиторной и самостоятельной работы………….................... 10 Элементы математической статистики……............................................ 10.1 Статистическое распределение выборки....................................... 10.2 Эмпирическая функция распределения………………................. Вопросы для самопроверки............................................................................. Задания для аудиторной и самостоятельной работы………….................... Примерный вариант контрольной работы № 2……………………………. Ответы............................................................................................................... Приложение 1………………………………………………………………... Приложение 2………………………………………………………………... Литература........................................................................................................

68

68 72 73 74 75 75 75 79 80 80 81

81 83

85 87 93 93 103 103 104 105 105 106 108 118 119 121

Page 5: М.Н. Середина, Л.Ю. Шипик МАТЕМАТИКАачии.рф/files/2018-11-16-7b4836c8-02e7-466b-aa5c...2018/11/16  · функции нескольких переменных,

5

ВВЕДЕНИЕ

Данный практикум соответствует Федеральному государственному об-разовательному стандарту высшего профессионального образования дисци-плины «Математика», изучаемой студентами основной образовательной про-граммы бакалавриата по направлению подготовки 110400.62 − Агрономия.

Учебное пособие поможет разобраться с такими разделами математики как: введение в математический анализ, дифференцирование функции, ис-следование функции и построение графика, неопределенный интеграл, опре-делѐнный интеграл и его применение, несобственные интегралы, понятие функции нескольких переменных, элементы дискретной математики, теория вероятностей, элементы математической статистики. Каждый раздел предва-ряется необходимыми теоретическими пояснениями, включая основные определения и теоремы, которые приведены без доказательств. При изложе-нии материала применяются традиционные обозначения и терминология.

В пособии сделан основной упор на приобретение навыков использова-ния математического аппарата. Все разделы снабжены большим количеством решенных примеров и задач, помогающих усвоить и закрепить изучаемый материал. Затем дается блок задач с ответами для аудиторной и самостоя-тельной работы студентов. Также каждый раздел содержит вопросы для са-мопроверки и примерные варианты контрольных работ.

Настоящий практикум будет весьма полезен преподавателю при прове-дении занятий со студентами как очного, так и заочного обучения по указан-ному направлению. Кроме того, практикум может быть использован студен-тами для самостоятельного изучения соответствующего материала и является базой для подготовки к экзамену.

Пособие способствует формированию у студентов: − знаний основных терминов, понятий, определений разделов математиче-ского анализа, дискретной математики, теории вероятностей и математиче-ской статистики (ОК-1, ПК-1); − умений решать типовые задачи по предложенным методам, графически ил-люстрировать задачи (ПК-1); − владений способностью логически верно и ясно излагать понятия и форму-лировки основных разделов предметной области (ОК-1), а также основными способами представления математической информации (ПК-1).

Изучение математики позволит студенту не только приобрести необхо-димые базовые навыки, используемые при его дальнейшем обучении, но и сформировать компоненты своего мышления: уровень, кругозор и культуру. Всѐ это понадобится для успешной работы и для ориентации в будущей про-фессиональной деятельности.

Page 6: М.Н. Середина, Л.Ю. Шипик МАТЕМАТИКАачии.рф/files/2018-11-16-7b4836c8-02e7-466b-aa5c...2018/11/16  · функции нескольких переменных,

6

1 Введение в математический анализ

1.1 Множества и операции над ними

Под множеством понимается совокупность объектов, называемых эле-

ментами множества: ...,,,...,, YXBA

Запись Aa означает, что объект a есть элемент множества A (при-надлежит множеству A ); в противном случае пишут Aa .

Множество, не содержащее ни одного элемента, называется пустым и обозначается символом Ø.

Запись BA ( A содержится в B ) означает, что каждый элемент мно-жества A является элементом множества B ; в этом случае множество A называется подмножеством множества B .

Множества A и B называются равными ( BA ) , если BA и AB . Способы задания множеств: 1. Перечисление элементов. Например: 11,7,5,2A ;

NnA ...,,...,4,3,2,1 – множество натуральных чисел.

R – множество действительных чисел. 2. Имеется некоторое основное множество E и свойство P , по которо-

му мы будем выбирать элементы из множества E : PEaA : .

Например: 01: 2xRxA Ø.

2,204: 2 xRxA .

Дадим определение промежутков: Пусть baRba ,, , тогда − отрезок – это множество bxaRxba :; ;

− интервал – это множество bxaRxba :; ;

− полуинтервал – это множество bxaRxba :; либо

множество bxaRxba :; .

Операции над множествами Рассмотрим множества A и B одной природы. 1. Множество, состоящее из элементов, принадлежащих хотя бы одному

из данных множеств, называется объединением: BA (рисунок 1 а). 2. Множество, состоящее из тех и только тех элементов, которые при-

надлежат как множеству A так и множеству B , называется пересечением: BA (рисунок 1 б). 3. Множество, состоящее из тех и только тех элементов, которые при-

надлежат множеству B , но не принадлежат множеству A , называется разно-стью множеств B и A : AB \ (из B удалены элементы множества A ) (ри-сунок 1 в).

Page 7: М.Н. Середина, Л.Ю. Шипик МАТЕМАТИКАачии.рф/files/2018-11-16-7b4836c8-02e7-466b-aa5c...2018/11/16  · функции нескольких переменных,

7

а б

в

Рисунок 1

Пример 1.1. Пусть даны множества 5;0A и 10;1B .

Тогда 10;0BA ; 5;1BA ; 10;5\ AB .

Пример 1.2. Пусть даны множества 7,5,3,0,1,2 A ; ,5,3,1B

13,10,8 ; 13,8,5,1,1,3 C .

Тогда 13,10,8,7,5,3,0,1,2 BA ; 13,8,5,1CBA ;

7,0,2\ BA .

1.2 Понятие функции одной переменной

Пусть YX , – некоторые множества. Если каждому элементу x множе-ства X можно поставить в соответствие по некоторому закону f единствен-ный элемент y из множества Y , то говорят, что задана функция f , дей-ствующая из X на Y . При этом x – аргумент функции, xfy – значение функции.

Множество X называется областью определения функции и обознача-ется fD .

Область значений функции – это множество значений, которые прини-мает функция в результате еѐ применения. Таким образом, областью значе-ний функции называется подмножество множества Y вида xfyXxYyfE ,: .

Пусть заданы прямоугольная система координат Oxy и функция xfy . Графиком функции xf называется множество всех точек плос-

кости с координатами xfx; , где fDx .

Формула xfy определяет явный способ задания функции.

Page 8: М.Н. Середина, Л.Ю. Шипик МАТЕМАТИКАачии.рф/files/2018-11-16-7b4836c8-02e7-466b-aa5c...2018/11/16  · функции нескольких переменных,

8

Однако во многих случаях приходится использовать неявный способ за-дания функции.

Пусть данная функция определена на множестве X . Тогда, если каждое значение Xx и соответствующее ему значение функции y удовлетворяют некоторому (одному и тому же) уравнению 0, yxF , то говорят, что эта функция задана неявно уравнением 0, yxF . Сама функция в этом случае называется неявной функцией.

Графиком уравнения 0, yxF называется множество всех точек коор-динатной плоскости Oxy , координаты которых удовлетворяют этому уравне-нию.

В некоторых случаях мы рассматриваем параметрически заданные функции.

Пусть на некотором множестве RT заданы две функции txx и tyy . Тогда множество всех точек плоскости Oxy с координатами

tytx ; , где Tt , называется кривой (или линией), заданной параметри-чески.

Если кривая, заданная параметрически, является графиком некоторой функции xfy , то эта функция называется функцией, заданной пара-метрически.

Пример 1.3. Найти области определения и значений функции 543 xy .

Решение. 1. Область определения данной функции определяется нера-венством 04x или 4x , то есть ;4yD .

2. Чтобы найти область значений функции, произведем оценку: 04 x ;

043 x ; 5543 x . Таким образом ;5yE .

1.3 Понятие предела функции

Интервал (а – ε; а + ε) называется окрестностью точки а и обозна-

чается aU .

Проколотой окрестностью точки a называется окрестность

этой точки без a : aaUo

a .

Пусть функция f определена на множестве X .

Величина А (конечная или бесконечная) называется пределом функции xf , если для любого сколь угодно малого числа > 0 найдется положи-

тельное число такое, что для всех XUx a

o

выполняется

AUxf .

При этом пишут xfAax

lim .

Page 9: М.Н. Середина, Л.Ю. Шипик МАТЕМАТИКАачии.рф/files/2018-11-16-7b4836c8-02e7-466b-aa5c...2018/11/16  · функции нескольких переменных,

9

При вычислении пределов функций необходимо знать следующие тео-ремы:

CCax

lim ,

где constC ; ax

ax

lim .

xfCxfCaxax

limlim ,

где constC . Отсюда следует следующее правило вычисления пределов. Правило 1. Чтобы вычислить предел функции при ax , нужно в

функцию вместо x подставить еѐ предельное значение. Если существуют конечные пределы Axf

ax

lim и Bxg

ax

lim , то

BAxgxfax

lim ,

BAxgxfax

lim ,

B

A

xg

xf

ax

lim ,

где 0B .

Будем говорить, что отношение двух функций xg

xf при ax есть не-

определѐнность вида

0

0 или

, если 0limlim

xgxf

axax или .

Раскрыть неопределѐнность – это значит вычислить предел xg

xf

axlim , если

он существует, или установить, что он не существует. При этом можно вос-пользоваться следующими правилами.

Правило 2. Если функции xf и x − многочлены, то при x до-статочно найти предел отношения их старших степеней.

Функцию f называют бесконечно малой (б.м.) вблизи точки а (при ax ), если 0lim

xf

ax.

Пусть xf и x – б.м. при ax .

Если 1lim

x

xf

ax , то xf и x называют эквивалентными б.м.

При этом пишут xf ~ x при ax .

Правило 3. При рассмотрении предела произведения или отношения б.м. функций, эти функции можно заменять их эквивалентными вблизи точки рассмотрения, не изменив существования и величины предела.

При это будем пользоваться таблицей 1.

Page 10: М.Н. Середина, Л.Ю. Шипик МАТЕМАТИКАачии.рф/files/2018-11-16-7b4836c8-02e7-466b-aa5c...2018/11/16  · функции нескольких переменных,

10

Таблица 1 – Эквивалентные бесконечно малые функции

1. xsin ~ x при 0x

2. xtg ~ x при 0x

3. xarcsin ~ x при 0x

4. xarctg ~ x при 0x

5. x1ln ~ x при 0x

6. 1xe ~ x при 0x

Пример 1.4. Вычислить предел 523

42lim

2

2

xx

xx

x.

Решение. Применим правило 1 вычисления пределов: подставим в дан-ное выражение предельное значения аргумента x и получим неопреде-

лѐнность вида

:

523

42lim

2

2

xx

xx

x.

Применим правило 2 вычисления пределов и найдем предел отношения старших степеней:

3

2

3

2lim

523

42lim

2

2

2

2

х

хxx

xx

xx.

Пример 1.5. Вычислить предел xtg

xx

x 4

2sinlim

20.

Решение. Применим правило 1 вычисления пределов: подставим в дан-ное выражение предельное значения аргумента 0x и получим неопреде-

лѐнность вида

0

0:

0

0

4

2sinlim

20 xtg

xx

x.

Применим правило 3 вычисления пределов: заменим в отношении бес-конечно малые функции на им эквивалентные:

8

1

4

2lim

4

2sinlim

2020

x

xx

xtg

xx

xx,

здесь учитывается, что x2sin ~ x2 , xtg 42 ~ 24x при 0x .

Пример 1.6. Вычислить предел 48

12sinlim

5,0

xarctg

x

x.

Решение. Применим правило 1 вычисления пределов: подставим в дан- ное выражение предельное значения аргумента 5,0x и получим неопреде-

Page 11: М.Н. Середина, Л.Ю. Шипик МАТЕМАТИКАачии.рф/files/2018-11-16-7b4836c8-02e7-466b-aa5c...2018/11/16  · функции нескольких переменных,

11

лѐнность вида

0

0:

0

0

48

12sinlim

5,0 xarctg

x

x.

Применим правило 3 вычисления пределов: заменим в отношении бес-конечно малые функции на им эквивалентные:

4

1

124

12lim

48

12lim

48

12sinlim

5,05,05,0

x

x

x

x

xarctg

x

xxx.

Пример 1.7. Вычислить предел 352

9lim

2

2

3

xx

x

x.

Решение. Применим правило 1 вычисления пределов: подставим в дан-ное выражение предельное значения аргумента 3x и получим неопреде-

лѐнность вида

0

0:

0

0

352

9lim

2

2

3 xx

x

x.

Для раскрытия неопределѐнности в таком примере нужно числитель и знаменатель разложить на произведение простых множителей. Для этого воспользуемся формулой 21

2xxxxacbxax , где 21, xx − корни

квадратного трехчлена.

В нашем случае: 1232

132352 2

xxxxxx , тогда

7

6

12

3lim

123

33lim

352

9lim

332

2

3

x

x

xx

xx

xx

x

xxx.

Для раскрытия неопределѐнностей вида 0 и необходимо

свести их к неопределѐнностям вида

0

0 или

.

Пример 1.8. Вычислить предел

1

4

1

2lim

21 xxx.

Решение. Применим правило 1 вычисления пределов: подставим в дан-ное выражение предельное значение аргумента 1x и получим неопреде-лѐнность вида :

0

4

0

2

1

4

1

2lim

21 xxx.

Для раскрытия этой неопределенности данное выражение необходимо преобразовать. Приведем дроби к общему знаменателю и найдем разность дробей:

Page 12: М.Н. Середина, Л.Ю. Шипик МАТЕМАТИКАачии.рф/files/2018-11-16-7b4836c8-02e7-466b-aa5c...2018/11/16  · функции нескольких переменных,

12

11

12lim

0

0

1

22lim

1

412lim

1

4

1

2lim

1212121 xx

x

x

x

x

x

xx xxxx

1

2lim

1

xx.

Подставив в полученное выражение 1x , получим:

12

2

1

2lim

1

4

1

2lim

121

xxx xx

.

Пример 1.9. Вычислить предел

224 224lim xxxx

.

Решение. Применим правило 1 вычисления пределов: подставим в дан-ное выражение предельное значение аргумента x и получим неопреде-лѐнность вида :

224 224lim xxxx

.

Для избавления от неопределѐнности в случае, когда функция содержит иррациональное выражение, необходимо домножить и разделить еѐ на со-пряженное:

224

224224

224

224

2224224lim224lim

xxx

xxxxxx

xxxxx

224

224

224

424lim

xxx

xxx

x

224

2

224

2lim

xxx

x

x.

Применим правило 2 вычисления пределов и найдем предел отношения старших степеней:

224 224lim xxxx 4

1

22lim

224

2lim

22

2

224

2

xx

x

xxx

x

xx.

Пример 1.10. Вычислить предел 34

5lim

5

x

x

x.

Решение. Применим правило 1 вычисления пределов: подставим в дан-ное выражение предельное значение аргумента 5x и получим неопреде-

лѐнность вида

0

0:

0

0

34

5lim

5 x

x

x.

Для раскрытия неопределѐнности применим тот же прием, что и в при-мере 1.9.

94

345lim

3434

345lim

34

5lim

555 x

xx

xx

xx

x

x

xxx

Page 13: М.Н. Середина, Л.Ю. Шипик МАТЕМАТИКАачии.рф/files/2018-11-16-7b4836c8-02e7-466b-aa5c...2018/11/16  · функции нескольких переменных,

13

634lim

5

345lim

55

xx

xx

xx.

2 Дифференцирование функции

2.1 Понятие производной

По определению производная функции xfy

,lim0 x

yy

x

(2.1)

где y – приращение функции; x – приращение аргумента.

Таблица 2 − Правила и формулы дифференцирования

1. 0С 12. ueeuu )(

2. 1x 13. u

uu

)(ln

3. wvuwvu )( 14. au

uua

ln)(log

4. vuvuuv )( 15. uuu cos)(sin

5. uCСu )( 16. uuu sin)(cos

6. 2

v

vuvu

v

u

17. u

utgu

2cos)(

7. 2

v

vC

v

C

18. u

uctgu

2sin)(

8. C

u

C

u

19. 21

)(arcsinu

uu

9. unuunn 1)( 20.

21)(arccos

u

uu

10. u

uu

2)(

21. 21

)(u

uarctgu

11. uaaauu ln)( 22. 21

)(u

uarcctgu

Чтобы найти производную функции, заданной неявно уравнением

0);( yxF необходимо продифференцировать обе части данного уравнения, а затем выразить из полученного равенства y .

Page 14: М.Н. Середина, Л.Ю. Шипик МАТЕМАТИКАачии.рф/files/2018-11-16-7b4836c8-02e7-466b-aa5c...2018/11/16  · функции нескольких переменных,

14

В случае параметрически заданной функции

,

,

tyy

txx производную

можно найти по формуле

t

tx

x

yy

. (2.2)

Если y – производная функции ),(xfy то )( yy – вторая произ-водная функции ).(xfy Таким образом, вторая производная – это произ-водная от первой производной, третья производная – это производная от вто-рой производной и т.д.

Геометрический смысл производной. Производная функции )(xf в

точке 0x равна угловому коэффициенту касательной, проведѐнной к графику этой функции в точке с абсциссой 0x , то есть tgxf 0 (рисунок 2).

Рисунок 2

Если функция xfy дифференцируема в точке 0x , то уравнение ка-сательной к графику этой функции в точке с абсциссой 0x имеет вид

000 xxxyxyy . (2.3)

При этом уравнение нормали к графику этой же функции в точке с абсцис-сой 0x выглядит следующим образом

00

01

xxxy

xyy или .0000 xyyxyxx (2.4)

Задача о скорости химической реакции. Пусть дана функция tmm ,

где m − количество некоторого вещества, вступившего в химическую реак-цию к моменту времени t .

Page 15: М.Н. Середина, Л.Ю. Шипик МАТЕМАТИКАачии.рф/files/2018-11-16-7b4836c8-02e7-466b-aa5c...2018/11/16  · функции нескольких переменных,

15

Приращению времени t будет соответствовать приращение m вели-чины m .

Отношение t

m

− средняя скорость химической реакции за промежуток

времени t . А предел этого отношения при стремлении t к нулю, то есть

t

m

t

0lim − это скорость химической реакции в данный момент времени t .

Таким образом, скорость химической реакции есть производная количе-ства вещества m по времени t .

Пример 2.1. Найти производную функции .2

9cos37ln 2

xxy

Решение. Используя правила и формулы дифференцирования из табли-цы 2, получаем

xx

x

xx

xxy

29cos

37

3729cos37ln

29cos37ln

2

222

xx

x

xxx

29cos

37

1429

29sin37ln

22

2

2 229sin37ln

xxx

.

Пример 2.2. Найти вторую производную функции .13

2xe

xy

Решение. Сначала найдем первую производную:

.

612632133131324

2

4

22

22

22

xx

x

x

xx

x

xx

e

x

e

xe

e

exe

e

exexy

Теперь найдем вторую производную:

x

x

x

xx

x

xx

e

xe

e

exe

e

exexy

4

2

4

22

22

22 12262616

)(

6161

.812

2xe

x

Пример 2.3. Найти производную параметрически заданной функции

.2cos

,2sin

tay

xax

Решение. Найдем taxt 2cos2 , tayt 2sin2 и воспользуемся форму-

лой (2.2): ttgta

tayx 2

2cos2

2sin2

.

Пример 2.4. Найти производную функции, заданной неявно уравнением

Page 16: М.Н. Середина, Л.Ю. Шипик МАТЕМАТИКАачии.рф/files/2018-11-16-7b4836c8-02e7-466b-aa5c...2018/11/16  · функции нескольких переменных,

16

xeyxy 7cos532 4 .

Решение. Продифференцируем обе части уравнения, учитывая, что xfy : xyeyxy

y 7sin354322 4 . Выразим из полученного равен-ства y :

yxyexy 27sin35122 4 или

yex

yxy

4122

27sin35

.

Пример 2.5. Составить уравнения касательной и нормали к графику функции 16 xy в точке с абсциссой .40 x

Решение. Поскольку ,

16

3

162

6

162

16

xxx

xy

,5

3

124

30

xy ,51240 xy то воспользовавшись формулой

(2.3), получаем

,45

35 xy ,43255 xy .123255 xy

Таким образом, касательная к графику заданной функции в точке 40 x за-даѐтся уравнением

.01353 yx

Теперь воспользуемся формулой (2.4):

,43

55 xy ,45153 xy .205153 xy

Следовательно, нормаль к графику заданной функции в точке с абсциссой 40 x имеет вид

.03535 yx

2.2 Правило Лопиталя раскрытия неопределенностей

При вычислении предела функции для избавления от неопределѐнностей

вида

0

0 или

можно воспользоваться следующей теоремой.

Правило Лопиталя. Пусть функции xf и xg определены и диффе-ренцируемы в некоторой окрестности точки 0x , 0limlim

00

xgxfxxxx

или , 0 xg при всех x из данной окрестности. Если существует

xg

xf

xx

0

lim , то существует xg

xf

xx 0

lim

и справедлива формула:

xg

xf

xg

xf

xxxx

00

limlim . (2.5)

При этом если производные xf и xg удовлетворяют тем же требо-

Page 17: М.Н. Середина, Л.Ю. Шипик МАТЕМАТИКАачии.рф/files/2018-11-16-7b4836c8-02e7-466b-aa5c...2018/11/16  · функции нескольких переменных,

17

ваниям, что и сами функции xf и xg , то правило Лопиталя можно при-менить повторно.

Пример 2.6. Вычислить предел 20

cos1lim

x

x

x

.

Решение.

0

0cos1lim

20 x

x

x.

Воспользуемся правилом Лопиталя:

0

0

2

sinlim

cos1lim

cos1lim

02020 x

x

x

x

x

x

xxx.

Применим правило Лопиталя повторно: 2

1

2

coslim

2

sinlim

2

sinlim

000

x

x

x

x

x

xxx.

Пример 2.7. Вычислить предел xctgxx

2lim

.

Решение.

02lim xctgxx

.

Преобразуем произведение, стоящее под знаком предела, в частное, ис-

пользуя, что xtg

xctg2

12 :

0

0

2lim2lim

xtg

xxctgx

xx

.

Воспользуемся правилом Лопиталя:

2

1

2cos

2

1lim

2lim2lim

2

x

xtg

xxctgx

xxx

.

Вопросы для самопроверки

1. Сформулируйте определение функции. Что называется областью опреде-ления функции? 2. Какие способы задания функции вы знаете? 3. Какие функции называются элементарными? 4. Дайте определение понятия предела функции. 5. В каком случае функция называется бесконечно малой? 6. Сформулируйте основные теоремы о пределах. 7. Дайте определение непрерывности функции в точке. 8. Укажите основные свойства непрерывных функций. 9. Сформулируйте определение производной. 10. Каков геометрический, химический смыслы производной? 11. Что называется дифференциалом функции? 12. Сформулируйте правило Лопиталя раскрытия неопределенностей.

Page 18: М.Н. Середина, Л.Ю. Шипик МАТЕМАТИКАачии.рф/files/2018-11-16-7b4836c8-02e7-466b-aa5c...2018/11/16  · функции нескольких переменных,

18

Задания для аудиторной и самостоятельной работы 1. Даны множества ,7,6,5,4,3,2,1,0А ,10,8,6,4,2,1B ,1,4 C

.8,7,4,3,0 Найти: а) ;BA б) ;CA в) ;\ BA г) ;\ CAB д) .CBA

2. Даны множества ],3;7(A ],5;1[B ).1;10[C Найти .\ BCA

3. Даны множества ,11,10,6,3,1,2,6,9 А ,3,2,1,0,1,2,3 B

.15,10,8,6,3,1C Найти: а) ;CA б) ;\CB в) ).(\ BAC

4. Даны множества ,63: xRxA .82: xNxB Найти: а) ;BA б) .\ AB

5д. Даны множества ,4,3,2,1,0А ,10,8,6,4,2,1B ,1,3,5 C

.7,5,3,1 Найти .\ CBA

6д. Даны множества ,15: xNxA .72: xRxB Найти .BA

7д. Даны множества ),4;2[A ,1;5B ].7;2[C Найти: а) ;BA

б) ;CB в) .\ CA

8д. Даны множества ,2;7A ].1;11( B Найти: а) ;BA б) .BA

Найти область определения функции:

9. 52

23log 2

x

xy . 10. 4 8 хy .

11д. 65lg 2 xxy . 12д. xxy lg37 .

Найти область значений функции:

13. .3sin57 xy 14. .182 x

y

15д. .2cos6 xy 16д. .242 xxy

Найти область определения и область значений функций:

17. .8 )1(log 25 x

y 18. .36411 xy

19д. .732 xy 20д. .4 2xy

Найти множества, на которые данные функции отображают множество Х:

21. .27;9

1,log

3

1

Xxy 22. ].2;0[,2 13

Xyx

23д. ].5,0;3[,2 Xxy 24д. ].0;3[,133 Xxy

25. Найти ),0(f ),4(f ),1(f ,2

3

f если .12

9)(

xxf

26. Найти ),2(f ,3

2

f если .53)( 2 xxf

27. Найти ),2(f ),0(f ),2(f ),1(f ),6(f если

Page 19: М.Н. Середина, Л.Ю. Шипик МАТЕМАТИКАачии.рф/files/2018-11-16-7b4836c8-02e7-466b-aa5c...2018/11/16  · функции нескольких переменных,

19

.35

,3013

,0

)( 2

3

хприх

хприx

xприx

xf

28д. Найти ),1(f ),2(f ),10(f ,5

2

f если .15

122)( 2

xxxf

29д. Найти ),5,1(),1(,2

1),3(

ffff если

.132

,19)(

2 хприx

xприxxf

Вычислить указанные пределы:

30. 232

44lim

2

2

2

xx

xx

x. 31.

12

32lim

2

2

1

xx

xx

x.

32. 2

42lim

2

2

2

xx

xx

x. 33. .

48

7132lim

2

5,0

x

xx

x

34. 754

235lim

2

2

xx

xx

x. 35.

522

34lim

2

2

xx

xx

x.

36. 24

1072lim

3

2

xx

xx

x. 37.

353

5723lim

2

32

xx

xxx

x.

38. 32

23

734

1243lim

xxx

xxx

x

. 39. 44

322

325

527lim

xx

xxxx

x

.

40. 3 32

2

22787

35234lim

xxx

xxx

x. 41.

93516

8972lim

2

2

xx

xxx

x.

42. xxx

234lim

. 43.

xxx

x3129lim 2 .

44. 4

2lim

4

x

x

x. 45.

9

21lim

23

x

x

x.

46. 1

213lim

21

x

x

x. 47.

x

xarctgx

x 3sin

2lim

20

.

48. xxtg

x

x 2

arcsinlim

2

0. 49.

xxtg

xx

x 5cos4

21ln3sinlim

20

.

50. xxtg

x

x 41ln3

2sin2lim

2

0 . 51.

xxarctg

xx

x 32

sin21lnlim

2

0

.

Page 20: М.Н. Середина, Л.Ю. Шипик МАТЕМАТИКАачии.рф/files/2018-11-16-7b4836c8-02e7-466b-aa5c...2018/11/16  · функции нескольких переменных,

20

52. xxtg

xex

x 5sin3

7arcsinlim

2

22

0 . 53.

x

x

x4

4sin

lim

4

.

54. 4arcsin

63lim

22

x

xtg

x. 55.

1

1sinlim

3

1

xtg

x

x.

56.

9

6

3

1lim

23 xxx. 57.

8

12

2

1lim

32 xxx.

58.

4

5

2

1lim

22 xxx. 59.

x

x

x

x 1lim

2

3

.

60.

2

2

4

3lim

22 xx

x

x. 61.

8

842lim

3

23

2

x

xxx

x.

62. 23

12lim

3

2

1

xx

xx

x. 63.

12

1lim

2

34

1

xx

xxx

x.

64. 1cos

1lim

0

x

xex

x. 65.

x

xx

x cos1

cos2coslim

0

.

66. xtg

eexx

x 3lim

sin2sin

0

. 67. x

x

x6sin

1sin2lim

6

.

68. x

x

x 3sin

2coslnlim

20. 69.

x

xex

2

1lim .

70. 2

limx

tgxx

. 71. xxx

lnlim0

.

72д. 5

107lim

2

5

x

xx

x. 73д.

1574

352lim

2

2

3

xx

xx

x.

74д. 473

154lim

2

2

1

xx

xx

x. 75д.

524

754lim

2

23

xx

xx

x.

76д. 634

653lim

2

2

xx

xx

x. 77д.

854

45lim

2

2

xx

xx

x.

78д. 34

7542lim

42

23

xx

xxx

x. 79д.

32

32

412

235lim

xxx

xxx

x

.

80д. 3 3

2

3828

528lim

xx

xx

x. 81д.

xxx

x6lim 2 .

82д. x

x

x 2

11lim

0

. 83д. 325

2lim

2

x

x

x.

Page 21: М.Н. Середина, Л.Ю. Шипик МАТЕМАТИКАачии.рф/files/2018-11-16-7b4836c8-02e7-466b-aa5c...2018/11/16  · функции нескольких переменных,

21

84д. xtg

x

x 18

9arcsinlim

0.

86д. xarctgx

x

x 22

4sinlim

2

0 .

88д. xx

xx

x 21ln5arcsin

4sincoslim

2

0

.

90д. )1arcsin(

1lim

2

2

1

x

xarctg

x.

92д.

21 1

5

1

2lim

xxx.

94д.

27

27

3

1lim

33 xxx.

96д. 1

23lim

23

3

1

xxx

xx

x.

98д. x

x

x3cos

sin21lim

6

.

100д. 30

3coslnlim

x

x

x.

102д. ctgxex

x

2

01lim .

85д. x

x

x 31ln

4sinlim

0 .

87д. xarctg

xx

x 3

2arcsinsinlim

20

.

89д. xarctg

xx

x 4

51lnlim

20

.

91д.

1212lim

2

2

3

x

x

x

x

x.

93д.

4

4

2

1lim

22 xxx.

95д. 24

23

2 4

43lim

xx

xx

x

.

97д. 30

limx

arctgxx

x

.

99д. x

x

x

2

sin1lim .

101д. 2

1lim1

xtgx

x

.

103д. .8lim x

xex

Найти производные следующих функций:

104. .6

1

3

29 3 xxy 105. .168 23 xxxy

106. .112

44

12

2 x

xxy 107. .4

62,06 54

xxxxy

108. .2

923

3 228

xxxxy 109. .3

8 5 4

3 x

xy

110. .32

582

xx

xy 111. .595cos2 xxy

112. ).9ln(395 xxtgy 113. .)57( 4sin xexy

114. .5arcsin38xy

ctgx 115. ).76(log)2( 25

3xxxy

116. .29xarctgxy 117. .

9arccos

3x

y

118. .2sin3

37

x

xy

119. .

9cos8

7

xy

120. ).25(6 3 xtgy 121. .35 2xarctgy

Page 22: М.Н. Середина, Л.Ю. Шипик МАТЕМАТИКАачии.рф/files/2018-11-16-7b4836c8-02e7-466b-aa5c...2018/11/16  · функции нескольких переменных,

22

122. .4

768 2

x

xxy

123. .)112(5

65

x

y

124. .1359

73

4

x

xxy 125. .)215(

6

1 5 33xy

126. ).37(ln2 25xxy 127. .

4

534

x

xxy

128. .1

1x

x

xarctgy

129. .92

arcsin8

5

xex

y

130. ).32(cos25

1 210 xy 131. .8

sin37 43

xxy

132. ).9ln( 4xxy 133. .

3

1arcsin

4

9

xy

134. ).58(6 3xtgey

x 135. .ln6

cos 32

xxy

136. .93 4xarcctg

y 137. .6

sin

cos32 xx

xy

138. ).2cos(7342 x

exy 139. .173)59ln(3 72 x

exxy

140. .1

8 94

xarctgy

x 141. .11356 542 xxy

142д. .111712 23 xxxy 143д. .5

2

78

2

4

x

xy

144д. .594 3

xxxy 145д. .9

23

76

9

x

xy

146д. .)295( 34xxy 147д. .)59(35 62

xy

148д. .cos39

ln8

2

x

xy 149д. .

6318

3

4

xey

x

150д. .)18( 15 xexy

151д. .sin

2

3

52

x

xtgy

152д. .4

749log6 2

2x

xxy 153д. .5 )23(cos3 xy

154д. .)12(

9

12

192

xx

y 155д. .13 2 xarctgy

156д. ).178(sin 36 xy 157д. .2

432 2

x

xxy

Page 23: М.Н. Середина, Л.Ю. Шипик МАТЕМАТИКАачии.рф/files/2018-11-16-7b4836c8-02e7-466b-aa5c...2018/11/16  · функции нескольких переменных,

23

158д. .58

34

23

x

ex

y 159д. .679352

xxy

160д. .)6)23((ln 287 xxy 161д. .

54

12cos63

x

xy

162д. .23sin2

7cos 2 x

xy 163д. .18ln 32sin3 xey

x

Для функций, заданных параметрически, найти xy :

164.

.34

,91ln 2

tarctgy

tx 165.

.8sin

,4cos3 2

ty

tx

166.

.5

,12

t

ty

t

tx

167д.

.5618

,13

2

32

tty

tx

168д.

.26

,4

ty

tx 169д.

.24

,23

5

8

t

t

ey

ex

Найти xy для следующих функций, заданных неявно: 170. .52 232

yxeyx 171. .232sin 3 yxyx

172д. .42 22xyyx 173д. .3235

yexy

Найти производные второго порядка следующих функций: 174. ).76cos( xy 175. .)72(3 4 xy

176. ).129ln( 2 xxy 177. .27 3 xxey

178д. .5 29 xy

179д. .785

1 35xxxy

180д. ).611sin( 2xy 181д. .6 2

xarctgy

182. Вычислить ),0(f если .)13(

5)(

3

xxf

183д. Вычислить ),2(f если .2)( 24 xexf

184. Показать, что функция xxeey

32 47 удовлетворяет уравнению .065 yyy

185д. Показать, что функция xey

4118 удовлетворяет уравнению .04 yy

186. Составить уравнения касательной и нормали к кривой 32 235 xxxy в точке с абсциссой .10 x

187. Составить уравнения касательной и нормали к кривой 416 xy в точке с абсциссой .50 x

Page 24: М.Н. Середина, Л.Ю. Шипик МАТЕМАТИКАачии.рф/files/2018-11-16-7b4836c8-02e7-466b-aa5c...2018/11/16  · функции нескольких переменных,

24

188. Составить уравнения касательной и нормали к кривой )52ln(2 xxy

в точке с абсциссой .30 x

189д. Составить уравнения касательной и нормали к кривой 12 xey в точке

с абсциссой .10 x

190д. Составить уравнения касательной и нормали к кривой 182 2 xxy

в точке с абсциссой .30 x

Примерный вариант практического рейтинга № 1

Найти y :

1. xyx 2sin2 3cos ; 2. 254ln xarctgy ; 3. 32 38 xctgy ;

4. 7

4

252

x

xxy ; 5.

xex

y

x

3sin3

15ln 224

;

6. 4 22

125

1ln x

ex

xtgy .

Найти y : 7. ,3sin3xxy 8. 32ln 2 xy .

Даны множества 7,6,5,4,3,2,1,0A ; 9,8,7,6,5,4,3B ;

4,3,2,1,0,1,2,3 C . Задайте списками множества: 9. CBA ; 10. CBA .

3 Исследование функции и построение графика

3.1 Монотонность функции. Экстремумы

Функция xf называется возрастающей (убывающей) на интервале

ba; , если для любых baxx ;, 21 , удовлетворяющих условию 21 xx ,

справедливо неравенство 21 xfxf 21 xfxf .

Пусть функция xf дифференцируема на интервале ba; . В этом слу-чае, если производная 0 xf ( 0 xf ) для всех значений bax ; , то функция xf возрастает (убывает) на интервале ba; .

Точка 0x из области определения функции xf называется точкой максимума (минимума), если существует такая – окрестность точки 0x ,

что для всех 0xx из этой окрестности выполняется неравенство 00 xfxfxfxf .

Точки максимума и минимума функции называются точками экстре-мума, а значения функции в этих точках – экстремумами функции.

Если функция xf имеет экстремум в точке 0x , то ее производная в этой точке равна нулю или не существует.

Page 25: М.Н. Середина, Л.Ю. Шипик МАТЕМАТИКАачии.рф/files/2018-11-16-7b4836c8-02e7-466b-aa5c...2018/11/16  · функции нескольких переменных,

25

Точки области определения функции, в которых xf обращается в ноль или не существует, называются критическими точками 1-го рода функции xf .

Пусть функция xf дифференцируема в окрестности точки 0x и не-прерывна в самой точке 0x . Если при переходе через 0x производная функ-ции xf меняет знак, то 0x – точка экстремума, причем, если с на ,

то точка максимума, если с на , то точка минимума. План исследования функции xfy на монотонность, экстремум. 1. Найти область определения функции. 2. Найти критические точки 1-го рода ( 0y или не существует).

3. Разбить область определения критическими точками 1-го рода на интервалы и найти знак производной функции xf в каждом ин-тервале.

4. Определить промежутки убывания и возрастания функции и найти экстремумы функции.

Пример 3.1. Найти интервалы монотонности функции xxxy 23 2 .

Решение. 1. Область определения функции – множество всех действи-тельных чисел, то есть ;yD .

2. 143 2 xxy .

Производная 0y при 3

1,1 21 xx .

3. На оси Ox отметим область определения функции и критические точ-ки 1-го рода. На полученных интервалах определим знаки производной (ри-сунок 3).

Рисунок 3

Итак, функция xxxy 23 2 возрастает на интервалах

3

1; и ;1 ,

убывает на интервале

1;3

1.

Пример 3.2. Исследовать функцию 1

32

x

xy на экстремум.

Решение. 1. Область определения функции: 1: xyD или ;11; yD .

Page 26: М.Н. Середина, Л.Ю. Шипик МАТЕМАТИКАачии.рф/files/2018-11-16-7b4836c8-02e7-466b-aa5c...2018/11/16  · функции нескольких переменных,

26

2.

22

2

2

2

1

13

1

32

1

312

x

xx

x

xx

x

xxxy .

Критическими точками 1-го рода являются точки 1,3 21 xx , в кото-рых производная 0y . Точка 13 x , в которой производная y не суще-ствует, не принадлежит области определения функции, следовательно, она не является критической.

3. На оси Ox отметим область определения функции и критические точ-ки 1-го рода. На полученных интервалах определим знаки производной (ри-сунок 4).

Рисунок 4

Итак, функция имеет максимум в точке 11 x : 21max y и минимум в точке 32 x : 63max y .

Пример 3.3. Исследовать функцию x

xy

ln на экстремум.

Решение. 1. Область определения функции:

,1

,0:

x

xyD или

;11;0 yD .

2. x

x

x

xxx

y22 ln

1ln

ln

1ln1

.

Производная 0y при 01ln x ; 1ln x ; ex .

Критической точкой 1-го рода является точка ex , в которой производ-ная 0y . Точка 1x , в которой производная y не существует, не принад-лежит области определения функции, следовательно, она не является крити-ческой.

3. На оси Ox отметим область определения функции и критические точ-ки 1-го рода. На полученных интервалах определим знаки производной (ри-сунок 5).

Рисунок 5

Page 27: М.Н. Середина, Л.Ю. Шипик МАТЕМАТИКАачии.рф/files/2018-11-16-7b4836c8-02e7-466b-aa5c...2018/11/16  · функции нескольких переменных,

27

Итак, функция имеет минимум в точке ex : eey min .

3.2 Наибольшее и наименьшее значения функции на отрезке

Пусть функция xfy непрерывна на отрезке bа; . Тогда на этом от-

резке она достигает наибольшего и наименьшего значений. Эти значения мо-гут достигаться или в точках экстремума или на концах отрезка, поэтому необходимо:

1. Найти все критические точки 1-го рода на отрезке. 2. Вычислить значения функции в этих точках и на концах отрезка. 3. Из найденных значений выбрать наибольшее и наименьшее.

Пример 3.4. Найти наибольшее и наименьшее значения функции

91594

1 23 xxxy на отрезке 0;3 .

Решение. Для этого посчитаем значения заданной функции на концах отрезка 0;3 , в критических точках 1-го рода, попавших на отрезок, и сра-вним результаты.

154

356

4

315183

4

1 22 xxxxxxy .

Таким образом, функция имеет две критические точки 1-го рода 51 x ,

12 x . При этом 0;31 x .

03 у ; 41 у ; 4

90 у .

Очевидно,

4max0;3

y ;

0min0;3

y .

3.3 Выпуклость и вогнутость графика функции.

Точки перегиба

График функции xfy называется выпуклым (вогнутым) на данном интервале, если весь график лежит не выше (не ниже) любой касательной, проведенной к графику на этом интервале.

Если во всех точках интервала bа; 0 xf 0 xf , то график функции xfy на этом интервале является выпуклым (вогнутым).

Точка 000 ; yxM графика дифференцируемой функции xfy назы-вается точкой перегиба, если она отделяет выпуклую часть графика от во-гнутой.

Если 0x – абсцисса точки перегиба графика функции xfy , то в этой точке вторая производная функции 0xf либо равна нулю, либо не суще-ствует.

Точки области определения функции, в которых вторая производная

Page 28: М.Н. Середина, Л.Ю. Шипик МАТЕМАТИКАачии.рф/files/2018-11-16-7b4836c8-02e7-466b-aa5c...2018/11/16  · функции нескольких переменных,

28

xf равна нулю или не существует, называются критическими точками 2-го рода.

Пусть функция xfy дважды дифференцируема в окрестности кри-тической точки 2-го рода 0x и непрерывна в самой точке 0x . Если при пере-ходе через нее вторая производная функции xf меняет знак, то

000 ; yxM – точка перегиба. План исследования функции xfy на выпуклость,

вогнутость графика, точки перегиба. 1. Найти область определения функции. 2. Найти критические точки 2-го рода ( 0 xf или не существует).

3. Разбить область определения критическими точками 2-го рода на ин-тервалы и найти знак второй производной функции xf в каждом интервале.

4. Определить промежутки выпуклости и вогнутости графика функции и найти точки перегиба.

Пример 3.5. Найти интервалы выпуклости и вогнутости графика функ-

ции 3

42

2

x

xy .

Решение. 1. Область определения функции – множество всех действи-тельных чисел, то есть ;yD .

2.

2222

33

22

22

3

24

3

8248

3

2438

x

x

x

xxx

x

xxxxy ,

32

2

42

222

42

222

3

172

3

43324

3

23224324

x

x

x

xxx

x

xxxxy .

Таким образом, функция имеет две критические точки 2-го рода 11 x ,

12 x .

3. На оси Ox отметим область определения функции и критические точ-ки 2-го рода. На полученных интервалах определим знаки второй производ-ной (рисунок 6).

Рисунок 6

Итак, график функции выпуклый на интервалах 1; и ;1 , во-гнутый на интервале 1;1 .

Page 29: М.Н. Середина, Л.Ю. Шипик МАТЕМАТИКАачии.рф/files/2018-11-16-7b4836c8-02e7-466b-aa5c...2018/11/16  · функции нескольких переменных,

29

Пример 3.6. Найти точки перегиба графика функции 2

2

x

xy .

Решение. 1. Область определения функции yD : 2x или

;22; yD .

2. 32 2

4

2

2

22

x

x

x

xx

x

xy ,

46

2

6

23

2

224

2

3224

2

23424

x

x

x

xxx

x

xxxy

42

18

x

x.

Таким образом, функция имеет одну критическую точку 2-го рода 11 x .

Точка 22 x не принадлежит области определения функции. 3. На оси Ox отметим область определения функции и критические точ-

ки 2-го рода. На полученных интервалах определим знаки второй производ-ной (рисунок 7).

Рисунок 7

Итак, 1.. птx , 9

11.. yy пт . Таким образом

9

1;1M – точка пере-

гиба. Точки перегиба важны в биохимии, так как они определяют условия, при

которых некоторая величина, например скорость процесса, наиболее (или наименее) чувствительна к каким-либо воздействиям. Рассмотрим (см. [8])

способность буфера препятствовать изменению HlgpHpH , которое

может быть обусловлено, к примеру, добавлением щелочи. Эта ситуация описывается уравнением

Кислота

СольlgpKpH д ,

где дK − константа диссоциации кислоты,

дд KlgpK .

Для буфера, приготовленного добавлением x моль∙ 1л NaOH к раствору уксусной кислоты cHOA (для краткости через cA обозначено COCH3 ),

начальная концентрация которого равна A , мы имеем

Page 30: М.Н. Середина, Л.Ю. Шипик МАТЕМАТИКАачии.рф/files/2018-11-16-7b4836c8-02e7-466b-aa5c...2018/11/16  · функции нескольких переменных,

30

HOA

NaOHln4343,0pK

HOA

NaOHlgpKpH дpK

xA

x

ln4343,0 , (3.1)

поскольку до тех пор пока x меньше A , практически все добавляемые ионы OH стехиометрически превращают молекулы cHOA в ионы

cOA . Диффе-ренцируя по x , получаем

pH xAx

A

xA

xxA

x

xA

4343,04343,0

2.

Эта первая производная является мерой чувствительности pH к действию щелочи. Ясно, что буфер наиболее эффективен, когда производная мини-мальна (или обратная ей величина, известная под названием буферной емко-сти, максимальна). Для нахождения значения x , при котором pH мини-мальная, следует взять вторую производную

pH 22

24343,0

xAx

xAA

.

Отсюда видно, что вторая производная равна нулю в точке 2

Ax и при пере-

ходе через нее в направлении возрастания x меняет знак с (−) на (+). Следо-вательно, pH минимальна в точке перегиба функции pH . Подставив это значение в (3.1), найдем, что

дpKpH A

A

2

2ln4343,0 дpK .

Таким образом, буфер наиболее эффективен при pH , равном дpK кислоты, которая участвует в нитрировании.

3.4 Асимптоты графика функции

Асимптотой графика функции называется прямая, к которой неограни-ченно приближается уходящая в бесконечность ветвь графика функции.

Мы будем различать вертикальные и наклонные асимптоты. Прямая ax будет вертикальной асимптотой графика функции xfy , если

xf

axlim или один из односторонних пределов, то есть

xfax 0

lim или

xfax 0

lim . Следовательно, значения a следует ис-

кать среди точек разрыва второго рода данной функции. Уравнения наклонных асимптот имеют вид bkxy , где

x

xfk

x lim2,1 , kxxfb

x

lim2,1 . Если при вычислении k или b хотя

бы один из пределов не существует или равен бесконечности, то наклонных асимптот нет.

Page 31: М.Н. Середина, Л.Ю. Шипик МАТЕМАТИКАачии.рф/files/2018-11-16-7b4836c8-02e7-466b-aa5c...2018/11/16  · функции нескольких переменных,

31

Пример 3.7. Найти асимптоты графика функции 5

3 2

x

xy .

Решение. 1. Область определения функции yD : 5x или

;55; yD .

2. Найдем вертикальную асимптоту графика функции.

Поскольку 5

3lim

2

5 x

x

x, то прямая 5x является вертикальной

асимптотой графика функции. 3. Выясним, имеет ли график функции наклонные асимптоты вида

bkxy :

33

lim5

3lim

5

3lim

)(lim

2

2,1

x

x

x

x

xx

x

x

xfk

xxxx, тогда

x

x

x

xx

x

xkxxfb

xxxx

15lim

5

15lim3

5

3lim)(lim

2

2,1

15 . Таким образом, график функции имеет наклонную асимптоту 153 xy

при x .

Пример 3.8. Найти асимптоты графика функции 42

3

x

xy .

Решение. 1. Область определения функции – множество всех действи-тельных чисел, то есть ;yD .

2. Так как функция определена и непрерывна на всей числовой оси, то график функции не имеет вертикальных асимптот.

3. Выясним, имеет ли график функции наклонные асимптоты вида bkxy :

1lim4

lim4

lim)(

lim2

2

2

2

2

3

2,1

x

x

x

x

xx

x

x

xfk

xxxx, тогда

.04

lim

4lim

4

4lim

4lim)(lim

222

3

2,1

x

x

x

x

xx

x

xkxxfb

x

xxxx

Таким образом, график функции имеет наклонную асимптоту xy при x .

Пример 3.9. Найти асимптоты графика функции 12 xy .

Решение. 1. Область определения функции – множество всех действи-тельных чисел, то есть ;yD .

2. Так как функция определена и непрерывна на всей числовой оси, то график функции не имеет вертикальных асимптот.

Page 32: М.Н. Середина, Л.Ю. Шипик МАТЕМАТИКАачии.рф/files/2018-11-16-7b4836c8-02e7-466b-aa5c...2018/11/16  · функции нескольких переменных,

32

3. Выясним, имеет ли график функции наклонные асимптоты вида bkxy :

1limlimlim1

lim)(

lim22

1

x

x

x

x

x

x

x

x

x

xfk

xxxxx;

1limlimlim1

lim)(

lim22

2

x

x

x

x

x

x

x

x

x

xfk

xxxxx,

тогда

xxkxxfb

xx1lim)(lim 2

1

xxxx

xx

xx

xxxx

xxx 1

1lim

1

1lim

1

11lim

22

22

2

22

.01

xxxx

xx

xx

xxxx

xxkxxfb

xxx

xx

1

1lim

1

1lim

1

11lim

1lim)(lim

22

22

2

22

22

.01

Таким образом, график функции имеет наклонные асимптоты

xy при x и xy при x .

3.5 Построение графика функции

Наиболее наглядное представление о ходе изменения функции дает ее график. Построение графика функции проводится по следующей схеме:

1. Найти область определения функции yD .

2. Исследовать функцию на непрерывность; найти точки разрыва функ-ции и ее односторонние пределы в точках разрыва.

3. Найти экстремумы функции и определить интервалы ее монотонно-сти.

4. Найти точки перегиба графика функции и определить интервалы вы-пуклости и вогнутости графика.

5. Найти асимптоты графика функции. 6. Построить график, используя результаты предыдущих исследований.

Пример 3.10. Провести полное исследование функции 4

202

x

xy и по-

строить ее график. Решение. 1. Область определения функции: yD : 4x или

Page 33: М.Н. Середина, Л.Ю. Шипик МАТЕМАТИКАачии.рф/files/2018-11-16-7b4836c8-02e7-466b-aa5c...2018/11/16  · функции нескольких переменных,

33

;44; yD .

2. Исследование на непрерывность и классификация точек разрыва. Заданная функция непрерывна всюду, кроме точки 4x . Вычислим еѐ

односторонние пределы в этой точке:

4

20limlim

2

0404 x

xxf

xx;

4

20limlim

2

0404 x

xxf

xx.

Таким образом, точка 4x является для заданной функции точкой раз-рыва второго рода, а прямая 4x – вертикальной асимптотой графика.

3. Исследование на экстремум и промежутки монотонности.

22

2

2

2

4

102

4

208

4

2042

x

xx

x

xx

x

xxxy ;

Критическими точками 1-го рода являются точки 10,2 21 xx , в ко-торых производная 0y . Точка 43 x , в которой производная y не суще-ствует, не принадлежит области определения функции (рисунок 8).

Рисунок 8

Итак, 42max yy ; 2010min yy .

4. Определим точки перегиба графика функции и интервалы его выпук-лости и вогнутости.

4

22

4

22

4

208442

4

20842482

x

xxxx

x

xxxxxy

34

72

x.

Так как 0у и определена во всей области определения функции, то график заданной функции точек перегиба не имеет. Остаѐтся выяснить вопрос об ин-тервалах его выпуклости и вогнутости (рисунок 9).

Рисунок 9

Page 34: М.Н. Середина, Л.Ю. Шипик МАТЕМАТИКАачии.рф/files/2018-11-16-7b4836c8-02e7-466b-aa5c...2018/11/16  · функции нескольких переменных,

34

Итак, график функции выпуклый на интервале 4; , вогнутый на интер-вале ;4 .

5. Исследование графика на наличие наклонных асимптот bkxy .

1lim4

20lim

)(lim

2

2

2

2

2,1

x

x

xx

x

x

xfk

xxx;

4

204lim

4

20lim)(lim

2

2,1x

xx

x

xkxxfb

xxx

44

lim x

x

x.

Таким образом, прямая 4 xy – наклонная асимптота графика функции

при x .

Рисунок 10

Page 35: М.Н. Середина, Л.Ю. Шипик МАТЕМАТИКАачии.рф/files/2018-11-16-7b4836c8-02e7-466b-aa5c...2018/11/16  · функции нескольких переменных,

35

6. Построение графика. Очевидно, график заданной функции пересекает ось Оу в точке 5;0 и на основе обобщения результатов всех предыдущих исследований имеет вид, представленный на рисунке 10.

Вопросы для самопроверки

1. Как формулируется теорема Лагранжа? 2. Каковы признаки возрастания и убывания функции? 3. Докажите, что функция xxy cos убывает на любом промежутке. 4. Сформулируйте правила нахождения экстремумов функции. 5. Приведите пример, показывающий, что обращение в нуль производной не является достаточным условием экстремума функции. 6. Как найти интервалы выпуклости, вогнутости, точки перегиба графика функции. 7. Дайте определения асимптот графика функции. Как найти вертикальные и наклонные асимптоты.

Задания для аудиторной и самостоятельной работы

Найти интервалы возрастания и убывания следующих функций:

191. 33 xxy . 192. 2

32 1 xy .

193. xxey

. 194. 212 xxy .

195д. 123

2 3 xxy . 196д. 53

3 x

xy .

Исследовать на экстремум следующие функции:

197. 3159 23 xxxy . 198. 42

x

xy .

199. 22

3

3

2

423

4

xxx

y . 200. xxy ln .

201. xxy 3 . 202. 3

122

x

xxy .

203. 2

3

12

x

xy . 204.

22

132

23

x

xxxy .

205д. xxxxy 464 234 . 206д. 2

2x

xey

.

207д. 2

62 2

x

xy . 208д.

2

2

1 x

xy

.

Найти наименьшее и наибольшее значения следующих функций:

Page 36: М.Н. Середина, Л.Ю. Шипик МАТЕМАТИКАачии.рф/files/2018-11-16-7b4836c8-02e7-466b-aa5c...2018/11/16  · функции нескольких переменных,

36

209. 38 24 xxy на отрезке 2;2 .

210. 1

12

x

y на отрезке

2

1;

2

1.

211. 5243 23 xxxy на отрезке 1;5 .

212. xtgxy на отрезке

4;

4

.

213д. 7249 23 xxxy на отрезке 3;6 .

214д. 32 24 xxy на отрезке 2;3 .

215д. 2

2

18 хx

y на отрезке

4;2

1.

216*. Найти положительное число x , чтобы разность 2xx была наиболь-

шей. 217*. Скорость роста популяции x задана формулой xx,y 1000010 , когда время выражается в днях. При каком размере популяции эта скорость макси-мальна? 218д*. Реакция организма на два лекарства как функции t (время выражается в часах) составляют t

te)t(r1 и t

et)t(r 2

2 . У какого из лекарств выше максимальная реакция?

Найти точки перегиба и интервалы выпуклости и вогнутости следующих кривых: 219. 25015 23 xxxy . 220. 444 5 xxy .

221. 2

2

1 x

xy

. 222.

x

xy

1

2

2

.

223. xxey . 224. 1 xxy .

225д. 234 248 xxxy . 226д. x

xy

42 .

227д. arctgxx

y 2

. 228д. 3 1 xy .

Найти асимптоты следующих кривых:

229. 2

41

xy . 230.

x

xy

12 .

231. 1

542

3

x

xy . 232.

3

25 4

x

xy .

233. 4

342

x

xy . 234. x

xey25 .

235д. 1

2

x

xy . 236д.

1

532

4

x

xy .

Page 37: М.Н. Середина, Л.Ю. Шипик МАТЕМАТИКАачии.рф/files/2018-11-16-7b4836c8-02e7-466b-aa5c...2018/11/16  · функции нескольких переменных,

37

237д. 3

105

x

xy . 238д.

7

422

x

xy .

239д. 1

322

3

x

xy . 240д. 3

2

xey .

Исследовать следующие функции и построить их графики:

241. xxxy 18122 23 . 242. 162

x

xy .

243. 2

3 4

x

xy

. 244.

42

3

x

xy .

245. 2

2

x

xy . 246.

2

3

3

x

xy .

247д. 241

xx

y . 248д. x

xy

13 .

249д. 2

4

xxy . 250д.

1

21

2

x

xy .

Примерный вариант контрольной работы № 1

Найти y : 1. 3

3cos2

22

xe

xxy ; 2. x

xtgy 554 ; 3. 35

2sin2

x

xy ;

4. )5ln(arccos xy .

Вычислить указанные пределы:

5. 2

2

1 257

43lim

xx

xx

x

; 6. 334

275lim

2

3

xx

xx

x; 7.

)1ln(

2arcsinlim

2

0 xx

x

x .

8. Исследовать функцию на экстремум: 1

2

x

xy .

9. Найти асимптоты графика функции: 4

53 2

x

xy .

10. Найти точки перегиба; интервалы выпуклости, вогнутости графика функ-ции: 196 23 xxxy .

4 Неопределенный интеграл

Функция xF называется первообразной функции xf на промежутке X , если для всех Xx выполняется равенство xfxF .

Например, функция xxF sin является первообразной функции xxf cos)( на всей числовой прямой, так как xx cos)(sin для всех Rx .

Page 38: М.Н. Середина, Л.Ю. Шипик МАТЕМАТИКАачии.рф/files/2018-11-16-7b4836c8-02e7-466b-aa5c...2018/11/16  · функции нескольких переменных,

38

Множество всех первообразных функции )(xf называется неопреде-ленным интегралом от функции xf и обозначается: CxFdxxf .

Основные свойства неопределенного интеграла

1. Производная неопределенного интеграла равна подынтегральной функции.

2. Дифференциал от неопределенного интеграла равен подынтегрально-му выражению.

3. Неопределенный интеграл от дифференциала функции равен сумме этой функции и произвольной постоянной.

4. Постоянный множитель можно выносить за знак неопределенного ин-теграла, то есть dxxfkdxxkf , где constk .

5. Неопределенный интеграл от суммы (разности) функций равен сумме (разности) интегралов от этих функций, то есть

dxxgdxxfdxxgxf .

Таблица 3 – Основные формулы неопределенных интегралов

1. Cudu . 2.

Cn

uduu

nn

1

1

, 1n .

3. Cuu

du2 . 4. Cu

u

duln .

5. Ca

adua

uu

ln. 6. Cedue

uu .

7. Cuudu cossin . 8. Cuudu sincos .

9. Cu

tgu

du

2ln

sin. 10. C

utg

u

du

42ln

cos

.

11. Cctguu

du2sin

. 12. Ctguu

du2cos

.

13. Cutgudu cosln . 14. Cuctgudu sinln .

15.

Ca

uarctg

aau

du 122

. 16.

Cau

au

aau

duln

2

122

.

17.

Cauu

au

du 22

22ln . 18.

C

a

u

ua

duarcsin

22.

19. Ca

uauauduua

arcsin

2

1 22222 .

20. Cauuaauuduau

2222222 ln

2

1.

Page 39: М.Н. Середина, Л.Ю. Шипик МАТЕМАТИКАачии.рф/files/2018-11-16-7b4836c8-02e7-466b-aa5c...2018/11/16  · функции нескольких переменных,

39

4.1 Непосредственное интегрирование

Отыскание неопределенного интеграла с помощью таблицы основных интегралов и тождественных преобразований называют непосредственным интегрированием.

Если подынтегральная функция представляет собой произведение или частное различных степеней x , то еѐ представляют как степенную функцию вида n

x .

Пример 4.1. Найти dxxx 34 .

Решение. CxxCxdxxdxxxdxxx 353

16

3

13

3

1434

16

3

16

3.

Пример 4.2. Найти dxx

x5

.

Решение. Cxx

Cxdxxdxxxdxx

x

3

2

7

2

952

1

5 7

2

7

2.

При отыскании интеграла от суммы нескольких функций сначала ис-пользуют правила интегрирования суммы и вынесения постоянного множи-теля за знак интеграла (свойства неопределенного интеграла).

Пример 4.3. Найти

.8

9

6cos35

22

dxx

xx

Решение.

xdxdxxdxx

xx cos3589

6cos35 2

22

3

2sin33

58

33

6sin3

358

96

33

2

xarctgx

xCx

xarctgx

xdx

x

dx

Cx 8 . Замечание. Нет необходимости после каждого слагаемого ставить про-

извольную постоянную, потому что сумма произвольных постоянных есть также произвольная постоянная, которую записывают в конце.

Некоторые интегралы могут быть найдены после выполнения тожде-ственных преобразований (раскрытие скобок, почленное деление числителя на знаменатель, применение тригонометрических и других формул).

Пример 4.4. Найти

dxx

xx

2

4 65.

Решение.

dxxx

dxdxxdx

xxxdx

x

xx 222

22

4

656565

Cx

xx

6

ln53

3

.

Page 40: М.Н. Середина, Л.Ю. Шипик МАТЕМАТИКАачии.рф/files/2018-11-16-7b4836c8-02e7-466b-aa5c...2018/11/16  · функции нескольких переменных,

40

Пример 4.5. Найти

dx

x

x23

.

Решение.

dx

xx

x

x

xdx

x

xxdx

x

x 96963 222

xxxxCxxxx

dxdxxdxx 4

5

2294

5

296 22

3

2

5

2

1

2

3

.18 Cx

Пример 4.6. Найти dxx

x

cos

sin 2

.

Решение.

xdxdxx

dxx

xdx

x

xcos

cos

1

cos

cos1

cos

sin 22

Cxx

tg

sin

42ln

.

Пример 4.7. Найти 299

2

x

dx.

Решение.

2222 13

2

13

2

19

2

99

2

x

dx

x

dx

x

dx

x

dx

Cx arcsin3

2.

Иногда полезно воспользоваться искусственными приемами, например, к числителю дроби прибавить и из него вычесть одно и то же выражение, по-сле чего разделить числитель на знаменатель.

Пример 4.8. Найти

dxx

x

82

2

.

Решение.

dxdx

xdx

x

xdx

x

x

8

81

8

88

8 22

2

2

2

Cx

arctgxCx

arctgxx

dx

2222

2222

18

88

2.

4.2 Интегрирование методом подстановки

Существует два варианта этого метода. Метод подведения под знак дифференциала:

если )(xt − дифференцируемая функция (при этом )(tddt dxx)( ),

то

)()()()()()(

xtdttfxdxfdxxxf

.

Page 41: М.Н. Середина, Л.Ю. Шипик МАТЕМАТИКАачии.рф/files/2018-11-16-7b4836c8-02e7-466b-aa5c...2018/11/16  · функции нескольких переменных,

41

Пример 4.9. Найти xdx7sin .

Решение. Так как dxxd 77 , то xddx 77

1 .

Имеем: Cxxdxxdxxdx 7cos7

177sin

7

17

7

17sin7sin .

Пример 4.10. Найти dxx974 .

Решение. Так как dxxd 474 , то 744

1 xddx .

Имеем: 74744

174

4

17474 999

xdxxdxdxx

CxCx

1010

7440

1

10

74

4

1.

Метод подстановки:

пусть требуется найти dxxf )( , применяя подходящую подстановку )(tx

(при этом dttdx )( ), получим dtttfdxxf )()()( . То есть вычисле-ние данного интеграла сводится к вычислению нового интеграла (который может оказаться проще данного) и последующей подстановке )(1

xt .

Пример 4.11. Найти 72

x

xdx.

Решение. Сделаем замену переменной.

CxCuu

du

u

du

duxdx

xdxdu

xu

x

xdx

7ln2

1ln

2

1

2

12

2

2

7

7

2

2

2.

Пример 4.12. Найти

dx

x

x

97

462

.

Решение. Разделим почленно числитель на знаменатель:

97

4

97

6

97

46222

x

dx

x

xdxdx

x

x.

В каждом из интегралов сделаем замену переменных, тогда

Page 42: М.Н. Середина, Л.Ю. Шипик МАТЕМАТИКАачии.рф/files/2018-11-16-7b4836c8-02e7-466b-aa5c...2018/11/16  · функции нескольких переменных,

42

9

74

146

7

7

7

7

14

14

97

97

4

97

6

97

462

22

2

222v

dv

u

du

dvdx

dxdv

xv

xv

duxdx

xdxdu

xu

x

dx

x

xdxdx

x

x

.37

37ln

73

2

97ln7

3

3

3ln

32

1

7

4ln

7

3

37

4

14

6 222

Cx

x

xCv

vu

v

dv

u

du

Замечание. Интегралы, которые находят подведением под знак диф-ференциала, можно также найти методом подстановки.

Пример 4.13. Найти 32x

dx.

Решение. Сделаем замену переменной.

CxCuu

du

u

du

dudx

dxdu

xu

x

dx

32ln

2

1ln

2

1

2

12

2

2

32

32.

Пример 4.14. Найти dx

x 3

1.

Решение. Сделаем замену переменной.

362

3

312

3

332

3

22

3

2

t

dtdtdt

tdt

t

t

t

tdt

xt

tdtdx

tx

x

dx

Cxxxt

Cttt

tdt 3ln623ln62

3

362 .

Пример 4.15. Найти 833cos

522

xtgx

dx.

Решение. Сделаем замену переменной.

83

5

8

35

33cos

3cos

3

3

833cos

522

2

222u

du

u

du

du

x

dx

dxx

du

xtgu

xtgx

dx

Page 43: М.Н. Середина, Л.Ю. Шипик МАТЕМАТИКАачии.рф/files/2018-11-16-7b4836c8-02e7-466b-aa5c...2018/11/16  · функции нескольких переменных,

43

Cxtg

arctgCu

arctg 22

3

26

5

2222

1

3

5.

4.3 Интегралы, содержащие квадратный трехчлен

Интегралы вида cbxax

dx2

и cbxax

dx

2 приводятся к табличным

интегралам путем выделения полного квадрата в квадратном трехчлене:

a

c

a

b

a

bx

a

bxa

a

cx

a

bxacbxax

22222

2222

2

22

4

4

2 a

acb

a

bxa .

Пример 4.16. Найти 7105 2

xx

dx.

Решение. Вынесем в знаменателе коэффициент при 2x :

5

7257105 2

2xx

dx

xx

dx.

Так как 5

121

5

71112

5

72 2222 xxxxx , то

.1215

1215ln

154

1

5

121

5

121

ln

5

122

1

5

1

5

121

1

5

1

5

1215

1

5

7257105 2

222

2

Cx

xC

x

x

x

xd

x

dx

xx

dx

xx

dx

Пример 4.17. Найти 267 xx

dx.

Решение. Так как 7667 22xxxx

79332 22xx 22 316163 xx , то

Cx

x

xd

x

dx

xx

dx

4

3arcsin

34

3

31667 2222.

Page 44: М.Н. Середина, Л.Ю. Шипик МАТЕМАТИКАачии.рф/files/2018-11-16-7b4836c8-02e7-466b-aa5c...2018/11/16  · функции нескольких переменных,

44

4.4 Метод интегрирования по частям

Если xu и xv – дифференцируемые функции, то справедлива форму-ла интегрирования по частям:

duvvudvu . (4.1)

Перечислим основные типы интегралов, которые находят методом инте-грирования по частям: 1. dxexP

kxn , dxkxxPn sin , dxkxxPn cos ,

где 011

1 ... CxCxCxCxPn

nn

nn – многочлен n-ой степени с дей-

ствительными коэффициентами; constk . В интегралах такого типа полагают xPu n .

2. dxkxxPn ln , kxdxxPn arcsin , dxkxxPn arccos , dxarctgkxxPn ,

dxarcctgkxxPn .

За u принимают множитель при xPn , так как при дифференцировании ло-гарифмическая и обратные тригонометрические функции упрощаются.

Пример 4.18. Найти xdxх sin25 .

Решение. Полагаем

xxdxvxdxdv

dxduxu

cossinsin

525, тогда

Cxxxxdxxхxdxх sin5cos25cos5cos25sin25 .

Замечание. При нахождении v произвольную постоянную C полагают равной нулю, то есть в качестве v берут одну из первообразных.

Пример 4.19. Найти dxx3arcsin2 .

Решение. Полагаем

xdxvdxdv

dxx

duxu291

33arcsin

, тогда

291

33arcsin23arcsin2

x

dxxxxdxx .

Далее сделаем замену переменной.

xx

dtdxx

dxxdt

xt

x

dxxxxdxx 3arcsin2

18

1

18

91

91

33arcsin23arcsin2

2

2

xxCtxxt

dtxx

t

dt

3arcsin223

13arcsin2

3

13arcsin218

1

6

Page 45: М.Н. Середина, Л.Ю. Шипик МАТЕМАТИКАачии.рф/files/2018-11-16-7b4836c8-02e7-466b-aa5c...2018/11/16  · функции нескольких переменных,

45

Cx 2913

2.

Пример 4.20. Найти dxx

x 5

2ln.

Решение. Полагаем

4

55

2

4

1

1ln2ln

xdxxv

x

dxdv

dxx

xduxu

, тогда

dxx

x

x

xdx

x

x54

2

5

2

2

ln

4

lnln.

К стоящему справа интегралу снова применяем формулу интегрирования по

частям, полагая

4

55 8

1

2

1

2

1ln

xdxxv

x

dxdv

dxx

duxu

.

Следовательно, Cxx

x

x

x

x

dx

x

x

x

xdx

x

x444

2

544

2

5

2

32

1

8

ln

4

ln

8

1

8

ln

4

lnln.

Пример 4.21. Найти dxexx 72 1 .

Решение. Полагаем

xxxedxevdxedv

xdxduxu

777

2

7

1

21, тогда

dxexex

dxeх xxx 772

72

7

2

7

11 .

К стоящему справа интегралу снова применяем формулу интегрирования по

частям, полагая

xxxedxevdxedv

dxduxu

777

7

1 .

Следовательно,

dxeex

ex

dxexxxxx 777

272

7

1

77

2

7

11

xxe

xe

x 772

49

2

7

1

Cex 7

343

2.

Пример 4.22. Найти dxxarctg 118 .

Решение. Полагаем

Page 46: М.Н. Середина, Л.Ю. Шипик МАТЕМАТИКАачии.рф/files/2018-11-16-7b4836c8-02e7-466b-aa5c...2018/11/16  · функции нескольких переменных,

46

xvdxdv

xx

dxdx

xxduxarctgu

11821182

18

1181

1118

2 , тогда

1181182

118118 xarctgxxx

xdxxarctgxdxxarctg

Cxxarctgx

x

xd

11818

1118

118

118

18

1

2

1.

4.5 Интегрирование рациональных дробей

Рациональной дробью называется дробь вида xQ

xP

n

m , где xPm и xQn

– многочлены с действительными коэффициентами.

Если степень xPm больше или равна степени xQn , то дробь xQ

xP

n

m − не-

правильная. Тогда в этой дроби надо выделить целую часть (делением чис-лителя на знаменатель).

Выделив в неправильной дроби целую часть, мы сведѐм интегрирование этой дроби к интегрированию многочлена и правильной рациональной дро-би.

Чтобы проинтегрировать правильную рациональную дробь xQ

xP

n

m , надо

разложить еѐ на сумму простейших дробей, то есть дробей вида

ax

A

, kax

A

,

qpxx

NMx

2

, kqpxx

NMx

2

,

где ,...3,2k ,

qpaNMA ,,,,, – постоянные,

квадратный трѐхчлен qpxx 2 не имеет действительных корней.

Для того чтобы разложить правильную рациональную дробь xQ

xP

n

m ,

представим знаменатель xQn в виде произведения линейных и квадратич-ных множителей:

skv

ss

vtk

ttnn qxpxqxpxaxaxaxCxQ 2

112

21 ......121

где nvvttt sk ...2... 121 .

Правильную рациональную дробь xQ

xP

n

m можно единственным спосо-

бом представить в виде суммы простейших дробей, причѐм:

Page 47: М.Н. Середина, Л.Ю. Шипик МАТЕМАТИКАачии.рф/files/2018-11-16-7b4836c8-02e7-466b-aa5c...2018/11/16  · функции нескольких переменных,

47

1) линейному множителю ax знаменателя соответствует дробь вида

ax

A

;

2) линейному множителю kax знаменателя соответствует сумма дробей

вида k

k

ax

A

ax

A

ax

A

...

221 , где ,...3,2k ;

3) квадратичному множителю qpxx 2 знаменателя соответствует дробь

вида qpxx

NMx

2

;

4) квадратичному множителю kqpxx 2 знаменателя соответствует сум-

ма дробей вида k

kk

qpxx

NxM

qpxx

NxM

qpxx

NxM

222

222

11 ... , где ,...3,2k

Пример 4.23. Если в правильной рациональной дроби xQ

xPm

3

знамена-

тель 123 xxxxQ ,

то 123

x

C

x

B

x

A

xQ

xPm ,

где CBA ,, – коэффициенты, которые надо определить.

Пример 4.24. Если в правильной рациональной дроби xQ

xPm

4

знамена-

тель 13 224 xxxxQ , то

13 22

4

xx

DCx

x

BAx

xQ

xPm ,

где DCBA ,,, – коэффициенты, которые надо определить.

Пример 4.25. Если в правильной рациональной дроби xQ

xPm

4

знамена-

тель 34 52 xxxQ , то 32

4 5552

x

D

x

C

x

B

x

A

xQ

xPm ,

где DCBA ,,, – коэффициенты, которые надо определить. Замечание. При интегрировании рациональных дробей следует вос-

пользоваться формулами:

CaxAdxax

A

ln ; (4.2)

Ck

axAdx

ax

Ak

k

1

1

, где ,...3,2k . (4.3)

Методы нахождения коэффициентов рассмотрим на примерах.

Page 48: М.Н. Середина, Л.Ю. Шипик МАТЕМАТИКАачии.рф/files/2018-11-16-7b4836c8-02e7-466b-aa5c...2018/11/16  · функции нескольких переменных,

48

Пример 4.26. Найти dxxx

x

32

23.

Решение. Правильную рациональную дробь, стоящую под знаком инте-грала, разложим на сумму простейших:

3232

23

x

B

x

A

xx

x.

Умножим обе части данного равенства на знаменатель 32 xx :

2323 xBxAx .

Первый метод нахождения неопределенных коэффициентов заключает-ся в том, что, полагая последовательно в этом тождестве значения x равными действительным корням знаменателя, сразу находим коэффициенты.

При 2x , получим A58 или 5

8A .

При 3x , получим B57 или 5

7B .

Тогда Cxxdxx

dxx

dxxx

x

3ln5

72ln

5

8

35

7

25

8

32

23.

Замечание. В некоторых случаях вычисление коэффициентов с помо-щью первого метода приводит к решению громоздких систем линейных уравнений.

Пример 4.27. Найти dxxx

xxx

925

7046422

23

.

Решение. Правильную рациональную дробь, стоящую под знаком инте-грала, разложим на сумму простейших:

.925925

704642222

23

x

DCx

x

BAx

xx

xxx

Умножим обе части данного равенства на знаменатель 925 22 xx :

25970464 2223 xDCxxBAxxxx

и раскроем скобки: DDxCxCxBBxAxAxxxx 25259970464 232323 .

Второй метод нахождения неопределенных коэффициентов основан на том, что коэффициенты при одинаковых степенях x в левой и правой частях последнего тождества должны совпадать, то есть

.70259

,4259

,6

,4

0

2

3

DB

CA

DB

CA

x

x

x

x

Решив систему полученных уравнений, имеем 1,2,5,6 DCBA .

Page 49: М.Н. Середина, Л.Ю. Шипик МАТЕМАТИКАачии.рф/files/2018-11-16-7b4836c8-02e7-466b-aa5c...2018/11/16  · функции нескольких переменных,

49

Тогда dxxx

xxx

925

7046422

23

25

23

9

12

25

56222

x

xdxdx

x

xdx

x

x

9

2

55

222x

xdx

x

dx

u

du

xdxdv

xv

xdxdu

xu

x

dx3

2

9

2

25

3

22

22

Cx

arctgvx

arctgux

arctgv

dvxarctg

33

1ln

5ln3

33

1

55

15

.33

19ln

525ln3 22

Cx

arctgxx

arctgx

Пример 4.28. Найти

32

5

xx

dxx.

Решение. Правильную рациональную дробь, стоящую под знаком инте-грала, разложим на сумму простейших:

3323

12222

5

xx

x

D

x

C

x

B

x

A

xx

x

DxxCxxBxxAx 2225 23 или

.24481265 22323DxxxCxxxBxxxAx

Применим оба метода нахождения коэффициентов:

При 0x , получим A85 или 8

5A ;

При 2x , получим D27 или 2

7D .

Приравняем коэффициенты при 3x и 2

x :

.460

,02

3

CBA

BA

x

x

Решив систему уравнений, имеем 4

5,

8

5 CB .

Тогда

xdx

xdx

xdx

xdx

xxx

dxxln

8

5

2

2

7

2

4

5

28

5

8

5

2

5323

1

2

4

52ln

8

5 1x

x

2ln8

5ln

8

5

2

2

2

7 2

xxCx

24

5

x

.

24

72

Cx

Вопросы для самопроверки

1. Сформулируйте определение первообразной. 2. Каковы основные свойства неопределенного интеграла?

Page 50: М.Н. Середина, Л.Ю. Шипик МАТЕМАТИКАачии.рф/files/2018-11-16-7b4836c8-02e7-466b-aa5c...2018/11/16  · функции нескольких переменных,

50

3. Укажите целесообразные подстановки для отыскания интегралов

dxx

x

ln, xdxe

x sincos , dxxx32 1 ,

21 x

xdx.

4. Каким путем интегралы, содержащие квадратный трехчлен, приводятся к табличным?

5. Запишите формулу интегрирования по частям. 6. Что называется рациональной дробью? 7. Как разложить правильную рациональную дробь на сумму простейших? 8. Какие методы нахождения неопределенных коэффициентов вы знаете?

Задания для аудиторной и самостоятельной работы

Используя приѐм непосредственного интегрирования, метод подстанов-

ки или подведение под знак дифференциала, найти следующие интегралы:

251. dxxx 423 2 . 252. dxxx

x

2

32. 253. dx

x

xx

2

3 25.

254. dxxx2

12 . 255. dxx87 . 256. dxx 8cos .

257. 52x

dx. 258. dxxctg 65 . 259. dxx

x 52 .

260. dxxx 4cos3 . 261. 13sin

22

x

xdx. 262. dxxe

x 32

.

263. dxxx 52sin 32 . 264. 43

2

2x

xdx. 265. dx

x

x

45

32

.

266.

dx

xx

x

53

322

. 267. .sincos4 6 dxxx 268. dxx

x

112

73

2

.

269. dxx

x

4

3

2

3. 270. dxxx

532 35 . 271.

dxx

extg

3cos2

13

.

272.

dx

x

x5ln23

. 273. 24 x

dx. 274.

dx

x

x

7

142

2

.

275. 19

32

x

dx. 276.

35 2x

dx. 277. dx

x 52

22

.

278. 232

4

x

dx. 279.

92 6

2

x

dxx. 280. dx

x

x

49

2.

281. 52cos

2sin2

x

xdx. 282. dx

x

x

94

732

. 283.

dx

x

x

294

32.

Page 51: М.Н. Середина, Л.Ю. Шипик МАТЕМАТИКАачии.рф/files/2018-11-16-7b4836c8-02e7-466b-aa5c...2018/11/16  · функции нескольких переменных,

51

284. dxx

x

73

532

. 285.

dxx

xtg2

4

cos

3. 286.

x

x

e

dxe

2

2

32.

287. dxxx2sin132cos 288.

dx

x

xarctgx

241

2. 289. dx

x

xx

ln35

.

290д. dxxx43 524 . 291д. dx

xx

7

13. 292д.

dx

x

xx3

4 6.

293д. dxxxx 253 2 . 294д. dxe

x 2 . 295д. dxxctg 8 .

296д. dxx 13sin . 297д. dx

x 54

2. 298д. dxxxtg 12 .

299д. 62

x

xdx. 300д. dx

x

х

7

3

2. 301д.

dx

x

x

5

22

2

.

302д. 224

4

x

dx. 303д.

235 x

dx. 304д.

84x

xdx.

305д.

.3 2x

x

e

dxe 306д. dx

x

x

3

27

2 307д.

.

32

432

dxx

x

308д. dxx

x2

1sin

. 309д. .32 55dxectge

xx 310д. dxe

ex

x

2

5 1.

Используя метод интегрирования квадратного трѐхчлена, найти следу-ющие интегралы:

311. 142

xx

dx. 312.

223 2xx

dx.

313. 62

xx

dx. 314.

762xx

dx.

315д. 102

32

xx

dx . 316д. 852

xx

dx.

317д. 2215 xx

dx . 318д. 782 2

xx

dx.

Используя метод интегрирования по частям, найти следующие интегра-лы: 319. xdxx 2sin . 320. xdxx 3cos23 . 321. dxex

x23 .

322. dxexx2 . 323. xdxarcctg2 . 324. xdx2ln .

325. xdxarcsin . 326. dxx

x

ln. 327. xarctgxdx .

328. xdxx ln3 . 329. dxxarctg 14 . 330. x

xdx2sin

.

Page 52: М.Н. Середина, Л.Ю. Шипик МАТЕМАТИКАачии.рф/files/2018-11-16-7b4836c8-02e7-466b-aa5c...2018/11/16  · функции нескольких переменных,

52

331д. xdxx sin32 . 332д. xdxx 2cos1 . 333д. dxxe

x .

334д. xdxarccos . 335д. xdxarctg3 . 336д. .ln12 dxxx

Используя метод интегрирования рациональных дробей, найти следую-щие интегралы:

337.

.25

516dx

xx

x 338.

.

432

4132 2

dxxxx

xx

339.

.

5

152

2

dxxx

x 340.

.

32

13342

2

dxxx

xx

341.

dx

xx

xx

21

592

2

. 342д.

.93

1dx

xx

x

343д.

.24

6426dx

xxx

x 344д.

.73 xxx

dx

345д.

.

41

732

2

dxxx

xx 346д.

dx

xx

x

31

572

.

5 Определѐнный интеграл и его применение

5.1 Вычисление определѐнных интегралов

Если xF – одна из первообразных функции xf , непрерывной на от-резке ba; , то справедлива формула Ньютона-Лейбница

aFbFxFdxxf

b

a

b

a

. (5.1)

Замечание. При вычислении определѐнного интеграла применяют те же методы, что и при нахождении неопределенных интегралов.

Пример 5.1. Вычислить

1

03

2

1 x

dxх.

Решение. Применим метод подведения под знак дифференциала.

Так как dxxxd23 31 , то 32 1

3

1xddxx .

Имеем: 2ln3

11ln2ln

3

11ln

3

1

1

1

3

1

1

1

0

31

03

31

03

2

x

x

xd

x

dxх.

Если функция xf непрерывна на отрезке ba; , а функция tx

непрерывно дифференцируема на отрезке 21; tt , причѐм, 1ta , 2tb ,

то справедлива формула замены переменной в определѐнном интеграле:

Page 53: М.Н. Середина, Л.Ю. Шипик МАТЕМАТИКАачии.рф/files/2018-11-16-7b4836c8-02e7-466b-aa5c...2018/11/16  · функции нескольких переменных,

53

2

1

t

t

b

a

dttxfdxxf . (5.2)

Замечание. При вычислении определѐнного интеграла с помощью за-мены переменной нет необходимости возвращаться к прежней переменной, надо только пересчитать пределы интегрирования. Замену пределов интегрирования удобно записывать в виде таблицы:

x t

a 1t

b 2t

Пример 5.2. Вычислить

8

3 1 x

dxx.

Решение. Сделаем замену переменной.

tdtdx

tx

tx

x

dxx

2

1

1

1

2

2

8

3

3

2

33

2

23

2

2

3212

21t

tdtt

t

tdtt

.3

322

3

82392

Пример 5.3. Вычислить dxxx sincos2

0

5

.

Решение. Здесь xcos с нечетным положительным показателем степени. Отделим от x

5cos один сомножитель, используем формулу xx22 sin1cos

и сделаем замену переменных:

2

0

222

0

42

0

5 cossinsin1cossincossincos

dxxxxdxxxxdxxx

1

0

1

0

4222 211 duuuuduuu xdxdu

xu

cos

sin

0

1

11

2

7

4

3

2

0

1

2

11

2

72

2

32 2

11

2

7

2

32

11

2

7

2

31

0

2

9

2

5

2

1

uuuuuu

duuuu

.231

64

11

2

7

4

3

2

Если функции xuu и xvv имеют непрерывные производные на

отрезке ba; , то справедлива формула

x t 8 3 3 2

x u 2 1

0 0

Page 54: М.Н. Середина, Л.Ю. Шипик МАТЕМАТИКАачии.рф/files/2018-11-16-7b4836c8-02e7-466b-aa5c...2018/11/16  · функции нескольких переменных,

54

b

a

b

a

b

a

duvvudvu , (5.3)

которая называется формулой интегрирования по частям для определѐнно-го интеграла.

Пример 5.4. Вычислить 2/

0

cos

dxxx .

Решение. Полагаем:

xxdxvxdxdv

dxduxu

sincoscos, тогда

2

0

2

0

2

0

2

0

cos02

sin2

sinsincos

xdxxxxdxxx 12

.

Пример 5.5. Вычислить e

dxxx

1

2ln .

Решение. Полагаем:

2

1ln2ln

2

2

xdxxvxdxdv

dxx

xduxu

, тогда

e

e

xx

dxxx

1

22

1

2 ln2

ln

ee

dxxxe

dxx

xx

1

2

1

2

ln02

1ln2

2.

К стоящему справа интегралу снова применяем формулу интегрирования по

частям, полагая

2

1ln

2x

dxxvdxxdv

dxx

duxu

.

Следовательно,

0

22

11

2ln

22ln

22

2

1

22

1

2 eedx

x

xx

xedxxx

ee

14

1

22

1

2

1 2

1

2

1

ex

dxx

ee

.

5.2 Вычисление площадей плоских фигур

Площадь фигуры, ограниченной двумя линиями xfy 1 , xfy 2 на отрезке ba; , где xf2 xf1 (рисунок 11), вычисляется по формуле

Page 55: М.Н. Середина, Л.Ю. Шипик МАТЕМАТИКАачии.рф/files/2018-11-16-7b4836c8-02e7-466b-aa5c...2018/11/16  · функции нескольких переменных,

55

dxxfxfb

a

12S . (5.4)

Рисунок 11

Если кривая AB задана параметрическими уравнениями txx ,

tyy , то площадь криволинейной трапеции, ограниченной прямыми ax ,

bx , кривой AB и осью Ох, находим по формуле

dttxty

t

2

1t

S , (5.5)

где btxatx 21 , .

В случае, когда кривая задана в полярных координатах уравнением rr , площадь криволинейного сектора, ограниченного дугой кривой и

двумя полярными радиусами, проведѐнными под углом и , выражается интегралом

dr

2

2

1S . (5.6)

Пример 5.6. Вычислить площадь фигуры, ограниченной линиями 23 xxy и xy .

Решение. Строим искомую фигуру (рисунок 12). Находим абсциссы точек пересечения данных линий:

xxx 23 или 042 xx ; 01 x , 42 x .

Согласно формуле (5.4) имеем:

Page 56: М.Н. Середина, Л.Ю. Шипик МАТЕМАТИКАачии.рф/files/2018-11-16-7b4836c8-02e7-466b-aa5c...2018/11/16  · функции нескольких переменных,

56

.3

32

3

64162

3243S

4

0

4

0

4

0

3222

xxdxxxdxxxx

Рисунок 12

Пример 5.7. Вычислить площадь фигуры, ограниченной линиями 22 2 xy , xy 210 , 2y .

Решение. Строим искомую фигуру (рисунок 13). Для вычисления площади данной фигуры целесообразно воспользовать-

ся следующей формулой:

dyyyd

c

12S , (5.7)

где y2 y1 на отрезке dc; .

В нашем случае 22 2 xy или 2

2

yx , следовательно,

2

21

yy ;

xy 210 или 2

10 yx

, следовательно,

2

102

yy

; 2c ; 6d .

Тогда

6

2

2

16

2

6

2

6

2

222

1

2

15

2

2

2

10ydydyydydy

yyS

Page 57: М.Н. Середина, Л.Ю. Шипик МАТЕМАТИКАачии.рф/files/2018-11-16-7b4836c8-02e7-466b-aa5c...2018/11/16  · функции нескольких переменных,

57

6

2

2

3

6

2

2

36

2

262

23

2436

4

1265

2

3

2

2

1

4

15 y

yyy

.3

64

3

3296216

3

23208

3

2840 2

3

Рисунок 13

Пример 5.8. Вычислить площадь фигуры, ограниченной линиями xy 2 , yx 3 , yx 210 .

Решение. Строим искомую фигуру (рисунок 14). Находим абсциссы точек пересечения данных линий:

а)

yx

xy

3

2 01 x ;

б)

yx

yx

3

210 62 x ;

Page 58: М.Н. Середина, Л.Ю. Шипик МАТЕМАТИКАачии.рф/files/2018-11-16-7b4836c8-02e7-466b-aa5c...2018/11/16  · функции нескольких переменных,

58

в)

xy

yx

2

210 23 x .

Рисунок 14

Для вычисления площади данной фигуры необходимо разбить ее на две части вертикальной прямой 2x и посчитать как сумму площадей получен-ных фигур:

21 SSS , где согласно формуле (5.4)

3

10

3

212

6

44

63

12

2

0

22

2

01

xxdxxxS ;

12

36565

12

55

6

55

325

6

2

26

2

6

22

xxdx

xdx

xxS

3

20

3

5101530

12

4525

.

Тогда 103

30

3

20

3

10S .

5.3 Вычисление длины дуги

Пусть дуга AB кривой задана уравнением xfy , где bxa . Тогда длина l дуги AB вычисляется по формуле

b

a

dxyl .1 2 (5.8)

Page 59: М.Н. Середина, Л.Ю. Шипик МАТЕМАТИКАачии.рф/files/2018-11-16-7b4836c8-02e7-466b-aa5c...2018/11/16  · функции нескольких переменных,

59

В случае, когда кривая AB задана параметрическими уравнениями txx , tyy , длина l дуги вычисляется по формуле

2

1

22t

t

tt dtyxl , (5.9)

где t1 и t2 – значения параметра t, соответствующие концам дуги AB . Если кривая задана в полярных координатах уравнением rr , то еѐ

длина l вычисляется по формуле

drrl

22 , (5.10)

где и − пределы изменения полярного угла.

Пример 5.9. Вычислить длину дуги кривой 3

3

2xy , абсциссы концов

которой 31 x и 82 x .

Решение. Воспользуемся формулой (5.8). Так как xy , то

3

38827

3

249

3

2

2

3

1111 2

3

2

38

3

8

3

8

3

2

3

2

12

xxdxdxxl .

Пример 5.10. Вычислить длину одной арки циклоиды tay cos1 ,

ttax sin .

Решение. Поскольку все арки циклоиды одинаковы, рассмотрим первую еѐ арку, вдоль которой параметр t изменяется от 0 до 2. Воспользуемся формулой (5.9). Учитывая, что taxt cos1 , tayt sin , имеем:

2

0

2

0

222222 sincoscos21sincos1 dttttadttatal

2

0

2

0

2

0

2

0 2cos4

22sin22

2sin2cos12

ta

td

tadt

tadtta

aaa 8240coscos4 .

Здесь мы воспользовались формулой 2

sin2cos1 2 tt .

Пример 5.11. Вычислить длину первого витка логарифмической спира-ли

er .

Решение. Воспользуемся формулой (5.10). Так как er , то

1222 2

2

0

2

0

2

0

222

0

22

eededeedeel .

Page 60: М.Н. Середина, Л.Ю. Шипик МАТЕМАТИКАачии.рф/files/2018-11-16-7b4836c8-02e7-466b-aa5c...2018/11/16  · функции нескольких переменных,

60

5.4. Вычисление объѐмов тел вращения

Объѐм тела, полученного вращением вокруг оси Ох плоской фигуры, ограниченной линиями xyy 1 , xyy 2 , ax , bx (рисунок 15), выра-жается формулой

b

a

x dxxyxyV21

22 , (5.11)

где xyxy 12 на отрезке ba; .

Рисунок 15

Рисунок 16

Page 61: М.Н. Середина, Л.Ю. Шипик МАТЕМАТИКАачии.рф/files/2018-11-16-7b4836c8-02e7-466b-aa5c...2018/11/16  · функции нескольких переменных,

61

Объѐм тела, полученного вращением вокруг оси Oy плоской фигуры, ограниченной линиями yxx 1 , yxx 2 , cy , dy (рисунок 16), выра-жается формулой

d

c

y dyyxyxV21

22 , (5.12)

где yxyx 12 на отрезке dc; .

Пример 5.12. Найти объѐм тела, полученного вращением вокруг оси Oy

криволинейной трапеции, ограниченной дугой параболы 4 yx и осями координат.

Решение. Изобразим плоскую фигуру, ограниченную заданными лини-ями (рисунок 17).

Рисунок 17

Воспользуемся формулой (5.12). В нашем случае 01 yx , 42 yyx ,

4c , 0d . Тогда

8162

164

244

0

4

20

4

20

4

y

ydyydyyVy .

Пример 5.13. Найти объѐм тела, полученного вращением вокруг оси Ox

плоской фигуры, ограниченной линиями 2xy , xy 23 .

Решение. Изобразим плоскую фигуру, ограниченную заданными лини-ями (рисунок 18).

Page 62: М.Н. Середина, Л.Ю. Шипик МАТЕМАТИКАачии.рф/files/2018-11-16-7b4836c8-02e7-466b-aa5c...2018/11/16  · функции нескольких переменных,

62

Рисунок 18

Воспользуемся формулой (5.11). Учитывая, что 21 xxy , xxy 232 ,

3a , 1b , имеем

1

3

421

3

222 412923 dxxxxdxxxVx

.15

1088

5

243365427

5

1

3

469

53

469

1

3

532

xxxx

5.5 Численность популяции

Число особей в популяции (численность популяции) меняется со време-нем. Если условия существования популяции благоприятны, то рождаемость превышает смертность и общее число особей в популяции растет со време-нем. Назовем скоростью роста популяции прирост числа особей в единицу времени. Обозначим эту скорость tvv . В «старых» установившихся попу-ляциях, давно обитающих в данной местности, скорость роста tv мала и медленно стремиться к нулю. Но если популяция молода, еѐ взаимоотноше-ния с другими местными популяциями еще не установились или существуют внешние причины, изменяющие эти взаимоотношения, например, сознатель-

Page 63: М.Н. Середина, Л.Ю. Шипик МАТЕМАТИКАачии.рф/files/2018-11-16-7b4836c8-02e7-466b-aa5c...2018/11/16  · функции нескольких переменных,

63

ное вмешательство человека, то tv может значительно колебаться, умень-шаясь или увеличиваясь.

Если известна скорость роста популяции tv , то мы можем найти при-рост численности популяции за промежуток времени от 0t до T . В самом де-ле, из определения tv следует, что эта функция является производной от численности популяции tN в момент t , и, следовательно, численность по-пуляции tN является первообразной для tv . Поэтому

T

t

dttvtNTN

0

0 . (5.13)

Известно, что в условиях неограниченных ресурсов питания скорость роста многих популяций экспоненциальна, то есть kt

eatv . Популяция в этом случае как бы «не стареет». Такие условия можно создать, например, для микроорганизмов, пересаживая время от времени развивающуюся куль-туру в новые емкости с питательной средой. Применяя формулу (12), в этом случае получаем

0

00

000ktkT

T

t

ktT

t

ktee

k

atNe

k

atNdteatNtN . (5.14)

По формуле, подобной (5.14), подсчитывают, в частности, численность культивируемых плесневелых грибков, выделяющих пенициллин.

6 Несобственные интегралы

6.1 Несобственные интегралы с бесконечными пределами

интегрирования

Если функция xf непрерывна на промежутке ;a , то несобствен-ным интегралом с бесконечным пределом интегрирования (или несоб-ственным интегралом первого рода) называется предел

b

ab

a

dxxfdxxf lim . (6.1)

Если предел в правой части равенства (6.1) существует и конечен, то не-собственный интеграл называется сходящимся, в противном случае – расхо-дящимся.

Если xF – первообразная для подынтегральной функции xf , то

aFFaFbFdxxfb

a

lim ,

где bFFb

lim .

Аналогичным образом определяется интеграл на промежутке b; :

Page 64: М.Н. Середина, Л.Ю. Шипик МАТЕМАТИКАачии.рф/files/2018-11-16-7b4836c8-02e7-466b-aa5c...2018/11/16  · функции нескольких переменных,

64

b

aa

b

dxxfdxxf lim . (6.2)

Несобственный интеграл с двумя бесконечными пределами можно определить как сумму несобственных интегралов (6.1) и (6.2)

c

c

dxxfdxxfdxxf , (6.3)

где c − любое число. Несобственный интеграл с двумя бесконечными пределами сходится,

если сходятся оба интеграла, стоящие в правой части равенства (6.3).

Пример 6.1. Вычислить интеграл

125x

dx.

Решение.

1 11

12

12 5

1

1

555

5 x

xxdx

x

dx

5

1lim

bb 6

1

6

10

6

1 , то есть интеграл сходится.

Пример 6.2. Вычислить интеграл

2

41 x

dx.

Решение.

222

4124

1

41

41

4

1

41x

x

xd

x

dx

2

9

2

412

x

2

3

2

41lim

a

a, то есть интеграл расхо-

дится.

Пример 6.3. Вычислить интеграл

21 x

dx.

Решение. Воспользуемся формулой (6.3):

c

c

c

c

arctgxarctgxx

dx

x

dx

x

dx222 111

arctgaarctgc

alim

arctgbarctgcarctgb

bblimlim

,22

lim

arctga

a то есть интеграл сходится.

6.2 Несобственные интегралы от неограниченных функций

Если функция xf непрерывна на промежутке ba; , при этом

Page 65: М.Н. Середина, Л.Ю. Шипик МАТЕМАТИКАачии.рф/files/2018-11-16-7b4836c8-02e7-466b-aa5c...2018/11/16  · функции нескольких переменных,

65

xfbx 0

lim , то несобственным интегралом от неограниченной на от-

резке ba; функции (или несобственным интегралом второго рода) называ-ется предел

b

a

b

a

dxxfdxxf0

lim . (6.4)

Если предел в правой части равенства (6.4) существует и конечен, то не-собственный интеграл называется сходящимся, в противном случае – расхо-дящимся.

Аналогичным образом определяется интеграл от функции xf , неогра-ниченной на отрезке ba; , при условии, что

xf

ax 0lim :

b

a

b

a

dxxfdxxf

0lim . (6.5)

Если функция xf неограниченна во внутренней точке с отрезка ba; ,

то несобственный интеграл от функции xf определяется следующим обра-зом:

b

с

с

a

b

a

dxxfdxxfdxxf . (6.6)

Несобственный интеграл, стоящий в левой части равенства (6.6), схо-дится, если сходятся оба интеграла, стоящие в правой части равенства.

Пример 6.4. Вычислить интеграл

2

0 1cos

sin

dxx

x.

Решение. Сделаем замену переменной.

duxdx

xdxdu

xu

dxx

sin

sin

1cos

1cos

sinx2

0

10

0

1

1

0

limu

du

u

du

u

du

lnlim1lnlnlim1lnlnlimlnlim

00010u .

То есть интеграл

2

0 1cos

sin

dxx

x расходится.

x u

2

1

0 0

Page 66: М.Н. Середина, Л.Ю. Шипик МАТЕМАТИКАачии.рф/files/2018-11-16-7b4836c8-02e7-466b-aa5c...2018/11/16  · функции нескольких переменных,

66

Вопросы для самопроверки

1. Что называется интегральной суммой функции )x(f на отрезке b;a ?

2. Дайте определение определенного интеграла. 3. Каков геометрический смысл определенного интеграла? 4. Напишите формулу Ньютона-Лейбница. 5. Как выглядит формула интегрирования по частям для определенного инте-

грала? 6. Запишите формулы для вычисления объемов тел вращения. 7. Дайте определение несобственного интеграла с бесконечным верхним пре-

делом.

Задания для аудиторной и самостоятельной работы

Вычислить следующие определенные интегралы:

347.

2

13

2 23 dx

xx . 348.

1

1

2 1 dxx . 349.

0

1

2dxe

x .

350. 4

0

4cos

xdx . 351. dxx

x

1

0231

. 352. 8

16

3 4sin

4cos

x

xdx.

353.

dxx

xe

1

2ln2. 354. dx

x

xtg

8

02 2cos

123

. 355.

3

42

1

dxx

xtg

.

356д. 3

2

3 124 dxxx . 357д. 2

1

22 dxxx . 358д.

1

3

dxex .

359д.

1

12

54 dxx . 360д. dxx

x

4

02 9

2. 361д.

1

022 1x

xdx.

362д.

2

0216 x

dx. 363д.

3

02cos

sin

x

dxx. 364д.

2

1 1ln3

e

xx

dx.

Используя метод интегрирования по частям, вычислить следующие определенные интегралы:

365. 1

0

xarctgxdx . 366. 2

0

cos

xdxx . 367. 1

0

dxxex .

Page 67: М.Н. Середина, Л.Ю. Шипик МАТЕМАТИКАачии.рф/files/2018-11-16-7b4836c8-02e7-466b-aa5c...2018/11/16  · функции нескольких переменных,

67

368. 6

4

2cos

x

xdx. 369д.

0

2

22 dxex

x

. 370д. 8

13 2

lndx

x

x.

Вычислить следующие несобственные интегралы (или установить их расходимость):

371.

14

x

dx. 372.

02 3

dxexx . 373.

1 43 x

dx.

374.

04

3

116x

dxx. 375.

2

4 4x

dx. 376.

4

03 12cos

2sin

x

dxx.

377д.

1 x

dx. 378д.

0

5dxe

x . 379д.

5

2 63x

dx.

Вычислить площадь фигур, ограниченных указанными линиями: 380. 2

xy , 223 xy . 381. 3xy , 1y , 0x .

382. 21 yx , 1 yx . 383. 12 xy , 0y , 1x , 4x .

384. 3xy , xy 212 , 0y . 385. x

ey , xy 1 , 1x , 0y .

386. xlny , 2ex , ось Ox . 387д. 2

xy , 02 yx .

388д. 25 xy , 32 xy . 389д. 224 yx , 02 xy .

390д. 2xy , 2 yx , 0x . 391д. 22 xxy , xxy 42 2 .

Вычислить объем тела, образованного вращением вокруг указанной оси координат фигуры, ограниченной заданными линиями: 392. 2

xy , 4y ; Oy . 393. 2xy , 2

yx ; Ox .

394. 2

12

xy , 1 xy ; Oy . 395. 3

xy , 1y , 0x ; Ox ; Oy .

396. 2xy , 6 yx , 0y ; 397д. xy 42 , 0x ; Oy .

Ox ; Oy .

398д. xy , xy 3 , 3x ; Ox . 399д. 22xy , 122 xy , 0y ;

Ox ; Oy .

Вычислить длину дуги следующих кривых:

400. 23

2 23

1 xy , 30;x . 401. xsinlny ,

3

2

2

;x .

402. xxy , 10;x . 403. 11 2 xlny , 43;x .

404. ,5,6sin2,6cos2 tztytx 405. .2;0,3,3

4 3 ttytx

Page 68: М.Н. Середина, Л.Ю. Шипик МАТЕМАТИКАачии.рф/files/2018-11-16-7b4836c8-02e7-466b-aa5c...2018/11/16  · функции нескольких переменных,

68

.12

;0

t

406д. xxy3

2 , 30;x . 407д. xcoslny ,

30

;x .

408д. .2

;4

,3sin,3cos

ttytx 409д. .6;0,2,

3

1 23 ttyttx

7 Понятие функции нескольких переменных

7.1 Частные производные функции нескольких переменных. Полный дифференциал. Градиент

Частной производной по x (по y ) от функции yxfz ; называется предел отношения частного приращения zx ( zy ) к приращению x ( y )

при стремлении x ( y ) к нулю. Частная производная по x от функции yxfz ; обозначается одним

из символов x

z

, xz , xf , x

f

.

Частная производная по y от функции yxfz ; обозначается одним

из символов y

z

, yz , yf , y

f

.

Так как zx вычисляется при неизменном y , то частной производной по x от функции yxfz ; называется производная по x , вычисленная в предположении, что y – постоянная.

Аналогично определяется частная производная по y .

Пример 7.1. Дана функция 12ln335 yeyxzx , найти

x

z

, y

z

.

Решение. Считая y постоянной, находим производную по x :

xeyx

x

z 334 35

.

Считая x постоянной, находим производную по y : 12

23 25

yyx

y

z.

Для функции любого числа переменных частные производные опреде-ляются так же, как и для функции двух переменных.

Пример 7.2. Дана функция 23 832sin2 yxyzzxuz , найти

x

u

,

y

u

, z

u

.

Page 69: М.Н. Середина, Л.Ю. Шипик МАТЕМАТИКАачии.рф/files/2018-11-16-7b4836c8-02e7-466b-aa5c...2018/11/16  · функции нескольких переменных,

69

Решение. Считая y и z постоянными величинами, находим частную

производную по x : zxzx

x

u32cos23 2

.

Считая x и z постоянными величинами, находим частную производную

по y : yzy

u162

.

Считая x и y постоянными величинами, находим частную производ-

ную по z : 3ln32sin23 z

xyxz

u.

Полный дифференциал функции yxfz ; обозначается через dz и

вычисляется по формуле

dyy

zdx

x

zdz

. (7.1)

Пример 7.3. Найти полный дифференциал dz функции yxyxyz 3cos2 4 .

Решение. Находим частные производные x

z

и y

z

:

yxyx

z

3sin32 ; yxyxy

z

3sin42 3 .

Применяя формулу (20), получим полный дифференциал dz :

dyyxyxdxyxydz 3sin423sin32 3 .

Градиентом функции yxfz ; называется вектор, координаты кото-рого равны частным производным функции в точке:

jy

zi

x

zMzgrad

MM

00

0 , (7.2)

или

00

;0MM y

z

x

zMzgrad . (7.3)

Для функции любого числа переменных градиент определяется так же, как и для функции двух переменных.

Градиент функции характеризует направление и величину максималь-ной скорости возрастания этой функции в точке.

Пример 7.4. Найти градиент функции 24 32 yezxy в точке

2;20 M .

Решение. Найдѐм частные производные x

z

и y

z

:

42 xy

eyx

z, yex

y

z xy 62 4 .

Page 70: М.Н. Середина, Л.Ю. Шипик МАТЕМАТИКАачии.рф/files/2018-11-16-7b4836c8-02e7-466b-aa5c...2018/11/16  · функции нескольких переменных,

70

Вычислим их значения в точке 2;20 M :

422 422

0

ex

z

M

, 82622 422

0

ey

z

M

.

Тогда jiMzgrad 840 .

Пример 7.5. Найти градиент функции yzxyzyxu 23 в точке 3;2;10M .

Решение. Найдѐм частные производные x

u

, y

u

и z

u

:

223 yzyxx

u

, zxyzxy

u

23 , yyxz

u

3 .

Вычислим их значения в точке 3;2;10M :

2223213 2

0

Mx

u, 4321231

0

My

u, 0221

z

u.

Тогда jiMzgrad 4220 .

Если частную производную x

z

y

z продифференцировать по x (по

y ), то есть найти

x

z

x

y

z

y, то получим частную производную

второго порядка от функции yxfz ; , взятую два раза по x (по y ). Эта

производная обозначается одним из символов 2

2

x

z

, xxz , xxf ,

2

2

x

f

2

2

2

2

,,,y

ffz

y

zyyyy .

Если частную производную x

z

y

z продифференцировать по y (по

x ), то есть найти

x

z

y

y

z

x, то получим частную производную

второго порядка от функции yxfz ; , взятую сначала по x (по y ), а за-

тем по y (по x ). Эта производная обозначается одним из символов yx

z

2

,

xyz , xyf , yx

f

2

xy

ffz

xy

zyxyx

22

,,, . Производные yx

z

2

и xy

z

2

называ-

ются смешанными. Справедлива следующая теорема: Если функция yxfz ; и смешанные частные производные определе-

Page 71: М.Н. Середина, Л.Ю. Шипик МАТЕМАТИКАачии.рф/files/2018-11-16-7b4836c8-02e7-466b-aa5c...2018/11/16  · функции нескольких переменных,

71

ны и непрерывны в некоторой точке yxP ; и ее окрестности, то в этой точке порядок вычисления смешанных производных не влияет на результат, то есть имеет место равенство

yx

z

2

=xy

z

2

. (7.4)

Пример 7.6. Найти частные производные второго порядка функции 123 523 yxyxz .

Решение. Находим частные производные x

z

и y

z

:

422 153 xyxx

z

; 22 3

yxy

z.

Дифференцируя x

z

по x , получаем 322

2

606 xxyx

z

.

Дифференцируя x

z

по y , получаем yxyx

z 22

6

.

Дифференцируя y

z

по y , получаем 32

2

2xy

z

.

Дифференцируя y

z

по x , получаем yxxy

z 22

6

.

Как видно, yx

z

2

и xy

z

2

совпадают.

Пример 7.7. Найти частные производные второго порядка функции

y

xz

38sin

.

Решение. Находим частные производные x

z

и y

z

:

y

x

yx

z 38cos

8

; y

x

y

x

y

z 38cos

382

.

Дифференцируя x

z

по x , получаем

y

x

yyy

x

yx

z 38sin

64838sin

822

2

.

Дифференцируя x

z

по y , получаем

22

2 3838sin

838cos

8

y

x

y

x

yy

x

yyx

z

Page 72: М.Н. Середина, Л.Ю. Шипик МАТЕМАТИКАачии.рф/files/2018-11-16-7b4836c8-02e7-466b-aa5c...2018/11/16  · функции нескольких переменных,

72

y

x

y

x

y

x

y

38sin

38838cos

832

.

Дифференцируя y

z

по y , получаем

2232

2 3838sin

3838cos

382

y

x

y

x

y

x

y

x

y

x

y

z

y

x

y

x

y

x

y

x 38sin

3838cos

3824

2

3

.

Дифференцируя y

z

по x , получаем

yy

x

yy

x

yxy

z 838sin

838cos

822

2

y

x

y

x

y

x

y

38sin

38838cos

832

.

Как видно, yx

z

2

и xy

z

2

совпадают.

7.2 Экстремум функции нескольких переменных

Точка 000 ; yxP из области определения функции yxfz ; называется точкой максимума (минимума), если существует такая -окрестность точ-ки 000 ; yxP , что для всех 000 ;; yxPyxP из этой окрестности выполня-ется неравенство yxfyxf ;; 00 yxfyxf ;; 00 .

Точки максимума и минимума функции называются точками экстре-мума, а значения функции в этих точках – экстремумами функции.

Если дифференцируемая функция yxfz ; имеет экстремум в точке 000 ; yxP , то ее частные производные в этой точке равны нулю, то есть

0

0

Px

z; 0

0

Py

z. (7.5)

Точки, в которых x

z

= 0 и

y

z

= 0, называются стационарными точ-

ками функции yxfz ; .

Для исследования функции yxfz ; на экстремум воспользуемся сле-дующей теоремой.

Пусть 000 ; yxP является стационарной точкой функции yxfz ; ,

имеющей непрерывные частные производные первого и второго порядков в этой точке. Составим выражение

Page 73: М.Н. Середина, Л.Ю. Шипик МАТЕМАТИКАачии.рф/files/2018-11-16-7b4836c8-02e7-466b-aa5c...2018/11/16  · функции нескольких переменных,

73

22

2

2

2

2

yx

z

y

z

x

z. (7.6)

Если 00 P , то в стационарной точке 0P нет экстремума. Если 00 P , то в точке 0P есть экстремум, причем максимум, если

0

0

2

2

Px

z , и минимум, если 0

0

2

2

Py

z.

Если 0P = 0, то требуется дополнительное исследование. Пример 7.8. Исследовать на экстремум функцию

2263 yxyxyxz .

Решение. 1. Находим частные производные x

z

и y

z

:

yxx

z

23 ; yxy

z26

.

Каждую из них приравниваем нулю и решаем систему:

.026

,023

yx

yx Находим 0x и 3y . Следовательно, 3;00P есть стацио-

нарная точка функции z . 2. Составляем выражение . Для этого находим частные производные

второго порядка:

22

2

x

z; 2

2

2

y

z; 1

2

yx

z.

Тогда 3122 2 .

Так как 030 P , то в точке 3;00P функция имеет экстремум. Так

как 02

0

2

2

Px

z, то в точке 3;00P функция имеет максимум.

99183;0max zz .

Вопросы для самопроверки

1. Как определяется функция нескольких переменных? 2. Что называется частной производной функции нескольких переменных? 3. Запишите формулу градиента функции двух переменных. 4. Каковы необходимые условия экстремума функции двух переменных? 5. Сформулируйте правило исследования функции двух переменных на экс-тремум.

Page 74: М.Н. Середина, Л.Ю. Шипик МАТЕМАТИКАачии.рф/files/2018-11-16-7b4836c8-02e7-466b-aa5c...2018/11/16  · функции нескольких переменных,

74

Задания для аудиторной и самостоятельной работы

Найти частные производные первого порядка следующих функций:

410. yxyxz 235 3 411. 2232 7245 xxyxyyxz .

412. 22yxlnz . 413.

yx

xyz

2

.

414. x

ysinz . 415д.

y

xarctgz .

416д. yxyarctgxz 5 . 417д. 23yxlnysinxz .

418. arctgxyeylnxuz 2 . 419д. yz

xeyzsinxyzxu 2 .

420. Показать, что 2

y

zy

x

zx , если 22

yxyxlnz .

421д. Показать, что zy

zy

x

zx

, если

yx

xyz

.

Найти полные дифференциалы следующих функций: 422. 3

yeysinxzx . 423. 492 xtgyz .

424. 2232 5yxyxz 425д. 345 yxz .

в точке 1;2P .

426д. ysinlnxyz23 . 427д. 43

yxz в точке 2;1 P .

Найти частные производные второго порядка следующих функций: 428. 32

xyylnxarctgxz . 429. ycosyxz 43 .

430. yxlnz 2 . 431. x

yarctgz

2 .

432. 57 xyez . 433д. 16sin35 yxz .

434д. yxexzy 24 . 435д. 2332 4572 xyxxyyxz .

436. Показать, что 092

2

2

2

x

z

y

z , если yx

e

yxsinz

3

3

.

Найти градиент функции в указанной точке:

437. 432 22 xxyyxz в точке 438. x

yarctgz

3 в точке .1;1P

.2;2M

439. 436ln 2 yxyz в точке 440. 12 xzyzxyu в точке

.1;1M .3;1;1 P

441д. 32 325 yxyxz в точке 442д. yyxz 172 в точке

.1;2 P .2;1M

Page 75: М.Н. Середина, Л.Ю. Шипик МАТЕМАТИКАачии.рф/files/2018-11-16-7b4836c8-02e7-466b-aa5c...2018/11/16  · функции нескольких переменных,

75

443д. 253 yxez в точке .5;3P 444д. yzzxyu 32 23 в точке

.1;2;1 M

Исследовать на экстремум следующие функции: 445. 2244 yxyxz . 446. 206922 yxyxyxz .

447. 9663 22 yxyxyxz . 448*. xyyxz 633 .

449д. 26423 22 yxyxyxz . 450д. 2263 yxyxyxz .

Примерный вариант практического рейтинга №2

Найти неопределенные интегралы:

1. xdxсtg53 ; 2. dxxx3756 ; 3. dx

x

x

16

232

; 4. xdxx 2sin .

5. Вычислить dxxx 3

2

2 243 .

6. Вычислить площадь фигуры, ограниченной линиями 12 xy , xy 1 .

7. Вычислить объем тела, образованного вращением вокруг оси Ox фигуры, ограниченной линиями 2

xy , xy 2 .

8. Найти xyz , если yyxezyx 922 323 2

.

9. Найти xxz , если .ln 232yyxxyz

8 Элементы дискретной математики

8.1 Элементы математической логики

Высказыванием называется повествовательное предложение, о котором в данной ситуации можно сказать, что оно истинно или ложно, но не то и другое одновременно.

Приведем примеры высказываний: «Волга впадает в Каспийское море». «Два больше трех».

Первое высказывание является истинным, а второе – ложным. Таким образом, высказывание обладает свойством представлять истину

или ложь, поэтому на высказывание можно смотреть как на величину, кото-рая может принимать только одно из двух значений: «истина», «ложь».

Поставим в соответствие высказыванию логическую переменную x, ко-торая принимает значение 1, если высказывание истинно, и 0, если высказы-вание ложно.

Page 76: М.Н. Середина, Л.Ю. Шипик МАТЕМАТИКАачии.рф/files/2018-11-16-7b4836c8-02e7-466b-aa5c...2018/11/16  · функции нескольких переменных,

76

Значение истинности составного высказывания определяется значения-ми истинности его компонент.

Высказывания будем обозначать прописными буквами латинского алфа-вита X, Y, Z … .

Составные высказывания будем получать из простых с помощью ло-гических операций: отрицание, конъюнкция, дизъюнкция, импликация, эквивалентность, которые осуществляются при помощи логических связок:

; ; ; ; .

Таблица 4 – Логические операции

Название Прочтение Обозначение

Отрицание не

Конъюнкция и

Дизъюнкция или

Импликация если…то

Эквивалентность тогда и только тогда, когда

При рассмотрении той или иной связки мы хотим знать, каким именно

образом истинность составного высказывания, порожденного этой связкой, зависит от истинности его компонент. Очень удобно изображать эту зависи-мость, пользуясь таблицами истинности, которые называются также интер-претациями логических операций. Каждой строке таблицы истинности вза-имно однозначно соответствует набор составляющих высказываний и соот-ветствующее значение составного высказывания. Наборы из нулей и единиц, соответствующих составляющим высказываниям, в каждой строке таблицы истинности имеют стандартное расположение, то есть расположены в лекси-ко-графическом порядке (порядке возрастания).

Пусть даны два произвольных высказывания X и Y. Отрицанием высказывания X называется высказывание Х , которое ис-

тинно, когда X ложно, и ложно, когда X истинно.

Таблица 5 – Таблица истинности для отрицания

X Х 0 1 1 0

Конъюнкцией двух высказываний X и Y называется высказывание YX , которое истинно только в том случае, когда X и Y оба истинны.

Page 77: М.Н. Середина, Л.Ю. Шипик МАТЕМАТИКАачии.рф/files/2018-11-16-7b4836c8-02e7-466b-aa5c...2018/11/16  · функции нескольких переменных,

77

Таблица 6 – Таблица истинности для конъюнкций

X Y YX 0 0 0 0 1 0 1 0 0 1 1 1

Дизъюнкцией двух высказываний X и Y называется высказывание YX , которое истинно, когда хотя бы одно из них истинно.

Таблица 7 – Таблица истинности для дизъюнкций

Х Y YX 0 0 0 0 1 1 1 0 1 1 1 1

Импликацией двух высказываний X и Y называется высказывание YX , которое ложно тогда и только тогда, когда X истинно, а Y ложно.

Таблица 8 – Таблица истинности для импликации

X Y YX 0 0 1 0 1 1 1 0 0 1 1 1

Эквивалентностью высказываний X и Y называется высказывание YX , которое истинно тогда и только тогда, когда X и Y оба истинны или

ложны. Таблица 9 – Таблица истинности для эквивалентности

X Y YX 0 0 1 0 1 0 1 0 0 1 1 1

Для образования составных высказываний наряду с единичным исполь-

зованием каждой основной связки можно пользоваться основными связками многократно, получая более сложные составные высказывания – аналогично

Page 78: М.Н. Середина, Л.Ю. Шипик МАТЕМАТИКАачии.рф/files/2018-11-16-7b4836c8-02e7-466b-aa5c...2018/11/16  · функции нескольких переменных,

78

тому, как с помощью основных арифметических операций образуются слож- ные алгебраические выражения.

Например, составными будут высказывания: .;; XYXXXYX

Каждое составное высказывание имеет свою таблицу истинности, кото-рая может быть построена стандартным образом.

Пример 8.1. Докажите тождественную истинность формулы YXX .

Решение. Составим таблицу истинности:

X Y X YX YXX

0 0 1 1 1 0 1 1 1 1 1 0 0 0 1 1 1 0 1 1

Последний столбец состоит из 1, следовательно, доказана тождественная истинность формулы.

Пример 8.2. Докажите эквивалентность ZXYXZYZXX .

Решение. Пусть ZYZXX 1 .

Составим таблицу истинности:

X Y Z ZX ZY ZXX ZYZXX

0 0 0 0 0 0 0 0 0 1 1 1 0 0 0 1 0 0 1 0 0 0 1 1 1 1 0 0 1 0 0 1 0 1 0 1 0 1 1 1 1 1 1 1 0 1 1 1 1 1 1 1 1 1 1 1

Пусть ZXYX 2 .

X Y Z YX ZX ZXYX

0 0 0 0 0 0 0 0 1 0 0 0 0 1 0 0 0 0 0 1 1 0 0 0 1 0 0 0 0 0 1 0 1 0 1 1 1 1 0 1 0 1 1 1 1 1 1 1

Page 79: М.Н. Середина, Л.Ю. Шипик МАТЕМАТИКАачии.рф/files/2018-11-16-7b4836c8-02e7-466b-aa5c...2018/11/16  · функции нескольких переменных,

79

Заметим, что таблицы истинности для 1 и 2 совпадают, следователь-но, эквивалентность доказана.

8.2 Элементы комбинаторного анализа

На практике часто встречаются задачи, где необходимо подсчитать чис-ло всех возможных способов размещения некоторых предметов конечного множества или число всех возможных способов выполнения определенного действия из конечного множества таких действий. Задачи такого типа назы-ваются комбинаторными, а методы их решения − методами комбинатор-

ного анализа. Перестановками называют комбинации, состоящие из одних и тех же

n различных элементов и отличающиеся только порядком их расположения. Число всех возможных перестановок

!nPn , (8.1)

где 1231! nnn и называется факториалом числа n (читается n-факториал). Причем: 244!3!4;63!2!3;221!2;1!1;1!0 .

Размещениями называют комбинации, составленные из n различных элементов по k элементов, которые отличаются либо составом элементов, либо их порядком. Число всех возможных размещений

!

!

k

nA

kn . (8.2)

Сочетаниями называют комбинации, составленные из n различных элементов по k элементов, которые отличаются хотя бы одним элементом. Число сочетаний

!!

!

knk

nС kn . (8.3)

Выше предполагалось, что все n элементов различны. Если же некото-рые элементы повторяются, то в этом случае комбинации с повторениями вычисляют по другим формулам.

Если среди n элементов есть 1n элементов одного вида, 2n элементов другого вида и так далее, то число перестановок с повторениями

...!!

!

21

nn

nмиповторениясPn , (8.4)

где nnn ...21 .

Число размещений с повторениями (среди k элементов сколько угод-но может содержаться одинаковых)

kkn nмиповторениясA . (8.5)

Пусть имеются предметы n видов и из них составляется набор, содер-жащий k элементов, то есть различными исходами будут всевозможные наборы длины k , отличающиеся составом, и при этом отдельные наборы мо-гут содержать повторяющиеся элементы. Такие наборы называют сочетани-

Page 80: М.Н. Середина, Л.Ю. Шипик МАТЕМАТИКАачии.рф/files/2018-11-16-7b4836c8-02e7-466b-aa5c...2018/11/16  · функции нескольких переменных,

80

ями с повторениями, а их общее число определяется формулой

!1!

!11

nk

knCмиповторениясС k

knkn . (8.6)

При решении задач комбинаторики используют следующие правила: Правило суммы. Если некоторый объект A может быть выбран из сово-

купности объектов m способами, а другой объект B может быть выбран n способами, то выбрать либо A , либо B можно mn способами.

Правило произведения. Если объект A можно выбрать из совокупности объектов m способами и после каждого такого выбора объект B можно вы-брать n способами, то пара объектов BA, в указанном порядке может быть

выбрана mn способами. Пример 8.3. Сколько трехзначных чисел можно составить из цифр 1, 2,

3, если каждая цифра входит в число только один раз. Решение. Количество трехзначных чисел, составленных из трех различ-

ных цифр – это число перестановок из трех элементов: 6!33 P .

Пример 8.4. Дано множество 5,3,1А . Найти число размещений из элементов множества A по два, когда а) элементы не повторяются; б) эле-менты могут повторяться.

Решение. а) Число размещений без повторений вычисляется по формуле

(8.2): 6!1

321

!23

!323

А .

б) Число размещений с повторениями вычисляется по формуле (8.5):

93223 миповторениясA .

Пример 8.5. Сколько наборов из 7 пирожных можно составить, если в продаже имеются 4 сорта?

Решение. Количество наборов – это число сочетаний с повторениями, вычисляемое по формуле (8.6):

120321!7

1098!7

!3!7

!10710

7174

CС .

Вопросы для самопроверки

1. Что такое высказывание? 2. Дайте определения отрицания, конъюнкции, дизъюнкции, импликации, эк-вивалентности. 3. Запишите таблицу истинности для импликации. 4. Что такое перестановка, размещение, сочетание? 5. Запишите формулу для вычисления сочетаний.

Задания для аудиторной и самостоятельной работы

Построить таблицы истинности для следующих формул:

Page 81: М.Н. Середина, Л.Ю. Шипик МАТЕМАТИКАачии.рф/files/2018-11-16-7b4836c8-02e7-466b-aa5c...2018/11/16  · функции нескольких переменных,

81

451. )( ZXYX . 452. )()( ZYYX .

453д. )( YXZ . 454д. ZXYYX )()( .

455. С помощью таблиц истинности проверить будут ли эквивалентны сле-дующие формулы: )( ZYX и )()( ZXYX .

456. Сколько различных трехзначных чисел можно составить из цифр 0, 1, 2, 3, 4, 5, если: а) цифры не повторяются; б) цифры могут повторяться? 457. Сколькими способами могут быть распределены три первых места среди 20 соревнующихся? 458. В цветочном киоске 6 видов цветов. Сколькими разными способами можно составить букет, содержащий 3 цветка? 459. Сколько разных чисел (трехзначных) можно получить из цифр 1, 7, 9? 460. Студенты сдают 4 экзамена, из них 2 экзамена связаны с биологией. Сколькими способами можно распределить экзамены, но так, чтобы экзаме-ны, связанные с биологией, следовали один за другим? 461д. Если подбросить одновременно три игральные кости, то сколько име-ется различных возможных комбинаций выброшенных очков? 462д. Сколько имеется четырехзначных чисел, все цифры у которых различ-ны? 463д. Сколькими способами можно выбрать 3 яблока из корзины с 12 ябло-ками? 464д. Сколькими способами можно составить трехцветный полосатый флаг (три вертикальные полосы), если имеется материал 6 различных цветов? 465д. В комнате имеется 6 стульев. Сколькими способами можно разместить на них: а) 6 гостей; б) 3 гостей?

9 Теория вероятностей

9.1 Классическое, статистическое и геометрическое

определения вероятности. Теоремы сложения и умножения

Каждому событию A соответствует определенное число ,AP называ-емое его вероятностью и удовлетворяющее условию

.10 AP (9.1)

Классическое определение вероятности. Вероятностью события A называется отношение числа исходов опыта, благоприятных событию A , к числу всех исходов опыта.

,n

mAP (9.2)

где m – число благоприятных событию A исходов, n – число всех возможных исходов.

Относительной частотой события A называется отношение числа испытаний, в которых событие появилось, к общему числу произведенных испытаний.

Page 82: М.Н. Середина, Л.Ю. Шипик МАТЕМАТИКАачии.рф/files/2018-11-16-7b4836c8-02e7-466b-aa5c...2018/11/16  · функции нескольких переменных,

82

,n

mAw (9.3)

где m – число появлений события A , n – общее число произведенных испытаний.

Геометрической вероятностью события называется отношение меры области, благоприятной появлению события, к мере всей области.

.Gмераgмера

AP (9.4)

Суммой А + В двух событий А и В называется событие, состоящее в по-явлении события А или события В, или обоих этих событий.

Вероятность суммы двух несовместных событий определяется по формуле

.BPAPBAP (9.5)

Если события nAAA ,...,, 21 попарно несовместны, то ....... 2121 nn APAPAPAAAP (9.6)

Произведением А ∙ В двух событий А и В называется событие, состоя-щее в совместном появлении этих событий.

Вероятность произведения двух событий определяется по формуле ,/ ABPAPBAP (9.7)

где ABP / – условная вероятность события В.

В случае n событий имеем ..../.../... 12112121 nnn AAAAPAAPAPAAAP (9.8)

Если события независимы, то ,BPAPBAP (9.9)

....... 2121 nn APAPAPAAAP (9.10)

Противоположным событию A называют событие A , состоящее в не появлении события A .

Вероятность появления хотя бы одного из независимых событий проще находить методом от противного, используя формулу

.1 APAP (9.11)

Пример 9.1. В партии из 8 изделий имеется 5 первого сорта, остальные – второго. Найти вероятность того, что среди взятых наудачу четырех изде-лий три первого сорта.

Решение. Обозначим: событие А – среди взятых наудачу четырех изде-лий три первого сорта.

Применим формулу (9.2). Общее число всех возможных исходов опыта равно числу способов, которыми можно извлечь 4 изделия из 8 (число соче-таний из 8 элементов по 4), то есть

.704321

8765

!4!4

!848

Cn

Число исходов, благоприятствующих событию А определим по правилу произведения:

Page 83: М.Н. Середина, Л.Ю. Шипик МАТЕМАТИКАачии.рф/files/2018-11-16-7b4836c8-02e7-466b-aa5c...2018/11/16  · функции нескольких переменных,

83

.301

3

21

54

!2!1

!3

!2!3

!513

35

CCm

Таким образом, искомая вероятность

.7

3

70

30)( AP

Пример 9.2. В урне содержится 6 белых, 4 черных и 5 красных шаров. Из урны трижды вынимают наудачу по одному шару, не возвращая их в ур-ну. Найти вероятность того, что первый шар будет черный, второй – белый, а третий – красный.

Решение. Рассмотрим следующие события: А – первый из взятых шаров черный; В – второй шар белый; С – третий шар красный. Искомая вероятность определяется по формуле

.// ABCPABPAPABCP

Согласно условию имеем

,15

4AP ,

7

3

14

6/ ABP .

13

5/ ABCP

В результате получаем

.91

4

13

5

7

3

15

4ABCP

Пример 9.3. Имеется 2 ящика, содержащих по 15 деталей. В первом ящике 12 стандартных деталей, а во втором 9. Из каждого ящика наудачу вы-нимают по одной детали. Найти вероятность того, что хотя бы одна деталь будет стандартная.

Решение. Обозначим: событие A − хотя бы одна деталь будет стандарт-ная, тогда событие A − обе детали будут нестандартные. Воспользуемся формулой .1 APAP

Рассмотрим события: 1A − из первого ящика вынут нестандартную деталь; 2A − из второго ящика вынут нестандартную деталь. Тогда

21 APAPAP , где 5

1

15

31 AP

5

2

15

62 AP .

В результате, согласно формуле (9.11), получаем

.25

23

25

21

5

2

5

11 AP

9.2 Формула полной вероятности. Формулы Бейеса

Пусть событие А может произойти только вместе с одним из событий

nHHH ...,,, 21 , образующих полную группу несовместных событий (гипо-тез). Тогда вероятность события А вычисляется по формуле

Page 84: М.Н. Середина, Л.Ю. Шипик МАТЕМАТИКАачии.рф/files/2018-11-16-7b4836c8-02e7-466b-aa5c...2018/11/16  · функции нескольких переменных,

84

n

iii HAPHPAP

1

),/()()( (9.12)

которая называется формулой полной вероятности. Пусть в условиях рассуждения, относящегося к формуле полной вероят-

ности, произведено одно испытание, в результате которого произошло собы-тие A . Тогда вероятности гипотез могут быть переоценены по формулам Бейеса

n

iii

kkk

HAPHP

HAPHPAHP

1

/

// , ....,,1 nk (9.13)

Пример 9.4. Имеется 2 мешка семян второго сорта и 4 мешка семян первого сорта. Всхожесть семян второго сорта равна 75%, а первого сорта – 90%. Найти вероятность того, что взятое наудачу семя взойдет.

Решение. Рассмотрим гипотезы: Н1 – семя взято из мешка с семенами второго сорта; Н2 – семя взято из мешка с семенами первого сорта. Обозначим через А событие – взятое семя взойдет. Согласно условию имеем

,3

1

6

21 HP .

3

2

6

4)( 2 HP

Находим условные вероятности события А: ;75,0/ 1 HAP .9,0/ 2 HAP

Используя формулу (9.12), получаем

.,,,,,H/APHPH/APHPAP 85060250903

2750

3

12211

Пример 9.5. Большая популяция людей разбита на две группы одинако-

вой численности. Диета одной группы отличалась высоким содержанием не-насыщенных жиров, а диета контрольной группы была богата насыщенными жирами. После 10 лет пребывания на этих диетах возникновение сердечно-сосудистых заболеваний составило в этих группах соответственно 31% и 48%. Случайно выбранный из популяции человек имеет сердечно-сосудистое заболевание. Какова вероятность того, что этот человек принадлежит к кон-трольной группе?

Решение. Рассмотрим гипотезы: Н1 – человек придерживался специальной диеты; Н2 – человек принадлежал к контрольной группе. Обозначим через А событие – случайно выбранный из популяции человек имеет сердечно-сосудистое заболевание. Имеем

5,021 HPHP ;

31,0/ 1 HAP ; 48,0/ 2 HAP .

Используя формулу (9.13), получаем

61,0

48,05,038,05,0

38,05,0

//

//

2211

222

HAPHPHAPHP

HAPHPAHP .

Page 85: М.Н. Середина, Л.Ю. Шипик МАТЕМАТИКАачии.рф/files/2018-11-16-7b4836c8-02e7-466b-aa5c...2018/11/16  · функции нескольких переменных,

85

9.3 Формула Бернулли. Теоремы Лапласа. Формула Пуассона

Если производится n независимых испытаний, в каждом из которых не-

которое событие А появится с одной и той же вероятностью p и не появится с вероятностью ,1 pq то вероятность того, что событие А появится ровно

k раз вычисляется по формуле Бернулли:

,)( knkknn qpCkP

(9.14)

где .)!(!

!

knk

nC

kn

Наивероятнейшее число появления события A в n испытаниях определяется как целое число, удовлетворяющее неравенству

pnpqnp . (9.15)

Локальная теорема Лапласа. Если вероятность p события А в каждом из n независимых испытаний постоянна и отлична от нуля и единицы, то ве-роятность )(kPn того, что событие А появится k раз в n испытаниях при-ближенно равна (тем точнее, чем больше n )

),(1

)( xnpq

kPn (9.16)

где ,2

1)( 2

2x

ex

.npq

npkx

В таблице приложения 1 помещены значения функции )(x , соответ-ствующие положительным значениям аргумента х. При этом необходимо помнить, что ).()( xx

Интегральная теорема Лапласа. Если вероятность p наступления события А в каждом испытании постоянна и отлична от нуля и единицы, то вероятность ),( 21 kkPn того, что событие А появится в n испытаниях от 1k

до 2k раз (при этом n достаточно велико), приближенно равна

1221, xxkkPn , (9.17)

где x t

dtex

0

2

2

2

1

функция Лапласа (см. таблицу приложения 2);

;11

npq

npkx

.2

2npq

npkx

Необходимо помнить, что xx ; для 5х можно принять .5,0)( х

Формула Пуассона. Если вероятность p события А в каждом из n не-зависимых испытаний постоянна и отлична от нуля и единицы (при этом n − велико, а p − мала), то вероятность )(kPn того, что событие А появится k

Page 86: М.Н. Середина, Л.Ю. Шипик МАТЕМАТИКАачии.рф/files/2018-11-16-7b4836c8-02e7-466b-aa5c...2018/11/16  · функции нескольких переменных,

86

раз в n испытаниях приближенно равна

,!

ek

kPk

n (9.18)

где np 10npq .

Пример 9.6. Игральную кость бросают 5 раз. Найти вероятность того, что четверка выпадет три раза.

Решение. Вероятность появления четверки при одном броске игральной

кости постоянна и равна .6

1p Следовательно, вероятность не появления

четверки также постоянна и равна .6

51 pq

Искомая вероятность по формуле Бернулли равна

.3888

125

36

25

216

1

2

54

6

5

6

1

!2!3

!5)3(

23233

55

qpCP

Пример 9.7. Вероятность поражения мишени стрелком при одном вы-стреле равна 0,85. Найти вероятность того, что при 300 выстрелах стрелок поразит мишень не менее 245 раз и не более 262 раз.

Решение. Воспользуемся интегральной теоремой Лапласа: .260,245 12300 ххP

Вычислим значения аргументов 1х и 2х :

;62,125,38

10

15,085,0300

85,03002451

x

.13,125,38

7

15,085,0300

85,03002622

x

Таким образом, имеем 4474,03708,062,113,162,113,1262,245300P

.8182,0

Подсчет клеток под микроскопом. Предположим, что n клеток опре-деленного типа распределены случайным образом по площади предметного стекла, которое разбито квадратной решеткой на 900 ( 3030 ) равных участ-ков. Вероятность того, что конкретная клетка лежит в данном участке решет-

ки, есть 900

1p . Процесс размещения n клеток на предметном стекле мож-

но рассматривать как n повторных испытаний, где «успех» определяется как попадание клетки в конкретный участок решетки. Если n велико, то для вы-числения вероятности того, что конкретный участок решетки содержит k клеток, можно воспользоваться формулой Пуассона:

...,2,1,0,

!900!

900

kk

ene

k

npkP

nk

npk

n

Page 87: М.Н. Середина, Л.Ю. Шипик МАТЕМАТИКАачии.рф/files/2018-11-16-7b4836c8-02e7-466b-aa5c...2018/11/16  · функции нескольких переменных,

87

Величина kPn даѐт долю тех из 900 участков, в которых содержится по k

клеток. Общее число участков, содержащих по k клеток, .900 kPn Напри-

мер, ожидается, что в среднем 900900

n

e

участков не содержат ни одной клетки.

Это дает нам метод оценки общего числа имеющихся клеток путем определения числа тех участков квадратной решетки, которые не содержат этих клеток. Если, например, мы видим, что клеток нет в 75 участках квад-ратной решетки, то

90090075

n

e

.

Отсюда

224012ln90075

900ln900

n .

Основное допущение, сделанное нами, состоит в том, что n клеток рас-пределены по площади стекла случайно. Если это допущение справедливо, то формула Пуассона дает весьма эффективное средство оценки числа клеток на стекле.

9.4 Случайные величины

Случайной величиной называют переменную величину, которая в зави-симости от исхода испытания случайно принимает одно значение из множе-ства возможных значений.

Случайная величина, которая может принимать отдельно стоящие изо-лированные значения, называется дискретной.

Пусть Х – дискретная случайная величина (ДСВ), ,1х ,2х …, nх еѐ возможные значения, ii xXPp вероятность появления значения ix , где i=1, 2, …, n, тогда таблица

ix 1х 2х … nх

ip 1p 2p … np

называется рядом распределения или законом распределения.

Математическим ожиданием дискретной случайной величины назы-вается сумма произведений всех ее возможных значений на их вероятности:

n

iii pxXM

1

. (9.19)

Свойства ХМ :

1. ССМ , где constC .

2. XMCXCM .

3. YMXMYXM .

Page 88: М.Н. Середина, Л.Ю. Шипик МАТЕМАТИКАачии.рф/files/2018-11-16-7b4836c8-02e7-466b-aa5c...2018/11/16  · функции нескольких переменных,

88

4. YMXMYXM , если X и Y независимые случайные величины.

Дисперсией называют математическое ожидание квадрата отклонения случайной величины от еѐ математического ожидания

2XMXMXD . (9.20)

Для вычисления дисперсии пользуются формулой

XMXMXD22 . (9.21)

Свойства дисперсии: 1. 0CD , где constC .

2. XDCCXD 2 .

3. YDXDYXD , если X и Y независимые. Средним квадратическим отклонением случайной величины Х назы-

вают квадратный корень из дисперсии

XDХ . (9.22)

Дискретная случайная величина Х распределена по биномиальному закону, если еѐ возможные значения 0, 1, 2, …, п, а соответствующие вероят-ности вычисляются по формуле Бернулли:

knkkn qpCkXP

,

где k = 0, 1, …, п. Биноминальное распределение определяется двумя параметрами п и р. Для биноминального распределения:

npXM ; ;npqXD .npqX (9.23)

Дискретная случайная величина Х распределена по закону Пуассона, если еѐ возможные значения 0, 1, 2, …, а соответствующие вероятности вы-числяются по формуле Пуассона:

ek

kXPk

!, (9.24)

где k = 0, 1, 2, …; np .

Распределение Пуассона определяется одним параметром . Для распреде-ления Пуассона:

.;; XXDXM (9.25)

Функцией распределения случайной величины Х называют функцию xF , определяющую вероятность того, что случайная величина Х примет

значение, меньшее х, то есть xXPxF . (9.26)

Свойства функции )(xF :

1. 10 xF .

2. ,F 0 1F .

3. Функция xF неубывающая. 4. aFbFbXaP .

Page 89: М.Н. Середина, Л.Ю. Шипик МАТЕМАТИКАачии.рф/files/2018-11-16-7b4836c8-02e7-466b-aa5c...2018/11/16  · функции нескольких переменных,

89

Пример 9.8. Устройство состоит из трех независимо работающих эле- ментов. Вероятность отказа каждого элемента в одном опыте равна 0,1. Со-ставить закон распределения числа отказавших элементов в одном опыте.

Решение. Обозначим X − число отказавших элементов в одном опыте. Возможные значения этой случайной величины: 01 x , 12 x , 23 x , 34 x .

Отказы элементов независимы один от другого, вероятности отказа каж-дого элемента равны между собой, поэтому случайная величина X имеет биномиальное распределение. Воспользуемся формулой

knkkn qpCkXP

, где 3,2,1,0;9,01,01;1,0;3 kqpn .

Тогда

729,09,01,010 3003003

qpCXP ;

243,09,01,0!2!1

!31 211311

3

qpCXP ;

027,09,01,0!1!2

!32 122322

3

qpCXP ;

001,09,01,013 0333333

qpCXP .

Контроль: 1001,0027,0243,0729,0 .

Напишем искомый биномиальный закон распределения X :

ix 0 1 2 3

ip 0,729 0,243 0,027 0,001

Пример 9.9. Найти числовые характеристики случайной величины Х, которая задана следующим законом распределения:

хi − 3 − 2 1 4

рi 0,1 0,3 0,4 0,2

Решение. Найдем математическое ожидание XM :

.3,02,044,013,021,03 XM

Найдем математическое ожидание 2XM :

.7,52,34,02,19,02,044,013,021,03 22222 XM

Тогда дисперсия равна

,61,509,07,522 XMXMXD

а среднее квадратическое отклонение

.369,261,5 XD

Случайная величина Х называется непрерывной, если еѐ функция рас-пределения xF непрерывна при всех Rx .

Плотностью распределения непрерывной случайной величины назы-вают функцию

xFxf . (9.27)

Свойства функции xf :

1. 0xf при всех Rx .

Page 90: М.Н. Середина, Л.Ю. Шипик МАТЕМАТИКАачии.рф/files/2018-11-16-7b4836c8-02e7-466b-aa5c...2018/11/16  · функции нескольких переменных,

90

2.

1dxxf .

3. b

a

dxxfbXaP .

Функцию распределения xF можно найти через плотность распределения по формуле

x

dxxfxF . (9.28)

Числовые характеристики непрерывной случайной величины вычисля-ются по формулам:

dxxfxXM ; (9.29)

dxxfXMxXD

2 или

XMdxxfxXD

22 . (9.30)

XDX .

Равномерным называют распределение вероятностей непрерывной слу-чайной величины X , если на интервале ba; , которому принадлежат все возможные значения X , плотность распределения сохраняет постоянное значение, а именно:

ab

xf

1

; (9.31)

вне этого интервала 0xf .

Равномерное распределение определяется двумя параметрами a и b . Математическое ожидание, дисперсия и среднее квадратическое откло-

нение равномерного распределения соответственно равны:

2

baXM

;

12

2ab

XD

; 32

abX

. (9.32)

Показательным (экспоненциальным) называют распределение вероят-ностей непрерывной случайной величины X , которое описывается плотно-стью распределения

,0

,00

xприe

xприxf x

(9.33)

где − постоянная положительная величина. Функция распределения показательного распределения

.01

,00

xприe

xприxF x (9.34)

Вероятность попадания в интервал ba; непрерывной случайной вели-чины X , распределенной по показательному закону,

Page 91: М.Н. Середина, Л.Ю. Шипик МАТЕМАТИКАачии.рф/files/2018-11-16-7b4836c8-02e7-466b-aa5c...2018/11/16  · функции нескольких переменных,

91

baeebXaP

. (9.35)

Показательное распределение определяется одним параметром . Математическое ожидание, дисперсия и среднее квадратическое откло-

нение показательного распределения соответственно равны:

1

XM ; 2

1

XD ;

1

X . (9.36)

Непрерывная случайная величина Х называется распределѐнной по нормальному закону с параметрами a и , если еѐ плотность распределения имеет вид:

.2

1 2

2

2

ax

exf

(9.37)

Вероятностный смысл параметров a и :

,аХМ ,2XD X .

Вероятность попадания нормально распределѐнной случайной величины Х в интервал ; вычисляется по формуле

aaXP , (9.38)

где x t

dtex

0

2

2

2

1

функция Лапласа.

Вероятность того, что нормально распределѐнная случайная величина Х отклонится от математического ожидания аХМ по абсолютной вели-чине не более чем на 0 находят по формуле

2|| aXP . (9.39)

Пример 9.10. Случайная величина Х задана интегральной функцией:

.31

;516

1

6

1

;10

)(

хпри

xприх

хпри

xF

Найти: а) вероятность того, что в результате испытания Х примет значение, принадлежащее интервалу (0; 2);

б) плотность распределения вероятностей случайной величины Х; в) математическое ожидание случайной величины Х; г) дисперсию случайной величины Х.

Решение. а) Вероятность того, что случайная величина Х примет зна-чение, заключенное в интервале (а; b), определяется по формуле

.aFbFbXaP

Получаем

Page 92: М.Н. Середина, Л.Ю. Шипик МАТЕМАТИКАачии.рф/files/2018-11-16-7b4836c8-02e7-466b-aa5c...2018/11/16  · функции нескольких переменных,

92

.3

1

6

1

6

1

3

10220 FFXP

б) Плотность распределения вероятностей случайной величины Х найдем по формуле (9.27). Получаем

.30

;516

1

;10

)(

хпри

xпри

хпри

xf

в) Математическое ожидание случайной величины Х находим по формуле (9.29). Имеем

.212

1

12

25

126

15

1

25

1

x

xdxXM

г) Дисперсию случайной величины Х находим по формуле (9.30). Получаем

.3418

1

18

1254

182

6

15

1

322

5

1

x

dxxXD

Пример 9.11. Цена деления шкалы измерительного прибора равна 0,1.

Показания прибора округляют до ближайшего целого деления. Найти веро-ятность того, что при отсчете будет сделана ошибка, превышающая 0,02.

Решение. Ошибку округления отсчета можно рассматривать как слу-чайную величину X , которая распределена равномерно в интервале между двумя соседними целыми делениями. Плотность распределения равномерно-го распределения

ab

xf

1

,

где ab − длина интервала, в котором заключены возможные значения X ; вне этого интервала 0xf . В нашем случае длина интервала, в котором за-ключены возможные значения X , равна 0,1, поэтому

101,0

1xf .

Ошибка отсчета превысит 0,02, если она будет заключена в интервале 08,0;02,0 . По 3-му свойству функции плотности распределения получим

6,02,08,0101008,002,0

08,0

02,0

08,0

02,0

xdxXP .

Пример 9.12. Случайная величина Х распределена по нормальному за-кону. Математическое ожидание и среднее квадратическое отклонение этой величины соответственно равны 45 и 6. Найти вероятность того, что Х при-

Page 93: М.Н. Середина, Л.Ю. Шипик МАТЕМАТИКАачии.рф/files/2018-11-16-7b4836c8-02e7-466b-aa5c...2018/11/16  · функции нескольких переменных,

93

мет значение, принадлежащее интервалу (42; 54). Решение. Воспользуемся формулой (9.38). По условию ,45а ,6

,42 .54 Следовательно,

1915,04332,0)5,0()5,1(6

4542

6

455454;42P

.6247,0

Вопросы для самопроверки 1. Сформулируйте классическое определение вероятности события. 2. Сформулируйте теоремы сложения и умножения вероятностей. 3. Дайте определение полной группы событий. 4. Какие случайные величины называются дискретными (непрерывными)? 5. Каковы свойства математического ожидания и дисперсии случайной вели-

чины? 6. Запишите формулы для вычисления дисперсии дискретной (непрерывной)

случайной величины. 7. Как связаны функция распределения и плотность распределения вероятно-

стей случайной величины? 8. Как найти вероятность попадания случайной величины в заданный интер-

вал? 9. Какая случайная величина называется нормально распределенной? 10. Что такое «правило 3 »?

Задания для аудиторной и самостоятельной работы

466. В урне 9 белых и 6 синих шаров. Какова вероятность, что наудачу выну-тый шар окажется белым?

467. Из урны, в которой имеется 5 белых, 7 синих и 3 красных шара, наудачу берут один шар. Чему равна вероятность, что вынутый шар окажется: а) синим; б) не белым; в) белым; г) чѐрным?

468. Брошены две игральные кости. Какова вероятность того, что сумма вы-павших очков: а) равна 7; б) не более 5-и; в) кратна 4?

469. Какова вероятность, что наудачу выбранное двузначное число состоит из одинаковых цифр?

470. Куб, все грани которого окрашены, распилен на 64 кубика одинакового размера, которые потом тщательно перемешаны. Найти вероятность то-го, что наудачу извлечѐнный кубик будет иметь окрашенных граней: а) одну; б) две; в) три.

471. В первом ящике находятся шары с номерами от 1 до 8, а во втором – с номерами от 9 до 15. Из каждого ящика вынули по одному шару. Какова вероятность того, что сумма номеров вынутых шаров: а) не менее 21; б) равна 20; в) не более 13?

472. В ящике 20 деталей, 6 из них окрашены. Наудачу извлекают 4 детали.

Page 94: М.Н. Середина, Л.Ю. Шипик МАТЕМАТИКАачии.рф/files/2018-11-16-7b4836c8-02e7-466b-aa5c...2018/11/16  · функции нескольких переменных,

94

Какова вероятность того, что три из них окрашены, а одна нет? 473. В экзаменационный билет входит 3 вопроса программы, насчитывающей

40 вопросов. Студент не знает 15 вопросов программы. Какова вероят-ность, что он ответит на все вопросы взятого наудачу билета?

474. Из коробки, содержащей 6 пронумерованных жетонов, вынимают один за другим все находящиеся в ней жетоны и укладывают рядом. Найти вероятность того, что номера вынутых жетонов будут идти по порядку (по убыванию).

475. Из 450 деталей, изготовленных на автоматическом станке, оказалось 12 нестандартных. Определите частоту появления нестандартных дета-лей.

476. Какова статистическая вероятность прорастания семян, если из 1000 по-саженных семян проросло 745?

477. Внутрь круга радиуса 6 наудачу брошена точка. Найти вероятность того, что точка окажется внутри вписанного в круг правильного треугольника. (Предполагается, что вероятность попадания в треугольник пропорцио-нальна площади треугольника и не зависит от его расположения внутри круга).

478. Абонент ждѐт телефонного вызова в течение одного часа. Какова веро-ятность того, что вызов произойдѐт в последние 15 минут.

479. В партии из 10 деталей 7 стандартных. Найти вероятность того, что сре-ди наудачу извлечѐнных 2-х деталей есть хотя бы одна стандартная.

480. В урне 9 белых шаров и 3 чѐрных шара. Найти вероятность того, что среди 4 отобранных наудачу шаров окажется не более одного чѐрного шара.

481. Три орудия ведут огонь по цели. Вероятность попадания в цель из пер-вого орудия равна 0,5; из второго – 0,7; из третьего – 0,9. Каждое орудие стреляет один раз. Найти вероятность того, что два орудия попадут в цель.

482. Из колоды карт (36 штук) наудачу взяты три карты. Найти вероятность того, что все они окажутся черной масти.

483. В одном ящике – 12 деталей, из них 10 стандартных, в другом – 20 дета-лей, из них 15 стандартных. Взяли по одной детали из каждого ящика. Найти вероятность того, что хотя бы одна из них окажется стандартной.

484. Для сигнализации об аварии установлены три независимо работающих устройства. Вероятность того, что при аварии сработает первое устрой-ство, равна 0,7; второе – 0,95; третье – 0,8. Найти вероятность того, что при аварии сработает: а) только одно устройство; б) все три устройства.

485. Брошены 3 игральные кости. Найти вероятность того, что на каждой из костей выпадет 3 очка.

486. Брошены две игральные кости. Чему равна вероятность того, что хотя бы на одной из костей выпадет 4 очка?

487. Партия семян, состоящая из 8 мешков, подлежит приѐмке, если при про-верке наугад выбранных двух мешков окажется, что содержащиеся в них семена удовлетворяют стандарту. Найти вероятность приѐмки партии,

Page 95: М.Н. Середина, Л.Ю. Шипик МАТЕМАТИКАачии.рф/files/2018-11-16-7b4836c8-02e7-466b-aa5c...2018/11/16  · функции нескольких переменных,

95

содержащей в 2 мешках нестандартные семена. 488. В первом ящике 3 белых, 2 красных и 5 синих шара; во втором ящике 7

белых, 6 красных, 2 синих. Из каждого ящика вынули по шару. Какова вероятность того, что среди вынутых шаров нет синих?

489. Вероятность выигрыша по лотерейному билету равна 0,2. Приобретено три билета. Какова вероятность выиграть хотя бы по одному из них? 490. Вероятность получить высокие дивиденды по акциям на первом пред-

приятии – 0,15; на втором – 0,25; на третьем – 0,4. Определить вероят-ность того, что акционер, имеющий акции всех предприятий, получит высокие дивиденды: а) на всех предприятиях; б) хотя бы на одном предприятии.

491. В урне 8 красных, 3 зелѐных и 5 чѐрных шаров. Определить вероятность того, что взятые наудачу два шара будут: а) одного цвета; б) разных цветов.

492. Первый студент из 30 вопросов программы выучил 22, второй – 18. Каждому студенту задают по одному вопросу (вопросы могут повто-ряться). Определить вероятность того, что: а) оба студента правильно ответят на вопрос; б) хотя бы один ответит верно; в) правильно ответит только первый студент.

493д. В лототроне 45 шаров с номерами от 1 до 45. Какова вероятность собы-тий: А – номер первого разыгранного шара кратен 4; В – номер первого шара нечѐтный?

494д. В ящике 16 пронумерованных шаров с номерами от 1 до 16. Вынули один шар. Какова вероятность того, что номер вынутого шара: а) не пре-вышает 11; б) более 8?

495д. Брошены две игральные кости. Найти вероятность того, что: а) сумма очков не превосходит 5; б) произведение числа очков не превосходит 8.

496д. В урне 12 шаров: 5 белых и 7 чѐрных. Вынули два шара. Какова веро-ятность того, что оба шара белые?

497д. В партии готовой продукции из 8 изделий имеется 5 изделий высшего качества. Наудачу отбираются 4 изделий. Какова вероятность того, что три из них будут высшего качества?

498д. Произведя 150 выстрелов, стрелок попал в цель 126 раз. Найти частоту попадания данного стрелка в цель.

499д. В круге радиусом 12 см помещѐн меньший круг радиуса 8 см. Найти вероятность того, что наудачу брошенная в большой круг точка попадѐт также в меньший круг.

500д. В лотерее 30 билетов, из них 5 выигрышных. Найти вероятность того, что из двух взятых билетов хотя бы один выигрышный.

501д. Два станка работают независимо друг от друга. Вероятность того, что первый станок в течение смены выйдет из строя, равна 0,15, для второ-го – 0,3. Найти вероятность того, что в течение смены только один ста-нок выйдет из строя.

502д. Бросают две игральные кости. Найти вероятность того, что на первой кости появится нечетное число очков, а на второй – четыре очка.

Page 96: М.Н. Середина, Л.Ю. Шипик МАТЕМАТИКАачии.рф/files/2018-11-16-7b4836c8-02e7-466b-aa5c...2018/11/16  · функции нескольких переменных,

96

503д. В первом ящике 8 белых и 10 чѐрных шаров. Во втором ящике 12 бе-лых и 4 чѐрных шара. Из каждого ящика наудачу вынули по одному ша-ру. Какова вероятность того, что шары разного цвета?

504д. В магазин вошли 3 покупателя. Вероятность покупки для каждого из них равна 0,4. Найти вероятность того, что: а) два из них совершат по-купки; б) все три совершат покупки; в) ни один не совершит покупки; г) хотя бы один купит товар.

505д. На базу поступило 60 ящиков овощей, из них 48 первого сорта, осталь-ные второго. Наудачу для проверки берут два ящика. Какова вероят-ность того, что: а) оба содержат овощи первого сорта; б) разного сорта; в) одного сорта?

506. В урну, содержащую 2 шара, опущен черный шар. После чего из неѐ наудачу извлечѐн 1 шар. Найти вероятность того, что извлечѐнный шар окажется белым, если равновозможны все возможные предположения о первоначальном составе шаров (по цвету: белый, чѐрный).

507. В группе 20 студентов, из них 2 отличника, 6 хорошистов, 3 двоечника, остальные учатся удовлетворительно. Вероятность сдачи экзамена от-личником 0,98; хорошистом – 0,8; двоечником – 0,2; троечником – 0,65. Какова вероятность того, что произвольно взятый студент сдаст экза-мен?

508. На конвейер сбрасывают одинаковые детали, произведѐнные двумя ав-томатами. Производительность первого автомата вдвое больше произво-дительности второго автомата. Первый автомат производит 70% деталей отличного качества, второй – 90%. Какова вероятность того, что взятая наудачу деталь будет отличного качества?

509. При исследовании жирности молока коров всѐ стадо было разбито на три группы. В первой группе оказалось 70%, во второй 20% и в треть-ей 10% всех коров. Вероятность того, что молоко, полученное от от-дельной коровы, имеет не менее 4% жирности, для каждой группы соот-ветственно равна 0,6; 0,35 и 0,1. Определить вероятность того, что для взятой наудачу коровы жирность молока составит не менее 4%.

510. Имеются две урны. В первой – 5 красных шаров и 3 чѐрных, во второй – 2 красных и 6 чѐрных. Из первой урны переложили во вторую 1 шар, за-тем, перемешав шары, из второй урны извлекли 1 шар. Найти вероят-ность того, что шар, извлечѐнный из второй урны, окажется красным.

511. Для посева заготовлены семена 4 сортов пшеницы. Причѐм 40% всех семян 1-го сорта, 20% – 2-го сорта, 30% – 3-го сорта и 10% – 4-го сорта. Вероятность того, что из зерна вырастет колос, содержащий не менее 40 зѐрен, для первого сорта − 0,5; для второго − 0,3; для третьего − 0,2; для четвѐртого – 0,1. Найти вероятность того, что наудачу взятое зерно даст колос, содержащий не менее 40 зѐрен.

512. На ѐлочный базар поступают ѐлки с трѐх лесхозов, причѐм 1-й лесхоз поставил 40% ѐлок, 2-й – 35%, 3-й – 25%. Среди ѐлок 1-го лесхоза 12% голубых, 2-го – 18%, 3-го – 24%. Куплена одна ѐлка. Она оказалась голу-

бой. Какова вероятность, что она поставлена 2-м лесхозом?

Page 97: М.Н. Середина, Л.Ю. Шипик МАТЕМАТИКАачии.рф/files/2018-11-16-7b4836c8-02e7-466b-aa5c...2018/11/16  · функции нескольких переменных,

97

513. В комнату с мышеловкой поместили 8 белых и 12 серых мышей. Веро-ятность того, что в мышеловку попадется белая мышь, равна 0,7, серая – 0,9. Мышь попалась в мышеловку. Какова вероятность того, что в мы-шеловке белая мышь?

514д. В группе учащихся из 25 человек имеется 16 юношей, а остальные де-вушки. Половина юношей и треть девушек живут в общежитии. Найти вероятность того, что случайно выбранный учащийся группы живѐт в общежитии.

515д. Сборщик получил 4 коробки деталей, изготовленных заводом № 1, и 2 коробки деталей, изготовленных заводом № 2. Вероятность того, что деталь завода № 1 стандартна, равна 0,9, а завода № 2 – 0,75. Сборщик наудачу извлѐк деталь из наудачу взятой коробки. Найти вероятность то-го, что извлечѐнная деталь стандартна.

516д. Перед посевом 80% всех семян было обработано ядохимикатами. Веро-ятность поражения вредителями для растений из обработанных семян равна 0,08; для растений из необработанных семян – 0,45. Какова веро-ятность того, что растение, выращенное из взятого наудачу семени, бу-дет поражено вредителями?

517д. Стрелковое отделение получило 10 винтовок, из которых 8 пристре-лянных, 2 нет. Вероятность попадания в цель из пристрелянной винтов-ки равна 0,7; а из не пристрелянной − 0,4. Какова вероятность того, что стрелок из наудачу взятой винтовки попадѐт в цель при одном выстреле?

518д. Два колбасных цеха производят одинаковые колбасные изделия, кото-рые поступают в продажу. Производительность первого цеха вдвое больше второго. Первый цех производит 75% колбасных изделий отлич-ного качества, а второй 90%. Наудачу взятое колбасное изделие оказа-лось отличного качества. Найти вероятность того, что это изделие про-изведено вторым колбасным цехом.

519д. В канцелярии работают два секретаря, которые обрабатывают 40 и 60% исходящих документов за одно и то же время. Вероятность неверной ад-ресации документов секретарями равны соответственно 0,02 и 0,04. Найти вероятность того, что один из документов, оказавшийся неверно адресованным, отправлен первым секретарѐм.

520. Монету бросают 5 раз. Найти вероятность того, что «герб» выпадет: а) два раза; б) не менее двух раз.

521. Произведено 4 независимых испытания, в каждом из которых вероят-ность появления события А равна 0,8. Найти вероятность того, что собы-тие А появится не менее трѐх раз.

522. Вероятность того, что в течение данного времени ѐлочная гирлянда пе-регорит, равна 1/9. Найти вероятность того, что в течение данного вре-мени из 4 ѐлочных гирлянд 2 перегорят.

523. В хлопке число длинных волокон составляет 80%. Найти вероятность того, что среди взятых наудачу 6 волокон длинных окажется менее двух.

524. Всхожесть семян данного сорта растений составляет 82%. Найти наиве- роятнейшее число всхожих семян в партии из 15 семян.

Page 98: М.Н. Середина, Л.Ю. Шипик МАТЕМАТИКАачии.рф/files/2018-11-16-7b4836c8-02e7-466b-aa5c...2018/11/16  · функции нескольких переменных,

98

525. Вероятность поражения мишени при одном выстреле равна 0,8. Найти вероятность того, что при 100 выстрелах мишень будет поражена 75 раз.

526. Вероятность положительного результата при химическом анализе равна 0,8. Найти вероятность получения 86 положительных исходов при 100 анализах. 527. Вероятность появления события А в каждом из 300 независимых испы-

таний равна 0,75. Найти вероятность того, что в этих испытаниях собы-тие А появится не менее 210 раз и не более 225 раз.

528. Всхожесть зерна, хранящегося на складе, равна 80%. Какова вероятность того, что среди 100 зѐрен взойдет от 68 до 90 штук.

529. Магазин получил 3000 бутылок минеральной воды. Вероятность того, что при перевозке бутылка окажется разбитой, равна 0,002. Найти веро-ятность того, что магазин получит разбитых бутылок: а) ровно 3; б) хотя бы одну.

530. Вероятность выживания бактерий после радиоактивного облучения рав-на 0,004. Найти вероятность того, что после облучения из 500 бактерий останется не более 2-х бактерий.

531д. Орудие сделало 6 выстрелов по цели. Вероятность попадания по цели при каждом выстреле равна 2/3. Найти вероятность 5 попаданий.

532д. Всхожесть семян данного растения составляет 90%. Найти вероятность того, что из 5 посеянных семян взойдѐт не более 3.

533д. Вероятность поражения куста виноградника равна 0,15. Найти вероят-ность того, что среди 400 обследованных кустов винограда будет 360 неповреждѐнных.

534д. Найти вероятность того, что среди наудачу взятых 200 веточек роз 104 веточки окажутся красными розами, если в общей корзине красных – 60%.

535д. Баскетболист забрасывает мяч в корзину в среднем в 8 случаях из 10. Найти вероятность того, что будет не менее 112 и не более 118 попада-ний, если выполнено 150 бросков.

536д. Вероятность «сбоя» в работе телефонной станции при каждом вызове равна 0,006. Поступило 500 вызовов. Определить вероятность 5 «сбоев».

537д. Семена пшеницы содержат 0,2% сорняков. Найти вероятность того, что в 1000 семян будет 3 сорняка.

538. Игральная кость брошена 3 раза. Написать закон распределения вероят-ностей числа появления тройки.

539. В урне 4 белых и 12 чѐрных шаров. Вынули два шара. Случайная вели-чина Х – число вынутых белых шаров. Построить ряд распределения случайной величины Х.

540. Найти математическое ожидание случайной величины Х, зная закон еѐ распределения:

хi 7 5 2

рi 0,4 0,1 0,5

Page 99: М.Н. Середина, Л.Ю. Шипик МАТЕМАТИКАачии.рф/files/2018-11-16-7b4836c8-02e7-466b-aa5c...2018/11/16  · функции нескольких переменных,

99

541. Вероятность отказа детали за время испытания на надѐжность равна 0,15. Найти математическое ожидание числа отказавших деталей, если испытанию будут подвергнуты 20 деталей.

542. Случайная величина задана рядом распределения. С какой вероятностью случайная величина примет значение, равное единице? Вычислить ее

математическое ожидание и среднее квадратическое отклонение. хi -1 1 2 3 pi 0,1 p2 0,3 0,2

543. Найти математическое ожидание и дисперсию числа лотерейных биле-тов, на которые выпадут выигрыши, если приобретено 10 билетов, при-чѐм вероятность выигрыша по одному билету равна 0,1.

544. Проводятся три независимых измерения исследуемого образца. Вероят-ность допустить ошибку в каждом измерении равна 0,01. Найти число-вые характеристики случайной величины Х – числа ошибок, допущен-ных в измерениях.

545. Две независимые случайные величины Х и Y заданы своими законами распределения: xi -2 1 2 4 yi 2 6 pi 0,2 0,4 0,3 0,1 pi 0,7 0,3

Найти математическое ожидание и дисперсию случайной величины YXZ 23 .

546. Найти дисперсию случайной величины XYZ 53 , если 4)( XD , a

3)( YD .

547. Случайная величина задана функцией распределения:

.11

;10

;00

)( 3

хприxприx

xпри

xF

Найти: а) плотность распределения )(xf ; б) математическое ожидание; в) дисперсию.

548. Случайная величина Х задана интегральной функцией:

.31

,3219

8

,20

)(3

хпри

xприх

хпри

xF

Найти: а) дифференциальную функцию; б) вероятность попадания слу-чайной величины в интервал (2,5; 3); в) математическое ожидание, дис-персию и среднее квадратическое отклонение случайной величины Х.

549. Задана плотность распределения непрерывной случайной величины Х:

Page 100: М.Н. Середина, Л.Ю. Шипик МАТЕМАТИКАачии.рф/files/2018-11-16-7b4836c8-02e7-466b-aa5c...2018/11/16  · функции нескольких переменных,

100

.20

,215,0

,10

)(

xприxприx

xприxf

Найти функцию распределения )(xF .

550. Дана плотность распределения непрерывной случайной величины Х:

.20

,21)1(9

2

,10

)(

xпри

xприx

xпри

xf

Найти: а) функцию распределения )(xF ; б) математическое ожидание; в) вероятность попадания СВ Х на отрезок [1; 2].

551. Степан ожидает телефонный звонок между 18.00 и 19.00. Время ожида-ния звонка есть непрерывная случайная величина Х, имеющая равномер-ное распределение на отрезке [18; 19]. Найти вероятность того, что зво-нок поступит в промежутке от 18 часов 17 минут до 18 часов 29 минут.

552. Найти математическое ожидание, дисперсию и среднее квадратическое отклонение случайной величины Х, распределѐнной равномерно в ин-тервале (-2; 6).

553. Цена деления шкалы измерительного прибора равна 0,2. Показания при-бора округляются до ближайшего целого деления. Считая, что ошибки измерения распределены равномерно, найти вероятность того, что при отсчѐте будет сделана ошибка, меньшая 0,04.

554. Время Т выхода из строя радиостанции подчинено показательному зако- ну распределения с плотностью

.00

,025,0)(

25,0

tприtприe

tft

Найти: а) математическое ожидание и дисперсию случайной величи-ны Т; б) вероятность того, что радиостанция сохранит работоспособ-ность от 1 до 4 часов работы.

555. Случайная величина Х распределена по показательному закону с пара-метром λ = 0,6. Найти дифференциальную и интегральную функции распределения, а также вероятность попадания значений случайной ве-личины Х в интервал (0,5; 5).

556. Случайная величина подчинена закону Пуассона с математическим ожиданием 3 . Найти вероятность того, что данная СВ примет поло-жительное значение.

557. Случайная величина Х распределена нормально. Математическое ожи-дание и среднее квадратическое отклонение этой величины соответ-ственно равны 5 и 2,5. Найти вероятность того, что в результате испыта-ния Х примет значение, заключѐнное в интервале (4; 7).

558. Случайные отклонения размера детали от номинала распределены нор-

Page 101: М.Н. Середина, Л.Ю. Шипик МАТЕМАТИКАачии.рф/files/2018-11-16-7b4836c8-02e7-466b-aa5c...2018/11/16  · функции нескольких переменных,

101

мально. Математическое ожидание размера детали равно 160 мм, сред-нее квадратическое отклонение равно 0,4. Стандартными считаются де-тали, размер которых заключѐн между 159,4 мм и 160,6 мм. Найти про-цент годных деталей.

559. Урожайность озимой пшеницы по совокупности участков распределяет-ся по нормальному закону с параметрами: а=50 ц/га, =10 ц/га. Опреде-лить: а) какой процент участков будет иметь урожайность свыше 40 ц/га;

б) процент участков с урожайностью от 45 до 60 ц/га. 560. Количество зерна, собранного с каждой делянки опытного поля, есть

нормально распределѐнная случайная величина Х, имеющая математи-ческое ожидание m=60 кг и среднее квадратическое отклонение =1,5 кг. Найти симметричный относительно m интервал, в котором с вероятностью 0,9906 будет заключена величина Х.

561д. Составить закон распределения вероятностей числа появлений собы-тия А в трѐх независимых испытаниях, если вероятность появления этого события в каждом испытании постоянна и равна 0,7.

562д. Два стрелка произвели по одному выстрелу. Вероятность попадания в мишень первым стрелком равна 0,4; вторым – 0,8. Составить закон рас-пределения числа попаданий в мишень.

563д. Случайная величина Х задана законом распределения: хi -2 1 4 рi 0,1 0,5 0,4

Найти еѐ математическое ожидание. 564д. Найти числовые характеристики случайной величины, зная закон еѐ

распределения: хi -3 -2 0 2 рi 0,1 0,5 0,2 0,2

565д. Найти среднее квадратическое отклонение числа появления «5» при 10 бросках игральной кости.

566д. Две независимые случайные величины Х и Y заданы своими законами распределения:

xi -4 -1 0 2 yi 1 4 pi 0,2 0,4 0,3 0,1 pi 0,8 0,2

Найти математическое ожидание и дисперсию случайной величины YXZ 4 .

567д. Математическое ожидание и дисперсия случайной величины Х рав-ны −3 и 6 соответственно. Найти математическое ожидание и диспер-сию случайной величины: а) 57 X ; б) 112 X .

568д. Случайная величина Х задана функцией распределения:

.21

;211

;10

)(

xприxприx

xприхF

Page 102: М.Н. Середина, Л.Ю. Шипик МАТЕМАТИКАачии.рф/files/2018-11-16-7b4836c8-02e7-466b-aa5c...2018/11/16  · функции нескольких переменных,

102

Найти: а) вероятность попадания случайной величины Х в интервал (1,2; 2,5); б) плотность распределения; в) математическое ожидание случай-ной величины Х; г) дисперсию случайной величины Х.

569д. Случайная величина Х задана интегральной функцией:

.31

,318

1

8

,10

)(2

хпри

xприх

хпри

xF

Найти: а) дифференциальную функцию случайной величины Х; б) мате-матическое ожидание, дисперсию, среднее квадратическое отклонение случайной величины Х; в) вероятность попадания случайной величины в интервал (1; 2).

570д. Дана плотность распределения непрерывной случайной величины Х:

.10

,112

1

,10

)(

xпри

xпри

xпри

xf

Найти: а) функцию распределения )(xF ; б) математическое ожидание;

в) вероятность попадания СВ Х на отрезок

2

1;

2

1.

571д. Все значения равномерно распределѐнной СВ Х лежат на отрезке [2; 8]. Найти вероятность попадания СВ Х в промежуток (3; 5).

572д. Найти математическое ожидание, дисперсию и среднее квадратическое отклонение СВ Х, распределѐнной равномерно в интервале (2; 10).

573д. Непрерывная случайная величина Х распределена по показательному закону, заданному плотностью распределения

.00

,004,0)(

04,0

хприхприe

xfх

Найти вероятность того, что в результате испытания Х попадѐт в интер-вал (1; 2).

574д. Найти математическое ожидание, дисперсию и среднее квадратическое отклонение показательного распределения, заданного плотностью веро-ятности )0(8)( 8

xexfx .

575д. Для исследования продуктивности определѐнной породы домашней птицы измеряют диаметр яиц. Наибольший поперечный диаметр яиц представляет собой случайную величину, распределѐнную по нормаль-ному закону со средним значением 5 см и средним квадратическим от-клонением 0,3 см. Найти вероятность того, что: а) диаметр взятого наудачу яйца будет заключѐн в границах от 4,7 до 6,2 см; б) отклонение диаметра от среднего не превзойдѐт по абсолютной величине 0,6 см.

Page 103: М.Н. Середина, Л.Ю. Шипик МАТЕМАТИКАачии.рф/files/2018-11-16-7b4836c8-02e7-466b-aa5c...2018/11/16  · функции нескольких переменных,

103

576д. Урожайность овощей по участкам является нормально распределѐнной случайной величиной с математическим ожиданием 300 ц/га и средним квадратическим отклонением 30 ц/га. С вероятностью 0,9544 определить симметричные относительно математического ожидания границы, в ко-торых будет находиться средняя урожайность овощей на участках.

10 Элементы математической статистики

10.1 Статистическое распределение выборки

Одной из задач математической статистики является указание способов

сбора и группировки статистических сведений, полученных в результате наблюдений или экспериментов. На практике сплошное исследование (каж-дого объекта из интересующей нас совокупности) проводят крайне редко. Обычно из всей совокупности случайно отбирают ограниченное число объ-ектов и подвергают их исследованию. При этом выборкой называют сово-купность случайно отобранных объектов, генеральной совокупностью назы-вают совокупность объектов, из которых производится выборка. Объѐмом совокупности называют число объектов этой совокупности.

Пусть для изучения количественного (дискретного или непрерывного) признака X из генеральной совокупности извлечена выборка объѐма n . Причѐм значение 1x исследуемого признака наблюдалось 1n раз, 2x − 2n раз, …, kx − kn раз. Наблюдавшиеся значения ix признака X называют вари-антами; in − частотами (сумма всех частот равна объѐму выборки n );

n

nw i

i − относительными частотами (сумма всех относительных частот

равна единице). Последовательность вариант, записанных в возрастающем порядке, называют вариационным рядом.

Статистическим распределением выборки называют перечень вари-ант ix вариационного ряда и соответствующих им частот in или относитель-ных частот iw . Статистическое распределение выборки можно задать также в виде последовательности интервалов и соответствующих им частот (в каче-стве частоты интервала принимают сумму частот вариант, попавших в этот интервал).

Пример 10.1. Выборка задана в виде распределения частот: ix 4 6 10 14

in 1 3 5 1

Найти распределение относительных частот. Решение. Найдем объѐм выборки: 101531 n .

Page 104: М.Н. Середина, Л.Ю. Шипик МАТЕМАТИКАачии.рф/files/2018-11-16-7b4836c8-02e7-466b-aa5c...2018/11/16  · функции нескольких переменных,

104

Найдем относительные частоты: 1,010

11 w ; 3,0

10

32 w ;

5,010

53 w ; 1,0

10

14 w .

Тогда искомое распределение относительных частот будет иметь вид:

ix 4 6 10 14

iw 0,1 0,3 0,5 0,1

Контроль: 11,05,03,01,0 .

10.2 Эмпирическая функция распределения

Эмпирическая функция распределения определяется по статистическому распределению выборки по аналогии с функцией распределения дискретной случайной величины.

Эмпирической функцией распределения (функцией распределения вы-борки) называют функцию xF

* , определяющую для каждого значения x

относительную частоту события xX :

n

nxF x* ,

где xn − число вариант, меньших x ; n − объѐм выборки. Эмпирическая функция распределения обладает следующими свойства-

ми: 1. Значения эмпирической функции принадлежат отрезку 1;0 .

2. xF* − неубывающая функция.

3. Если 1x − наименьшая варианта, а kx − наибольшая, то 0* xF при

1xx и 1* xF при kxx .

Эмпирическая функция служит для оценки теоретической функции рас-пределения.

Пример 10.2. Найти эмпирическую фукцию по данному распределению выборки:

ix 2 6 10

in 12 18 30

Решение. Найдем объѐм выборки: 60301812 n .

Наименьшая варианта 21 x , следовательно, 0* xF при 2x .

Значение 6x , а именно 21 x , наблюдалось 12 раз, следовательно,

2,060

12* xF при 62 x .

Значение 10x , а именно 21 x и 62 x , наблюдались 301812 раз,

Page 105: М.Н. Середина, Л.Ю. Шипик МАТЕМАТИКАачии.рф/files/2018-11-16-7b4836c8-02e7-466b-aa5c...2018/11/16  · функции нескольких переменных,

105

следовательно, 5,060

30* xF при 106 x .

Так как 103 x − наибольшая варианта, то 1* xF при 10x .

Таким образом, искомая эмпирическая функция распределения:

.101

,1065,0

,622,0

,20

*

xприxприxприxпри

xF

График этой функции изображѐн на рисунке 19.

Рисунок 19

Вопросы для самопроверки

1. Как вы понимаете понятия выборки, генеральной совокупности? 2. Что такое вариационный ряд? 3. Чему равна сумма частот выборки? Сумма относительных частот? 4. В каком виде задается статистическое распределение выборки? 5. Как определяется эмпирическая функция распределения?

Задания для аудиторной и самостоятельной работы

577. Выборка задана в виде распределения частот:

хi 1 4 5 8 13

ni 2 5 7 4 2

Найти распределение относительных частот.

Page 106: М.Н. Середина, Л.Ю. Шипик МАТЕМАТИКАачии.рф/files/2018-11-16-7b4836c8-02e7-466b-aa5c...2018/11/16  · функции нескольких переменных,

106

580д. Из генеральной совокупности извлечена выборка объѐма 70n :

хi 4 5 6 7 8

ni 11 13 19 n4 10

Найти 4n .

578. Выборка задана в виде распределения относительных частот:

хi 1,3 1,4 1,6 1.9 2,2 2,5

wi 0,08 0,21 w3 0,25 0,18 0,12

Найти w3.

579. Найти эмпирическую функцию по данному распределению выборки:

хi 1 3 7 9

ni 5 4 8 3

581д. По данному распределению выборки найти эмпирическую функцию и

построить еѐ график:

хi 3 5 6 8 11

ni 20 10 15 25 30

Примерный вариант контрольной работы № 2

1. В партии из 10 деталей 8 стандартных. Найти вероятность того, что среди наудачу извлеченных 2-х деталей есть хотя бы одна стандартная. 2. Вероятность получить высокие дивиденды по акциям на первом предприя-тии – 0,15; на втором – 0,25; на третьем – 0,4. Определить вероятность того, что акционер, имеющий акции всех предприятий, получит высокие дивиден-ды только на одном предприятии. 3. В телестудии 5 телекамер. Вероятность того, что телекамера в данный мо-мент включена (для каждой из них), равна 0,6. Найти вероятность того, что в данный момент включено более трех камер.

Page 107: М.Н. Середина, Л.Ю. Шипик МАТЕМАТИКАачии.рф/files/2018-11-16-7b4836c8-02e7-466b-aa5c...2018/11/16  · функции нескольких переменных,

107

4. Дан закон распределения случайной величины X.

X –3 –1 0 0,5 2 P 0,4 0,2 0,2 0,1 0,1

Найти математическое ожидание и дисперсию случайной величины X. 5. Случайная величина X задана функцией распределения:

.31

,3219

8

,20

)(3

хпри

xприх

хпри

xF

Найти: 1) плотность распределения; 2) математическое ожидание случайной величины X. 6. Урожайность овощей по участкам является нормально распределенной случайной величиной с математическим ожиданием 300 ц/га и средним квад-ратическим отклонением 30 ц/га. Найти процент участков, для которых уро-жайность овощей не ниже 260 ц/га. 7. Выборка задана в виде таблицы распределения частот:

ix 2 3 4 8 11

in 12 9 6 10 13

Построить таблицу распределения относительных частот. 8. Из генеральной совокупности извлечена выборка объема n =50:

ix 3 8 9 12

in 12 8 5 4n

Тогда 4n равна?

Page 108: М.Н. Середина, Л.Ю. Шипик МАТЕМАТИКАачии.рф/files/2018-11-16-7b4836c8-02e7-466b-aa5c...2018/11/16  · функции нескольких переменных,

108

Ответы

1. а) ;6,4,2,1 б) ;8,7,6,5,4,3,2,1,0,1,4 в) ;7,5,3,0 г) ;10,8,6,2,1

д) .4 2. .1;7 3. а) ;10,6,3,1 б) ;2,0,1,2,3 в) .15,10,8,6

4. а) ;5,4,3 б) .7,6 5д. .3,1 6д. .7,6,5,4,3,2 7д. а) ;4;5

б) Ø; в) .)2;2[ 8д. а) ;2;11 б) .]1;7( 9. .3;5,2 10. .8;8

11д. .3;2 12д. .3

12;0

13. .12;2 14. .;2 15д. .8;4

16д. .;6 17. .1;0;1;1 18. .11;;;5,0

19д. .;7;;3 20д. .2;0;2;2 21. .2;3 22. .32;5,0

23д. .9;0 24д. .1;2 25. 9; 1; −9; 2,25. 26. 9; 7. 27. −8; 1; 13;

−1; −1. 28д. −4; 4; 7

2198 ; −11,68. 29д. 12; 3,5; 5; 10,5. 30. 0. 31. ∞.

32. 3

4. 33.

8

15. 34.

4

5. 35.

2

1 . 36. 0. 37. . 38. −3.

39. 5

2 . 40.

2

1. 41. −1. 42. 0. 43.

3

1. 44.

4

1. 45.

24

1.

46. 8

3. 47.

9

2. 48.

2

1. 49.

8

3 . 50.

3

2. 51. 0. 52. .

53. 4

1. 54.

4

3. 55. 3. 56.

6

1. 57.

2

1. 58. . 59. 0.

60. . 61. 0. 62. . 63. 3

2. 64. −1. 65. −3. 66.

3

1.

67. 6

3 . 68.

9

2 . 69. 2 . 70. 2. 71. 0. 72д. 3. 73д.

17

7.

74д. −3. 75д. . 76д. 4

3. 77д.

4

1 . 78д. 0. 79д.

4

5 .

80д. 2

1. 81д. 3 . 82д.

4

1. 83д. −3. 84д. 5,0 . 85д.

3

4 .

86д. 4. 87д. 9

2. 88д.

5

8 . 89д.

16

5. 90д. 0. 91д.

4

1. 92д. .

93д. 4

1. 94д.

3

1 . 95д. 0. 96д.

2

3. 97д.

3

1. 98д.

3

3.

99д. 0. 100д. . 101д. 2

. 102д. −2. 103д. 0. 182. −8100.

183д. 16. 186. ;87 xy .067 yx 187. ;0123 yx

.03432 yx 188. ;5418 xy .0318 yx 189д. ;012 yx

.032 yx 190д. ;0174 yx .0174 yx 191. Убывает на 1; и на ;1 , возрастает на 1;1 . 192. Убывает на 1; ,

Page 109: М.Н. Середина, Л.Ю. Шипик МАТЕМАТИКАачии.рф/files/2018-11-16-7b4836c8-02e7-466b-aa5c...2018/11/16  · функции нескольких переменных,

109

возрастает на ;1 . 193. Возрастает на 1; , убывает на ;1 .

194. Убывает на 1 ; и на ;1 , возрастает на 11; . 195д. Воз-растает на 1 ; и на ;1 , убывает на 11; . 196д. Возрастает на 1 ; и на ;1 , убывает на 01; и на 10; . 197. miny

285 y , 41max yy . 198. 4

12min yy ,

4

12max yy .

199. 12

171min yy ,

4

373min yy , 20max yy . 200. miny

eey

11

. 201.

4

13

4

11max

yy . 202. 01min yy ,

167max yy . 203. 8

273min yy . 204. Экстремумов нет.

205д. 11min yy . 206д. e

yy1

1min , e

yy1

1max .

207д. 123min yy , 41max yy . 208д. 00min yy .

209. 13наимy , 3наибy . 210. 3

4наимy , 1наибy . 211. 15наимy ,

85наибy . 212. 4

4

наимy , 4

4 наибy . 213д. 43наимy ,

23наибy . 214д. 2наимy , 66наибy . 215д. 6наимy , 125,16наибy .

216*.

2

1. 217

*. 50. 218д*

. У второго лекарства максимальная реакция

выше. 219. Вогнута на 5; , выпукла на ;5 , точка перегиба 5;5 . 220. Выпукла на 4; , вогнута на ;4 , точка перегиба 20;4 . 221. Выпукла на 1; и на ;1 , вогнута на 1;1 , точек пе-региба нет. 222. Вогнута на 2; и на ;0 , выпукла на 0;2 ,

точка перегиба 02; . 223. Выпукла на 2; , вогнута на ;2 ,

точка перегиба

2

2;2

e. 224. Вогнута на ;0 , точек перегиба нет.

225д. Вогнута на ; , точек перегиба нет. 226д. Выпукла на 0; , вогнута на ;0 , точек перегиба нет. 227д. Вогнута на 0; ,

выпукла на ;0 , точка перегиба 0;0 . 228д. Вогнута на 1; , вы-пукла на ;1 , точка перегиба 0;1 . 229. 0x , 1y . 230. 0x ,

xy . 231. xy 4 . 232. 3x . 233. 2x , 2x , 0y .

234. 0y при x . 235д. 1x , 1 xy . 236д. 1x , 1x .

237д. 3x , 5y . 238д. 0y . 239д. 1x , 1x xy 2 .

240д. 3x , 1y . 241. ;yD . Асимптот нет. Функция возрас-тает на 1; и на ;3 , убывает на 31; , 03min yy , 81max yy .

Кривая выпукла на 2; , вогнута на ;2 , точка перегиба 4;2 .

Page 110: М.Н. Середина, Л.Ю. Шипик МАТЕМАТИКАачии.рф/files/2018-11-16-7b4836c8-02e7-466b-aa5c...2018/11/16  · функции нескольких переменных,

110

242. ;yD . Асимптота 0y . Функция убывает на 4; и на

;4 , возрастает на 4;4 , 8

14min yy ,

8

14max yy . Кривая

выпукла на 34; и на 34;0 , вогнута на 0;34 и на ;34 ;

точки перегиба

16

3;34 , 0;0 ,

16

3;34 . 243. yD

;00; . Асимптоты 0x , xy . Функция возрастает на 0; и на ;2 , убывает на 2;0 , 32min yy . Кривая вогнута на 0; и на ;0 , точек перегиба нет. 244. ;22;22; yD .

Асимптоты 2x , 2x , xy . Функция возрастает на 32; и на ;32 , убывает на 2;32 , на 2;2 и на 32;2

3332min yy , 3332max yy . Кривая выпукла на 2; и на 2;0 , вогнута на 0;2 и на ;2 , точка перегиба 0;0 .

245. ;22; yD . Асимптоты 2x , 2 xy . Функция возраста-ет на 0; и на ;4 , убывает на 2;0 и на 4;2 , 84min yy ,

00max yy . Кривая выпукла на 2; , вогнута на ;2 , точек пере-гиба нет. 246. ;33; yD . Асимптоты 3x , 1y .

Функция возрастает на 3; и на ;3 , убывает на 3;3 ,

03min yy . Кривая вогнута на 3; и на 6;3 , выпукла на ;6 ,

точка перегиба

9

1;6 . 247д. ;00; yD . Асимптота 0x .

Функция убывает на 0; и на

2

1;0 , возрастает на

;

2

1, miny

32

1

y . Кривая вогнута на

3 4

1; и на ;0 , выпукла на

0;4

13

, точка перегиба

0;4

13

. 248д. ;00; yD .

Асимптоты 0x , 3y . Функция возрастает на 0; и на ;0 . Кривая вогнута на 0; , выпукла на ;0 , точек перегиба нет. 249д. ;22; yD . Асимптоты 2x , 0y . Функция воз-растает на 4; и на ;0 , убывает на 2;4 и на 0;2 ,

20min yy , 24max yy . Кривая выпукла на 2; , вогнута на ;2 , точек перегиба нет. 250д. ;yD . Асимптота 1y .

Функция возрастает на 1; и на ;1 , убывает на 1;1 ,

01min yy , 21max yy . Кривая вогнута на 3; и на 3;0 ,

Page 111: М.Н. Середина, Л.Ю. Шипик МАТЕМАТИКАачии.рф/files/2018-11-16-7b4836c8-02e7-466b-aa5c...2018/11/16  · функции нескольких переменных,

111

выпукла на 0;3 и на ;3 ; точки перегиба

2

31;3 , 1;0 ,

2

31;3 . 251. Cxxx 423 . 252. C

xx

x

34

2

2

. 253. 2

5 2x

Cx ln2 . 254. Cxx

x 23

4 234 . 255.

C

x

9

7 9

. 256. 8sin x

C . 257. Cx 52ln2

1. 258. Cx 65sinln

5

1. 259.

2ln5

25x

Cxx

3

2. 260. Cx

x 4sin

4

1

2

3 2

. 261. Cx

tg

2

13ln

3

1 2

.

262. Cex 32

2

1. 263. Cx 52cos

6

1 3 . 264. Cx 433

2 2 .

265. Cx 45ln10

3 2 . 266. Cxx 53ln 2 . 267.

Cx

7

cos4 7

.

268. Cx 112ln6

7 3 . 269. Cx 222

3. 270.

C

x

54

3563

.

271. Cextg 13

3

1. 272.

C

x

12

ln23 6

. 273. Cx

x

2

2ln

4

1.

274. Cx

xx

7

7ln

72

274 . 275. C

x

x

13

13ln

2

1. 276. x5ln

5

1

Cx 35 2 . 277. Cx

arctg 5

2

5

2. 278. C

x

2

3arcsin

3

4.

279. Cx

x

32

32ln

218

13

3

. 280. Cx

3

arcsin2

. 281. Cx

arctg 5

2cos

52

1.

282. Cx

xx

32

32ln

12

794ln

8

3 2 . 283. Cx

x 2

3arcsin94

9

2 2 .

284. Cxx

73ln6

5

7

3arcsin

7

3 2 . 285. Cxtg

tgx 5

35

.

286. Cex 232

3

1. 287. C

x

3ln2

3 2sin1

. 288. 6

241ln

8

1 32 xarctg

x

C . 289. Cxx

2

ln3

5

25

. 290д. Cxxx 54 2 . 291д. x6

Cxx 7ln . 292д. Cx

x

6

2

2

. 293д. Cxxx

234

3

5

4

3.

Page 112: М.Н. Середина, Л.Ю. Шипик МАТЕМАТИКАачии.рф/files/2018-11-16-7b4836c8-02e7-466b-aa5c...2018/11/16  · функции нескольких переменных,

112

294д. Cex 2 . 295д. Cx 8sinln . 296д. Cx 13cos

3

1.

297д. Cx 54 . 298д. Cx 1sinln2

1 2 . 299д. Cx 6ln2

1 2 .

300д. Cx 73 2 . 301д. Cx

arctgx 55

3. 302д. C

x

2arcsin22 .

303д. Cx

x

53

53ln

152

1. 304д. Cxx 8ln

2

1 42 . 305д. 3

arcsinx

e

.C 306д. Cxxx 323ln7 22 .

307д. 232ln2

1x C

x

x

23

23ln

3

2. 308д. C

x

1cos .

309д. Cex 532sinln

15

1. 310д. Cee

x

x

22

2

12 .

311. Cx

x

32

32ln

32

1. 312. .

5

13

5

1C

хarctg

313. Cx

x

2

3ln

5

1. 314. C

x

2

3arcsin . 315д. C

xarctg

3

1.

316д. Cxxx 855,2ln 2 . 317д. Cx

4

1arcsin . 318д. xln

2

1

Cxx 5,342 2 . 319. Cxxx

2sin4

12cos

3.

320.

xx

3sin3

23Cx 3cos

9

1. 321.

Cee

x xx 22

4

1

2

3.

322. Cexxx 222 . 323. Cxxxarcctg 241ln

4

12 .

324. Cxxx 2ln . 325. xxarcsin Cx 21 .

326. Cxxx 4ln2 . 327. Cx

arctgxx

22

1

2

2

.

328. Cx

xx

16

ln4

44

. 329. Cxxxarctg 144

114 . 330. xctgx

Cx sinln . 331д. Cxxx sin2cos32 . 332д.

xxx

2cos4

12sin

2

1

C . 333д. Cexexx . 334д. Cxxx 21arccos .

335д. Cxxxarctg 291ln6

13 . 336д. Cx

xxxx

2ln

22 .

337. Cxx 5ln32ln9 . 338. Сxxx 4ln43ln52ln .

Page 113: М.Н. Середина, Л.Ю. Шипик МАТЕМАТИКАачии.рф/files/2018-11-16-7b4836c8-02e7-466b-aa5c...2018/11/16  · функции нескольких переменных,

113

339. Cxx 5lnln3 2 . 340. Сxarctgxx

33

13ln

2

12ln5 2 .

341. Cxx

x

2ln31

11ln2 . 342д. Cxx 9ln

3

23ln

3

1.

343д. Cxxx 2ln34ln5ln8 . 344д. 3ln12

1ln

21

1xx

.7ln28

1Cx 345д. C

xarctgx

22

31ln . 346д.

1

11ln4

xx

Cx 3ln4 . 347. 4

37 . 348.

3

22 . 349.

2

12 e. 350. 0. 351. 4ln

6

1.

352. 8

1. 353.

3

7. 354.

9

7. 355.

2

1ln . 356д. 61. 357д.

12

5.

358д. e

e13 . 359д.

3

264 . 360д. 4. 361д.

4

1. 362д.

6

. 363д. .1

364д. 7ln3

1. 365.

4

2. 366. 1

2

. 367.

e

21 . 368.

2

3ln

36

932

.

369д. 24

e. 370д. 98ln6 . 371.

3

1. 372.

3

1. 373. Расходится.

374. Расходится. 375. Расходится. 376. 4

3 . 377д. Расходится.

378д. 5

1. 379д. 2 . 380. 4. 381.

4

3. 382. 4,5. 383. 18.

384. 20. 385. e

1

2

3 . 386. 12 e . 387д. 4,5. 388д.

3

121 .

389д. 16. 390д. 6

11 . 391д.

3

11 . 392. 8 . 393.

10

3. 394.

3

4.

395. 7

6;

5

3. 396.

15

416;

3

184. 397д.

15

482. 398д. 72 .

399д. 15

1664 ;

3

368. 400. 12. 401. 3ln . 402.

27

81313 .

403. 5

61 ln . 404.

12

13. 405.

9

7. 406д.

3

13. 407д.

12

5ln

tg .

408д. 4

3. 409д. .78 410. yx

x

z315 2

; xx

z32

.

411. 310xyx

zxyy 1424 2 ; xxyyx

x

z2815 22

.

412. 22

2

yx

x

x

z

; 22

2

yx

y

y

z

. 413. 2

3

yx

y

x

z

; 2

2

yx

yxxy

y

z

.

Page 114: М.Н. Середина, Л.Ю. Шипик МАТЕМАТИКАачии.рф/files/2018-11-16-7b4836c8-02e7-466b-aa5c...2018/11/16  · функции нескольких переменных,

114

414. x

y

x

y

x

zcos

2

; x

ycos

xy

z 1

. 415д. 22

yx

y

x

z

; 22

yx

x

y

z

.

416д. yxx

z

21

1; x

y

z

5 . 417д. x

yxx

z 1sin3 2

;

yyxy

z2cos3

. 418д. 21

1ln

xy

x

u

;

y

u ze

y

x 2 ; zye

z

u 22

.

419д. yzeyzxyz

x

u

sin2 ;

yzxzzxy

ucos2 yz

xze ;

yzxyeyzxyyx

z

u

cos2 . 422. dxyeydzx 3sin dyyeyx

x 23cos .

423.

dyxytgdxx

ydz 492

49cos

92

2

. 424. dydxdz 28 .

425д. .51251524334

dyyxydxyxdz

426д. .3sinln2 22dyctgyxydxyxdz 427д. dydxdz 323 .

428.

y

x

x

x

zln2

1

2222

2

; 3

2

32

yy

x

yx

z

; xyy

x

y

z6

2

2

2

2

.

429. xyx

z6

2

2

; 2

2

3xyx

z

; .4cos162

2

yy

z

430.

22

2

2

2 22

yx

xy

x

z

;

yx

z2

22

2

yx

x

;

.

1222

2

yxy

z

431.

2222

2

4

16

yx

xy

x

z

;

yx

z2

222

22

4

82

yx

xy

;

.

4

42222

2

yx

xy

y

z

432. ;49 572

2

2

xy

eyx

z

2

2

y

z

;49 572 xyex .497 5757

2

xyxy

xyeeyx

z 433д. ;16sin20 3

2

2

yx

x

z

;16sin336 52

2

yx

y

z .16cos30 4

2

yx

yx

z 434д. y

exx

z 22

2

12

;

yex

yx

z 32

4

; .4

2

2y

exy

z

435д. xy

x

z304 3

2

2

; yxy

yx

z8712 2

2

;

.812 22

2

xyxy

z

437. .16;17 438. .3,0;3,0 439. .9;6

440. .2

1;

3

1;

6

5

441д. .5;18 442д. .

2

3;

3

1

443д. .5;27

444д. .8;11;12 445. .82;2max zz 446. .11;4min zz

Page 115: М.Н. Середина, Л.Ю. Шипик МАТЕМАТИКАачии.рф/files/2018-11-16-7b4836c8-02e7-466b-aa5c...2018/11/16  · функции нескольких переменных,

115

447. Экстремумов нет. 448*. .82;2min zz 449д. Экстремумов

нет. 450д. .93;0max zz

451. Х Y Z F 452. Х Y Z F 0 0 0 1 0 0 0 0 0 0 1 0 0 0 1 1 0 1 0 1 0 1 0 0 0 1 1 1 0 1 1 0 1 0 0 0 1 0 0 0 1 0 1 0 1 0 1 0 1 1 0 0 1 1 0 1 1 1 1 0 1 1 1 0

453д. Х Y Z F 454д. Х Y Z F 0 0 0 1 0 0 0 0 0 0 1 1 0 0 1 0 0 1 0 1 0 1 0 1 0 1 1 1 0 1 1 0 1 0 0 1 1 0 0 1 1 0 1 1 1 0 1 0 1 1 0 1 1 1 0 0 1 1 1 0 1 1 1 0

455. Не эквивалентны. 456. а) ;100 б) .180 457. .6840 568. .56

459. .6 460. .12 461д. .216 462д. .4536 463д. .220

464д. .120 465д. а) ;720 б) .120 466. .6,0 467. а) ;15

7 б) ;

3

2 в) ;

3

1

г) .0 468. а) ;6

1 б) ;

18

5 в) .

4

1 469. .1,0 470. а) ;

8

3 б) ;

8

3 в) .

8

1

471. а) ;28

3 б) ;

14

1 в) .

28

5 472. .

969

56 473. .

494

115 474. .

720

1 475. .

75

2

476. .745,0 477. .4

33

478. .

4

1 479. .

15

14 480. .

55

42 481. .485,0

482. .35

4 483. .

24

23 484. а) ;076,0 б) .532,0 485. .

216

1 486. .

36

11

487. .28

15 488. .

30

13 489. .488,0 490. а) ;015,0 б) .6175,0 491. а) ;

120

41

б) .120

79 492. а) ;

25

11 б) ;

75

67 в) .

75

34 493д. ;

45

11 .

45

23 494д. а) ;

16

11

б) .2

1 495д. а) ;

18

5 б) .

12

5 496д. .

33

5 497д. .

7

3 498д. .84,0

Page 116: М.Н. Середина, Л.Ю. Шипик МАТЕМАТИКАачии.рф/files/2018-11-16-7b4836c8-02e7-466b-aa5c...2018/11/16  · функции нескольких переменных,

116

499д. .9

4 500д. .

29

9 501д. .36,0 502д. .

12

1 503д. .

36

19

504д. а) ;288,0 б) ;064,0 в) ;216,0 г) .784,0 505д. а) ;295

188 б) ;

295

96 в) .

295

199

506. .3

1 507. .6605,0 508. .

30

23 509. .5,0 510. .

24

7 511. .33,0

512. .19

7 513. .

41

14 514д. .

25

11 515д. .85,0 516д. .154,0 517д. .64,0

518д. .375,0 519д. .25,0 520. а) ;16

5 б) .

2

1 521. .5904,0 522. .

2187

128

523. .0016,0 524. 13. 525. .04565,0 526. .032375,0 527. .4772,0

528. .99245,0 529. .0892,0 530. .6767,0 531д. .243

64 532д. .08146,0

533д. .0011,0 534д. .004,0 535д. .2823,0 536д. .1008,0 537д. .1804,0

538. xi 0 1 2 3 539. xi 0 1 2

pi 125/216 75/216 15/216 1/216 pi 11/20 2/5 1/20

540. .3,4 541. .3 542. ;4,02 p ;5,1)( XM .1,1

543. ;1)( XM .9,0)( XD 544. .172,0)(;0297,0)(;03,0)( ХXDХМ

545. .44,40)(;4,3)( ZDZМ 546. 127. 547. б) ;4

3 в) .

80

3

548. ;599,0)б .28,0)(;079,0)(;566,2)() XXDХМв 549. 0)( xF

при ,1x )(2

1)( 2

xxxF при ,21 x 1)( xF при .2x

550. а) 0)( xF при ,1x 2)1(9

1)( xxF при ,21 x 1)( xF при

;2x б) 1; в) .9

5 551. .2,0 552. .

3

4)(,

3

16)(,2)( ХXDХМ

553. 0,4. 554. а) 4)( TM , 16)( XD ; б) .4109,0 555. .691,0

556. .9502,0 557. .4435,0 558. %.64,86 559. а) ;8413,0 б) %.45,77

560. ).9,63;1,56(

561д. xi 0 1 2 3 562д. xi 0 1 2

pi 0,027 0,189 0,441 0,343 pi 0,12 0,56 0,32

563д. .9,1 564д. ;9,0)( ХМ ;89,2)( XD .7,1)( X 565д. .6

25

566д. .04,26)(;4,7)( ZDZМ 567д. а) ;294,26 DМ б) ,5М

.24D 568д. а) ;8,0 в) ;2

3 г) .

12

1 569д. б)

6

12)( XM ,

36

11)( XD ,

Page 117: М.Н. Середина, Л.Ю. Шипик МАТЕМАТИКАачии.рф/files/2018-11-16-7b4836c8-02e7-466b-aa5c...2018/11/16  · функции нескольких переменных,

117

;6

11)( X в) .

8

3 570д. а) 0)( xF при ,1x )1(

2

1)( xxF при

,11 x 1)( xF при ;1x б) 0; в) 0,5. 571д. 0)( xf при ],8;2[x

6

1)( xf при .

3

153];8;2[ XPx 572д. ,

3

16)(,6)( XDХМ

.3

4)( Х 573д. .0377,0 574д. ,

8

1)( ХМ ,

64

1)( XD .

8

1)( Х

575д. а) ;8413,0 б) .9544,0 576д. ).360;240( 578. .16,0 580д. .17

Page 118: М.Н. Середина, Л.Ю. Шипик МАТЕМАТИКАачии.рф/files/2018-11-16-7b4836c8-02e7-466b-aa5c...2018/11/16  · функции нескольких переменных,

118

Приложение 1

Таблица значений функции 22

2

1)(

x

ex

0 1 2 3 4 5 6 7 8 9 0,0 0,3989 0,3989 0,3989 0,3988 0,3986 0,3984 0,3982 0,3980 0,3977 0,3973 0,1 0,3970 0,3965 0,3961 0,3956 0,3951 0,3945 0,3939 0,3932 0,3925 0,3918 0,2 0,3910 0,3902 0,3894 0,3885 0,3876 0,3867 0,3857 0,3847 0,3836 0,3825 0,3 0,3914 0,3802 0,3790 0,3778 0,3765 0,3752 0,3739 0,3726 0,3712 0,3697 0,4 0,3683 0,3668 0,3652 0,3637 0,3621 0,3605 0,3589 0,3572 0,3555 0,3538 0,5 0,3521 0,3503 0,3485 0,3467 0,3448 0,3429 0,3410 0,3391 0,3372 0,3352 0,6 0,3332 0,3312 0,3292 0,3271 0,3251 0,3230 0,3209 0,3187 0,3166 0,3144 0,7 0,3123 0,3101 0,3079 0,3056 0,3034 0,3011 0,2989 0,2966 0,2943 0,2920 0,8 0,2897 0,2874 0,2850 0,2827 0,2803 0,2780 0,2756 0,2732 0,2709 0,2685 0,9 0,2661 0,2637 0,2613 0,2589 0,2565 0,2541 0,2516 0,2492 0,2468 0,2444 1,0 0,2420 0,2396 0,2371 0,2347 0,2323 0,2299 0,2275 0,2251 0,2227 0,2203 1,1 0,2179 0,2155 0,2131 0,2107 0,2083 0,2059 0,2036 0,2012 0,1989 0,1965 1,2 0,1942 0,1919 0,1895 0,1872 0,1849 0,1826 0,1804 0,1781 0,1758 0,1736 1,3 0,1714 0,1691 0,1669 0,1647 0,1626 0,1604 0,1582 0,1561 0,1539 0,1518 1,4 0,1497 0,1476 0,1456 0,1435 0,1415 0,1394 0,1374 0,1354 0,1334 0,1315 1,5 0,1295 0,1276 0,1257 0,1238 0,1219 0,1200 0,1182 0,1163 0,1145 0,1127 1,6 0,1109 0,1092 0,1074 0,1057 0,1040 0,1023 0,1006 0,0989 0,0973 0,0957 1,7 0,0940 0,0925 0,0909 0,0893 0,0878 0,0863 0,0848 0,0833 0,0818 0,0804 1,8 0,0790 0,0775 0,0761 0,0748 0,0734 0,0721 0,0707 0,0694 0,0681 0,0669 1,9 0,0656 0,0644 0,0632 0,0620 0,0608 0,0596 0,0584 0,0573 0,0562 0,0551 2,0 0,0540 0,0529 0,0519 0,0508 0,0498 0,0488 0,0478 0,0468 0,0459 0,0449 2,1 0,0440 0,0431 0,0422 0,0413 0,0404 0,0396 0,0387 0,0379 0,0371 0,0363 2,2 0,0355 0,0347 0,0339 0,0332 0,0325 0,0317 0,0310 0,0303 0,0297 0,0290 2,3 0,0283 0,0277 0,0270 0,0264 0,0258 0,0252 0,0246 0,0241 0,0235 0,0229 2,4 0,0224 0,0219 0,0213 0,0208 0,0203 0,0198 0,0194 0,0189 0,0184 0,0180 2,5 0,0175 0,0171 0,0167 0,0163 0,0158 0,0154 0,0151 0,0147 0,0143 0,0139 2,6 0,0136 0,0132 0,0129 0,0126 0,0122 0,0119 0,0116 0,0113 0,0110 0,0107 2,7 0,0104 0,0101 0,0099 0,0096 0,0093 0,0091 0,0088 0,0086 0,0084 0,0081 2,8 0,0079 0,0077 0,0075 0,0073 0,0071 0,0069 0,0067 0,0065 0,0063 0,0061 2,9 0,0060 0,0058 0,0056 0,0055 0,0053 0,0051 0,0050 0,0048 0,0047 0,0046 3,0 0,0044 0,0043 0,0042 0,0040 0,0039 0,0038 0,0037 0,0036 0,0035 0,0034 3,1 0,0033 0,0032 0,0031 0,0030 0,0029 0,0028 0,0027 0,0026 0,0025 0,0025 3,2 0,0024 0,0023 0,0022 0,0022 0,0021 0,0020 0,0020 0,0019 0,0018 0,0018 3,3 0,0017 0,0017 0,0016 0,0016 0,0015 0,0015 0,0014 0,0014 0,0013 0,0013 3,4 0,0012 0,0012 0,0012 0,0011 0,0011 0,0010 0,0010 0,0010 0,0009 0,0009 3,5 0,0009 0,0008 0,0008 0,0008 0,0008 0,0007 0,0007 0,0007 0,0007 0,0006 3,6 0,0006 0,0006 0,0006 0,0005 0,0005 0,0005 0,0005 0,0005 0,0005 0,0004 3,7 0,0004 0,0004 0,0004 0,0004 0,0004 0,0004 0,0003 0,0003 0,0003 0,0003 3,8 0,0003 0,0003 0,0003 0,0003 0,0003 0,0002 0,0002 0,0002 0,0002 0,0002 3,9 0,0002 0,0002 0,0002 0,0002 0,0002 0,0002 0,0002 0,0002 0,0001 0,0001

Page 119: М.Н. Середина, Л.Ю. Шипик МАТЕМАТИКАачии.рф/files/2018-11-16-7b4836c8-02e7-466b-aa5c...2018/11/16  · функции нескольких переменных,

119

Приложение 2

Таблица значений функции dzexФx

z

0

22

2

1)(

х Ф(х) х Ф(х) х Ф(х) х Ф(х) 0,00 0,0000 0,48 0,1844 0,96 0,3315 1,44 0,4251 0,01 0,0040 0,49 0,1879 0,97 0,3340 1,45 0,4265 0,02 0,0080 0,50 0,1915 0,98 0,3365 1,46 0,4279 0,03 0,0120 0,51 0,1950 0,99 0,3389 1,47 0,4292 0,04 0,0160 0,52 0,1985 1,00 0,3413 1,48 0,4306 0,05 0,0199 0,53 0,2019 1,01 0,3438 1,49 0,4319 0,06 0,0239 0,54 0,2054 1,02 0,3461 1,50 0,4332 0,07 0,0279 0,55 0,2088 1,03 0,3485 1,51 0,4345 0,08 0,0319 0,56 0,2123 1,04 0,3508 1,52 0,4357 0,09 0,0359 0,57 0,2157 1,05 0,3531 1,53 0,4370 0,10 0,0398 0,58 0,2190 1,06 0,3554 1,54 0,4382 0,11 0,0438 0,59 0,2224 1,07 0,3577 1,55 0,4394 0,12 0,0478 0,60 0,2257 1,08 0,3599 1,56 0,4406 0,13 0,0517 0,61 0,2291 1,09 0,3621 1,57 0,4418 0,14 0,0557 0,62 0,2324 1,10 0,3643 1,58 0,4429 0,15 0,0596 0,63 0,2357 1,11 0,3665 1,59 0,4441 0,16 0,0636 0,64 0,2389 1,12 0,3686 1,60 0,4452 0,17 0,0675 0,65 0,2422 1,13 0,3708 1,61 0,4463 0,18 0,0714 0,66 0,2454 1,14 0,3729 1,62 0,4474 0,19 0,0753 0,67 0,2486 1,15 0,3749 1,63 0,4484 0,20 0,0793 0,68 0,2517 1,16 0,3770 1,64 0,4495 0,21 0,0832 0,69 0,2549 1,17 0,3790 1,65 0,4505 0,22 0,0871 0,70 0,2580 1,18 0,3810 1,66 0,4515 0,23 0,0910 0,71 0,2611 1,19 0,3830 1,67 0,4525 0,24 0,0948 0,72 0,2642 1,20 0,3849 1,68 0,4535 0,25 0,0987 0,73 0,2673 1,21 0,3869 1,69 0,4545 0,26 0,1026 0,74 0,2703 1,22 0,3883 1,70 0,4554 0,27 0,1064 0,75 0,2734 1,23 0,3907 1,71 0,4564 0,28 0,1103 0,76 0,2764 1,24 0,3925 1,72 0,4573 0,29 0,1141 0,77 0,2794 1,25 0,3944 1,73 0,4582 0,30 0,1179 0,78 0,2823 1,26 0,3962 1,74 0,4591 0,31 0,1217 0,79 0,2852 1,27 0,3980 1,75 0,4599 0,32 0,1255 0,80 0,2881 1,28 0,3997 1,76 0,4608 0,33 0,1293 0,81 0,2910 1,29 0,4015 1,77 0,4616 0,34 0,1331 0,82 0,2939 1,30 0,4032 1,78 0,4625 0,35 0,1368 0,83 0,2967 1,31 0,4049 1,79 0,4633 0,36 0,1406 0,84 0,2995 1,32 0,4066 1,80 0,4641 0,37 0,1443 0,85 0,3023 1,33 0,4082 1,81 0,4649 0,38 0,1480 0,86 0,3051 1,34 0,4099 1,82 0,4656 0,39 0,1517 0,87 0,3078 1,35 0,4115 1,83 0,4664 0,40 0,1554 0,88 0,3106 1,36 0,4131 1,84 0,4671 0,41 0,1591 0,89 0,3133 1,37 0,4147 1,85 0,4678 0,42 0,1628 0,90 0,3159 1,38 0,4162 1,86 0,4686 0,43 0,1664 0,91 0,3186 1,39 0,4177 1,87 0,4693 0,44 0,1700 0,92 0,3212 1,40 0,4192 1,88 0,4699 0,45 0,1736 0,93 0,3238 1,41 0,4207 1,89 0,4706 0,46 0,1772 0,94 0,3264 1,42 0,4222 1,90 0,4713

0,47 0,1808 0,95 0,3289 1,43 0,4236 1,91 0,4719

Page 120: М.Н. Середина, Л.Ю. Шипик МАТЕМАТИКАачии.рф/files/2018-11-16-7b4836c8-02e7-466b-aa5c...2018/11/16  · функции нескольких переменных,

120

Окончание таблицы приложения 2 х Ф(х) х Ф(х) х Ф(х) х Ф(х)

1,92 0,4726 2,18 0,4854 2,52 0,4941 2,86 0,4979 1,93 0,4732 2,20 0,4861 2,54 0,4945 2,88 0,4980 1,94 0,4738 2,22 0,4868 2,56 0,4948 2,90 0,4981 1,95 0,4744 2,24 0,4875 2,58 0,4951 2,92 0,4982 1,96 0,4750 2,26 0,4881 2,60 0,4953 2,94 0,4984 1,97 0,4756 2,28 0,4887 2,62 0,4956 2,96 0,4985 1,98 0,4761 2,30 0,4893 2,64 0,4959 2,98 0,4986 1,99 0,4767 2,32 0,4898 2,66 0,4961 3,00 0,49865 2,00 0,4772 2,34 0,4904 2,68 0,4963 3,20 0,49931 2,02 0,4783 2,36 0,4909 2,70 0,4965 3,40 0,49966 2,04 0,4793 2,38 0,4913 2,72 0,4967 3,60 0,499841 2,06 0,4803 2,40 0,4918 2,74 0,4969 3,80 0,499928 2,08 0,4812 2,42 0,4922 2,76 0,4971 4,00 0,499968 2,10 0,4821 2,44 0,4927 2,78 0,4973 4,50 0,499997 2,12 0,4830 2,46 0,4931 2,80 0,4974 5,00 0,499997 2,14 0,4838 2,48 0,4934 2,82 0,4976 2,16 0,4846 2,50 0,4938 2,84 0,4977

Page 121: М.Н. Середина, Л.Ю. Шипик МАТЕМАТИКАачии.рф/files/2018-11-16-7b4836c8-02e7-466b-aa5c...2018/11/16  · функции нескольких переменных,

121

Литература

1. Баврин, И.И. Основы высшей математики / И.И. Баврин. – Москва: Выс-

шая школа, 2004. – 520 с. 2. Баврин, И.И. Высшая математика / И.И. Баврин. – Москва: ИЦ «Акаде-

мия», 2003. – 616 с. 3. Гмурман, В.Е. Теория вероятностей и математическая статистика: учебное

пособие для вузов / В.Е. Гмурман. − 9-е изд., стер. − Москва: Высшая школа, 2003. – 479 с.

4. Гмурман, В.Е. Руководство к решению задач по теории вероятностей и математической статистике: учебное пособие для студентов вузов / В.Е. Гмурман. − 8-е изд., стер. − Москва: Высшая школа, 2003. − 405 с.

5. Данко, П.Е. Высшая математика в упражнениях и задачах. В 2 ч. Ч. 1: учебное пособие для вузов / П.Е. Данко, А.Г. Попов, Т.Я. Кожевникова. – 6-е изд. – Москва: Издательский дом «ОНИКС 21 век»: Мир и образова-ние, 2002. − 304 с.

6. Задачи и упражнения по математическому анализу для втузов: учебное пособие / Г.С. Бараненков др.; под ред. Б.П. Демидовича. − Москва: АСТ; Астрель, 2003. − 496 с.

7. Индивидуальные задания по высшей математике: учебное пособие. В 4 ч. Ч. 2. Комплексные числа. Неопределенные и определенные интегралы. Функции нескольких переменных. Обыкновенные дифференциальные уравнения / А.П. Рябушко. – 5-е изд. – Минск: Высшая школа, 2011. – 396 с.

8. Корниш-Боуден, Э. Основы математики для биохимиков / Э. Корниш-Боуден; пер. с англ. – Москва: Мир, 1983. − 144 с.

9. Пискунов, Н.С. Дифференциальное и интегральное исчисления для вту-зов. Т. 1: учебное пособие для втузов / Н.С. Пискунов. – 13 изд. − Москва: Наука, 1985. – 430 с.

10. Пискунов, Н.С. Дифференциальное и интегральное исчисление для вту-зов. Т. 2: учебное пособие для втузов / Н.С. Пискунов. – 13 изд. – Москва: Наука, 1985. – 560 с.

11. Сборник задач по высшей математике. 2 курс / К.Н. Лунгу и др.; под ред. С.Н. Федина. – 7-е изд. – Москва: Айрис-пресс, 2011. – 592 с.

12. Шипачев, В.С. Задачник по высшей математике. – Москва: Высшая шко-ла, 2003. − 304 с.

13. Шипачев, В.С. Высшая математика / В.С. Шипачев. – Москва: Высшая школа, 2007. − 479 с.

Page 122: М.Н. Середина, Л.Ю. Шипик МАТЕМАТИКАачии.рф/files/2018-11-16-7b4836c8-02e7-466b-aa5c...2018/11/16  · функции нескольких переменных,

122

Середина Марина Николаевна

канд. техн. наук, доцент

Шипик Людмила Юрьевна

канд. социол. наук, доцент

МАТЕМАТИКА

Практикум

Учебное пособие

Издается в авторской редакции

Подписано в печать 21.10.2013 г. Формат 60×84/16. Усл. п. л. 7,1. Тираж 30 экз. Заказ № 118.

РО и ОП ФГБОУ ВПО АЧГАА

347740, г. Зерноград Ростовской области, ул. Советская, 15.